You are on page 1of 169

Назва наукового напрямку (модуля): Семестр: 12

Внутрішня медицина 6 курс


Опис:
СЗ
Перелік питань:
1. A 34-yr-old woman points to an area of acute epigastric pain with his right index finger. The pain is
worse at night and taking food relieves it. Taking antacids also relieves it. Choose the single most
likely diagnosis
A. Gastric atrophy
B. Acute pancreatitis
C. Basal pneumonia
D. Chronic cholecystitis
E. * Esophagitis
2. A 35 -year-old man complained of squeezed epigastric pain 1 hour after meal and heartburn. She
has been ill for 2 years. On palpation, there was moderate tenderness in pyloroduodenal area. Antral
gastritis was revealed on gastroscopy. Which study can establish genesis of the disease?
A. USD of abdomen
B. Gastrin level in blood
C. Examination of stomach secretion
D. Examination of stomach motor function
E. * Revealing of Helicobacter infection in gastric mucosa
3. A 35 -year-old patient complains on severe weakness and fever. Disease started with pain in the
throat and fever. Physical examination: skin and mucous are pale, ecchymosis, purpura of skin and
subcutaneous tissue. Peripheral blood smear: RBC - 1,5 х 1012 /L, Нв - 50 g/L. Blast – 90 %,.
What is the most likely diagnosis?
A. * Acute leukemia
B. Thrombocytopenic Purpura
C. Hypoplastic Anemia
D. AIDS
E. Leukemoid reaction
4. A 35-year-old man complains to a physician of chronic vague gastric pain of several years'
duration. The pain is sometimes relieved by food. Which of the following is the most appropriate
test to noninvasively determine the H. pylori?
A. Culture of gastric biopsy
B. Rapid urease test
C. Repeat qualitative IgA and IgG antibodies against H. pylori
D. Repeat quantitative IgA and IgG antibodies against H. pylori
E. * Urea breath test
5. A 35-year-old man consults a gastroenterologist because of chronic heartburn for several years. The
heartburn tends to be worse at night, and he frequently tastes refluxed gastric contents when he
goes to bed. He found that his symptoms were a little better when he avoided his customary late
evening alcoholic drink; however, this modest improvement has subsequently deteriorated. The
gastroenterologist performs esophageal manometry with pH monitoring, which demonstrates
decreased pressure of the lower esophageal sphincter and the presence of acid in the esophagus.
Biopsy of the proximal end of one of these fingers shows surface epithelium with regular columnar
cells with small, ovoid nuclei admixed with goblet cells. Which of the following is the most likely
diagnosis?
A. Achalasia
B. * Barrett esophagus
C. Corrosive esophagitis
D. Esophageal adenocarcinoma
E. Zenker diverticulum
6. A 35-year-old patient with alcohol abuse and abnormal diet complains of pain in epigastrium that
occurs in 1-1,5 hours after having meal. Esophagogastroduodeno-scopy: marked hyperemia, small
defects, easy appearance of sores on the mucous membrane in the antral section of the stomach.
What is the most probable reason for this pathology?
A. * Helicobacter pуlori infection
B. Presence of antibodies to parietal cells
C. Alimentary factor
D. Nervous overstrain
E. Toxic action of alcohol
7. A 35-year-old woman who underwent successful therapy for acute leukemia 10 years ago is
seropositive for hepatitis C. Polymerase chain reaction testing confirms the presence of virus in a
blood sample. She has mildly elevated serum transaminases. A liver biopsy discloses moderately
severe chronic hepatitis without fibrosis. The most appropriate therapy is
A. * interferon plus ribavirin
B. acyclovir
C. ribavirin
D. propranolol
E. no therapy is indicated
8. A 35-year-old woman with amenorrhea is found to have an enlarged pituitary glan. Her prolactin
level is 80 ng/L (normal less than 20 ng/L), and her thyrotropin level is 100 mlU/ml (normal, 0.5 to
4.5 mlU/ml). Which of the following is the best treatment option for this patient?
A. Administration of bromocriptine
B. * Administration of L-thyroxine
C. Irradiation of the pituitary gland
D. Resection of the pituitary gland
E. Use of oral contraceptives
9. A 35-yr-old woman with bone pain, drowsiness and thirst: Calcium—3.3, phosphate—0.75,
ALP—190, PTH—low-normal, PTH—activity high, glucose—6 mmol/l. Choose the most likely
A. diagnosis
Primary hyperparathyroidism
B. Tertiary hyperparathyroidism
C. Hypoparathyroidism
D. Hyperthyroidism
E. * Hyperparathyroidism with ectopic PTH
10. A 36-yr-old woman presented with generalised pruritus for 5 months. On examination she was
tanned and there were spider naevi on her chest. The liver was palpable one finger breadth below
the costal margin as well as the tip of the spleen. Choose the single most likely diagnosis from the
list of options above.
A. Wilson's disease
B. * Primary biliary cirrhosis
C. Cholecystitis
D. Haemochromatosis
E. Alcoholic liver cirrhosis
11. A 37-year-old man complains of pains in epigastrium which are relieved by food intake. EGDS
shows antral erosive gastritis, biopsy of antral mucous presents Hеlicobacter Pylori. Diagnosis is:
A. Gastritis of type A
B. Reflux - gastritis

C. Menetrier's gastritis
D. Rigid antral gastritis
E. * Gastritis of type B
12. A 37-year-old man with asthma is evaluated because he continues to have frequent attacks and now
feels his short-acting ?2-agonist is not providing relief. He states he is using his medications,
including a long-acting ?2-agonist inhaler, inhaled high-dose corticosteroids, and a
short-acting ?2-agonist inhaler as rescue medication. He has symptoms daily and nocturnal
symptoms about twice per week. On physical examination, he is in mild respiratory distress. He is
afebrile. Pulse rate is 90/min and regular, respiration rate is 18/min, and blood pressure is 140/85
mm Hg. He has bilateral wheezing. Spirometry shows a forced expiratory volume in 1 sec (FEV1)
65% of predicted; it improves with bronchodilators to 85% of predicted. He has no history of recent
viral upper respiratory infections or rhinitis or symptoms of gastroesophageal reflux disease. Which
of the following is the best next step in this patient’s management?
A. * Observe the patient using the metered-dose inhaler
B. Add a leukotriene inhibitor
C. Switch to an oral ?2-agonist and have the patient return for a pill count
D. Initiate oral prednisone therapy and have the patient return for a pill count
E. Have the patient return with a symptom and treatment log.
13. A 37-year-old patient complains of epigastric pain that occurs in 1-1,5 hours after having meal.
Esophagogastroduodeno-scopy: marked hyperemia, small defects, easy appearance of sores on the
mucous membrane in the antral section of the stomach. What is the most probable reason for this
pathology?
A. * Helicobacter pуlori infection
B. Presence of antibodies to parietal cells
C. Alimentary factor
D. Nervous overstrain
E. Toxic action of alcohol
14. A 37-year-old woman has a 4-month history of epigastric discomfort and heartburn. Symptoms are
usually exacerbated postprandially, especially if she eats spicy foods. The patient denies dysphagia,
weight loss, and decreased appetite. Treatment with a proton pump inhibitor, once daily for 4
weeks, resulted in only minimal improvement. Increasing the medication to twice daily for an
additional 4 weeks did not improve her symptoms, and the patient wants to know what other
management options are available. Referral for which of the following procedures is most
A. * appropriate
Ambulatoryat24-hour
this time?
esophageal pH monitoring
B. Upper endoscopy with esophageal dilation
C. Barium swallow
D. Surgical fundoplication
E. USD
15. A 37-year-old woman has a 4-month history of epigastric discomfort and heartburn. Symptoms are
usually exacerbated postprandially, especially if she eats spicy foods. The patient denies dysphagia,
weight loss, and decreased appetite. Treatment with a proton pump inhibitor, once daily for 4
weeks, resulted in only minimal improvement. Increasing the medication to twice daily for an
additional 4 weeks did not improve her symptoms, and the patient wants to know what other
management options are available. Referral for which of the following procedures is most
A. appropriate at this time?
Upper endoscopy with esophageal dilation
B. Barium swallow
C. Surgical fundoplication
D. USD
E. * Ambulatory 24-hour esophageal pH monitoring

16. A 37-year-old woman sees her physician because of gum bleeding, menorrhagia. Physical
examination reveals petechiae, bruises on her legs. CBC: RBC 2.5 х 1012/L, Hemoglobin 67 g/L,
MCV 64 fl, CI – 0,7, Pl 41 х 109/L, Giant platelets +, WBС 7.9 х 109/L, ESR 7 mm/h, Bleeding
time 17 min. What is the most probably diagnosis?
A. Henoch–Schonlein purpura
B. Thrombotic thrombocytopenic purpura
C. Chronic lymphocytic leukemia
D. Macrocytic hyperchromic anemia
E. * Idiopathic thrombocytic purpura
17. A 37-yr-old woman presents with weight loss, muscular weakness, oligomenorrhea, diarrhea and
blurring of vision. On examination, there is exophthalmos and proximal myopathy. Choose the
most likely diagnosis
A. Thyroglossal cyst
B. * Graves' disease
C. Hypothyroidism
D. Lymphoma
E. Simple goiter
18. A 38-year-old former hemodialysis nurse is seen because of a 6-month history of fatigue and
amenorrhea. On examination she has scleral icterus, a mildly tender liver, and a tibial rash
consistent with erythema nodosum. ALT and AST levels are both in the range increased, while
alkaline phosphatase and serum albumin levels are normal. Hepatitis serologic testing detects
HBsAg and IgG anti-HBcAg. Liver biopsy discloses a mononuclear cell portal infiltrate and
hepatocyte destruction at the periphery of lobules. Which of the following therapeutic strategies is
A. * best?
Administration of interferon, 10 million units three times per week for 4 months
B. Administration of prednisone, 20 to 40 mg/d for 2 months and then a taper based on the response
C. Administration of prednisone, 10 mg every other day for 3 months
D. Administration of acyclovir, 400 mg every 6 h for 2 weeks
E. Administration of low-dose cyclophosphamide, 50 mg/d for 2 months
19. A 38-year-old male insurance agent with a benign past medical history presents to his primary care
physician complaining of indigestion. He notes that “heartburn” has occurred weekly for about 1
year, especially after eating a heavy meal. He has no risk factors for coronary artery disease and
does not complain of weight loss, vomiting, dysphagia, or bleeding. Physical and routine laboratory
examinations are unrevealing. Which of the following is the most appropriate next step?
A. Upper gastrointestinal barium radiography
B. Upper gastrointestinal endoscopy
C. Ambulatory esophageal pH testing
D. * Serology for H. Pylori
E. Prescribe omeprazole
20. A 38-year-old male insurance agent with a benign past medical history presents to his primary care
physician complaining of indigestion. He notes that “heartburn” has occurred weekly for about 1
year, especially after eating a heavy meal. He has no risk factors for coronary artery disease and
does not complain of weight loss, vomiting, dysphagia, or bleeding. Physical and routine laboratory
examinations are unrevealing. Which of the following is the most appropriate next step?
A. Upper gastrointestinal endoscopy
B. Ambulatory esophageal pH testing
C. Upper gastrointestinal barium radiography
D. Prescribe omeprazole
E. * Serology for H. Pylori

21. A 38-year-old man complains of chronic heartburn for several years. The heartburn tends to be
worse at night, and he frequently tastes refluxed gastric contents when he goes to bed. He found
that his symptoms were a little better when he avoided his customary late evening alcoholic drink;
however, this modest improvement has subsequently deteriorated. The esophageal manometry with
pH monitoring demonstrates decreased pressure of the lower esophageal sphincter and the presence
of acid in the esophagus. Biopsy of the proximal end of esophagus shows surface epithelium with
regular columnar cells with small, ovoid nuclei admixed with goblet cells. Which of the following
is the most likely diagnosis?
A. Achalasia
B. * Barrett esophagus
C. Corrosive esophagitis
D. Esophageal adenocarcinoma
E. Zenker diverticulum
22. A 38-year-old man who works as a reporter for a travel magazine comes to his physician because of
the acute onset of jaundice, malaise, and temperatures to 38.5 С .He had returned from Burma 2
weeks ago, where he spent 4 weeks. He says that he abstains from alcohol beverages and does not
take any medications. Laboratory studies show elevated serum aminotransferases, high bilirubin
(both total and direct), and negative serology for hepatitis A virus (HAV) and С virus (HCV)
infection. He was vaccinated for hepatitis В virus (HBV) 3 years ago and is now positive for
anti-HBsAg antibodies. Which of the following serologic markers should be tested as the most
appropriate next step in diagnosis?
A. Anti-HCV IgG antibodies by RIBA
B. Anti-HDV IgG antibodies
C. * Anti-HEV IgM antibodies
D. Anti-HGV IgG antibodies
E. HBsAg
23. A 39-year-old patient complains of weight loss, night sweating, general weakness, fatigue, fever,
gum bleeding. Bone marrow examination reveals blasts with Auer rods 87 %. What is the most
likely diagnosis?
A. * Acute nonlymphoblastic (myeloid) leukemia
B. Erythremia
C. Chronic lymphocytic leukemia
D. Chronic myeloid leukemia
E. Multiple myeloma
24. A 39-yr-old woman presented with jaundice and painless depigmented patches on her hands, neck
and face. On exami_nation multiple spider naevi were found. Choose the single most likely
diagnosis from the list of options above.
A. Haemochromatosis
B. Primary biliary cirrhosis
C. Wilson's disease
D. * Chronic active hepatitis
E. Pancreatic carcinoma
25. A 40-year-old male with long-standing alcohol abuse complains of abdominal swelling, which has
been progressive over several months. He has a history of gastrointestinal bleeding. On physical
exam, there are spider angiomas and palmar erythema. Abdominal collateral vessels are seen
around the umbilicus. There is shifting dullness, and bulging flanks are noted. An important first
step in the patient’s evaluation is
A. * Diagnostic paracentesis
B. UGI series
C. Ethanol level

D. CT scan
E. Esophagogastroduodenoscopy
26. A 40-year-old white female complains of pruritus. She has an elevated alkaline phosphatase and
positive antimitochondrial antibody test. What is the most likely disease?
A. * Primary biliary cirrhosis
B. Sclerosing cholangitis
C. Anaerobic liver abscess
D. Hepatoma
E. Hepatitis C
27. A 40-year-old woman, a nurse, is evaluated because of worsening asthma symptoms. She has had
mild, intermittent asthma since college, for which she has been using an albuterol inhaler as
needed, usually less than once a month. During the past 3 months, she has experienced cough,
tightness of the chest, and wheezing, which improve after the use of inhaled albuterol. She uses the
inhaler twice a day on average and has awakened at least twice a week with nocturnal cough. She
works three consecutive 12-hour day shifts, and the cough is regularly worse at the end of each
shift. During her days off, she has fewer asthma symptoms and feels significantly better by the time
she returns to work. She has a history of allergic rhinitis that has also recently become more
symptomatic. Approximately 6 months ago, she acquired a kitten that sleeps in the bedroom. She
has lived in her home for 6 years, and it is carpeted and has heavy draperies. Chest examination is
notable for good air entry. There are scattered end-expiratory wheezes. In addition to treatment with
inhaled corticosteroids, which of the following interventions is most likely to benefit this patient?
A. * Avoiding exposure to latex products
B. Treatment with an oral antihistamine
C. Getting rid of the kitten
D. Removing the carpets and draperies from her home
E. Treatment with a leukotriene-modifying drug
28. A 40-yr-old diabetic actor is started on Propanolol for stage fright. He collapses after a day
shooting. He has not changed his insulin regimen. Choose the most likely management.
A. Sugary drink
B. Insulin sliding scale, Heparin and 0.45% saline
C. Insulin sliding scale, 0.9% NS and potassium replacement
D. Chest X-ray
E. * 50 ml of 50% dextrose IV
29. A 40-yr-old man presents to his GP complaining of change in appearance and headaches. His brow
is more prominent and his nose had broadened. He states that his shoes are too small and he has
tingling in certain fingers worse at night. Choose the most likely management.
A. Propanolol
B. Calciferol
C. Carbimazole
D. Thyroxine
E. * Octreotide (somatostatin)
30. A 40-yr-old woman has a long history of pruritus, arthralgia and mild jaundice. She presents with
haematemesis and is found to have splenomegaly. Endoscopy shows oesophageal varices. Choose
the single most likely diagnosis from the list of options above. Choose the single most likely
diagnosis from the list of options above.
A. * Portal hypertension
B. Myelofibrosis
C. Infective endocarditis
D. None of above

E. Gilbert's syndrome
31. A 40-yr-old woman presents with a hard, nodular midline neck mass. Blood tests reveal the
presence of antibodies to thyroglobulin. Choose the most likely diagnosis.
A. Thyroglossal cyst
B. * Hashimoto's thyroiditis
C. Toxic multinodular goiter
D. Thyroid storm
E. Graves' disease
32. A 40-yr-old woman presents with a solitary nodule in the right thyroid lob. FNAC suggests
follicular adenoma. Choose the most likely management.
A. * Total thyroid lobectomy
B. Ablative dose of radioactive iodine
C. External beam radiation
D. Chemotherapy
E. Reassure and repeat FNAC in 1 yr
33. A 40-yr-old woman presents with fatigue, dyspnoea, paraesthesiae and a sore red tongue. Her blood
film shows hypersegmented polymorphs, an MCV of > 110 fl and a low Hb. What is your
A. * diagnosis?
Pernicious anaemia
B. Iron-deficiency anemia.
C. Sickle cell anaemia
D. Hemolytic anemia.
E. Aplastic anemia.
34. A 41-year-old morbidly obese female comes to the emergency department with colicky abdominal
pain in her right upper abdomen. She complains that this is similar to, yet more severe than, the
pain that often occurs after meals for the past 4 months. Her past medical history is positive for
diabetes mellitus type 2, hypertension, hyperlipidemia, and smoking. On physical exam, her
temperature is 100.5°F (38.1°C) and her sclera appear mildly icteric. What imaging modality may
be limited in this patient?
A. MRCP.
B. CT scan.
C. * Ultrasonography.
D. Esophogastroduodenoscopy (EGD).
E. IDA scan.
35. A 42-year-old patient complains of weight loss, night sweating, general weakness, fatigue. Physical
examination reveals lymphadenopathy, splenomegaly. CBC reveals: RBC 2.34 х 1012/L,
Hemoglobin 78 g/L, Hematocrit 40 %, MCV 90 fl, Reticulocyte Count 1.0 %, Pl 245 х 109/L,
WBC 71.9 х 109/L, Segmented Neutrophils 2 %, Band Neutrophils 0 %, Monocytes -0%
Eosinophils -0 %, Basophils -0%, Lymphocytes 98%, ESR 24mm/h,Smudge cells +. What is the
most likely diagnosis?
A. Acute leukemia
B. * Chronic lymphocytic leukemia III st
C. Chronic lymphocytic leukemia I st
D. Chronic myeloid leukemia
E. Chronic lymphocytic leukemia IV st
36. A 42-year-old patient complains of weight loss, night sweating. CBC reveals: RBC 3.75 х 1012/L,
Hb 125 g/L, Ht 42 %, MCV 91 fl, -Reticulocyte Count 1.0 %, Pl 280 х 109/L, WBC 47.9 х 109/L,
Segmented Neutrophils 3 %, Band Neutrophils 1 %, Monocytes 2 %, Eosinophils 1 %, Basophils
0 %, Lymphocytes 93 %, ESR 7 mm/h, Smudge cells +. What is the most likely diagnosis?
A. Acute leukemia
B. Erythremia

C. * Chronic lymphocytic leukemia


D. Chronic myeloid leukemia
E. Multiple myeloma
37. A 42-year-old patient complains of weight loss, night sweating. Physical examination reveals
lymphadenopathy, splenomegaly. CBC reveals: RBC 3.75 х 1012/L, Hb 125 g/L, Ht 42 %, MCV
91 fl, -Reticulocyte Count 1.0 %, Pl 280 х 109/L, WBC 47.9 х 109/L, Segmented Neutrophils 3 %,
Band Neutrophils 1 %, Monocytes 2 %, Eosinophils 1 %, Basophils 0 %, Lymphocytes 93 %, ESR
7 mm/h, Smudge cells +. What is the most likely diagnosis?
A. Acute leukemia
B. Chronic lymphocytic leukemia III st
C. * Chronic lymphocytic leukemia II st
D. Chronic myeloid leukemia
E. Chronic lymphocytic leukemia IV st
38. A 42-year-old patient suffering from alco_holism has advanced liver disease with ascites.
Examination reveals asterixis of the hands, ankle clonus, and spider angiomas on the face and
chest. Precipitat_ing factors to look for include all of the fol_lowing EXCEPT:
A. * heart insufficiency
B. bleeding esophageal varices
C. excessive diuretic therapy
D. non-compliance with lactulose therapy
E. spontaneous bacterial peritonitis
39. A 42-yr-old man has hypertension, hyperglycaemia, myopathy, thinning of the skin, buffalo hump
and truncal obesity. Choose the most likely investigation
A. * Dexamethasone suppression test
B. ACTH stimulation test
C. Fasting blood glucose
D. T3, T4 and TSH levels
E. Water deprivation test
40. A 44-yr-old mother of five children complained of yellow skin and abdominal pain, especially after
meals. She was overweight and she said that she did not like going out to restaurants because of
embarrassing flatulence. Choose the single most likely diagnosis from the list of options above.
A. * Gallstones
B. Hepatitis
C. Carcinoma of bile duct
D. Alcoholism
E. Carcinoma pancreas
41. A 44-yr-old woman complains of intense pruritus and yellowing of her skin. On physical
examination you notice xanthomata and skin pigmentation. Choose the single most likely diagnosis
from the list of options above.
A. * Primary biliary cirrhosis
B. Hepatitis
C. Carcinoma of bile duct
D. Alcoholism
E. Haemolytic anaemia
42. A 44-yr-old woman presents with tachycardia, atrial fibrillation, double vision and swelling above
her ankles. She has lid lag on examination. Choose the most likely investigation
A. Dexamethasone suppression test
B. ACTH stimulation test

C. Fasting blood glucose


D. * T3, T4 and TSH levels
E. Water deprivation test
43. A 45 year-old man undergoes a routine physical examination with screening blood studies. Physical
examination is notable for an increased liver diameter; the liver edge is palpable and without
irregularities. Blood studies show elevated liver enzymes. The clinician suspects alcoholic hepatitis.
Which of the following findings would tend to support this diagnosis?
A. Alanine aminotransferase = 2000 U/L
B. * Aspartate aminotransferase (AST)/alanine aminotransferase (ALT) ratio = 2.5
C. Gamma-glutamyl transferase (GGT) = 20 U/L (norm
D. Mean corpuscular volume (MCV) = 65 urn3
E. Platelet count = 600,000/mm3
44. A 45-year-old female with long-standing alcohol abuse complains of abdominal swelling, which
has been progressive over several months. On physical exam, there are spider angiomas and palmar
erythema. Abdominal collateral vessels are seen around the umbilicus. A paracentesis is performed.
The serum albumin minus ascitic fluid albumin equals 1.4 g/dL. The most likely diagnosis is
A. * Portal hypertension
B. Pancreatitis
C. Tuberculous peritonitis
D. Hepatoma
E. No ascitis
45. A 45-year-old man for 1 month has complained of epigastric and right subcostal aching pain,
pruritus, indigestion, dark color of the urine and acholic stool, fever, and significant weight loss. On
exam: jaundice, presence of Curvuasier’s sign. US scan did not reveal stones in the gallbladder and
choledochus. What is the most likely diagnosis?
A. * Cancer of the pancreas head
B. Gallbladder stones
C. Chronic pancreatitis
D. Chronic cholangitis
E. Chronic hepatitis
46. A 45-year-old man with history of high blood pressure that has been difficult to control with a
variety of antihypertensive medications presents with persistent headaches, excessive sweating, and
palpitations. Routine blood tests are normal. Electrocardiogram shows supraventricular tachycardia.
What is this patient’s most likely diagnosis?
A. * Pheochromocytoma
B. Primary hyperaldosteronism
C. Cushing’s disease
D. Hyperthyroidism
E. Malignant hypertension
47. A 45-year-old man with history of high blood pressure that has been difficult to control with a
variety of antihypertensive medications presents with persistent headaches, excessive sweating, and
palpitations. Routine blood tests are normal. Electrocardiogram shows supraventricular tachycardia.
It was suspected pheochromocytoma. What is this patient’s most likely test?
A. cortisol level
B. aldosteron level
C. 17-hydroxycorticosteroids
D. * urine metanephrines
E. TSH

48. A 45-year-old man with history of high blood pressure that has been difficult to control with a
variety of antihypertensive medications presents with persistent headaches, excessive sweating, and
palpitations. Routine blood tests are normal. Electrocardiogram shows supraventricular tachycardia.
Which of the following is the best initial pharmacologic therapy choice for this patient?
A. ?-Blockers
B. Diuretics
C. Steroids
D. Methimazole
E. * ?-Blockers
49. A 45-year-old woman complains of right upper quadrant pain, which occurs after she eats a large
meal. Occasionally the episodes are accompanied by nausea and vomiting. A plain x-ray of the
abdomen discloses gallstones. Ultrasonography reveals gallstones and a normalsized common bile
duct. The patient’s blood chemistry and CBC are normal. The most therapeutic maneuver at this
time would be
A. * laparoscopic cholecystectomy
B. observation
C. ursodeoxycholic acid
D. shock wave lithotripsy
E. ursodeoxycholic acid and shock wave lithotripsy
50. A 45-yr-old patient complains of giddiness and falls. She also suffers with intermittent vomiting
and sweating and occasional faecal incontinence at night. Choose the most likely diagnosis
A. Coronary artery disease
B. Cerebrovascular disease
C. Nephropathy
D. Retinopathy
E. * Autonomic neuropathy
51. A 45-yr-old woman presents with a diffuse swelling of the thyroid gland. On examination she has a
stare, lid lag, and lid retraction. On the dorsum of her legs she has areas of raised peau d'orange-like
thickened skin. Blood tests reveal thyroid-stimulating immunoglobulins against the TSH receptor
site. Choose the most likely diagnosis.
A. Thyroglossal cyst
B. Hashimoto's thyroiditis
C. Toxic multinodular goiter
D. Thyroid storm
E. * Graves' disease
52. A 46-year-old woman who works as a nurse is evaluated because of a 2-year history of episodic
wheezing and a squeaky voice. This past spring, her symptoms worsened, requiring her to seek
medical attention; she was placed on a short-acting ?2-agonist that did not provide much relief. She
has no history of wheezing and says that these changes began after a severe influenza infection 3
years ago. Currently she feels well and has had no symptoms for several months; she is not taking
any medications. Physical examination shows no abnormalities, and baseline spirometry is normal.
What is the best test to evaluate this patient’s condition?
A. * Methacholine challenge testing
B. Bronchoscopy to evaluate her trachea
C. Exercise echocardiogram
D. CT scan of the sinuses
E. Non of above

53. A 47-year-old female presents to the health clinic for symptoms of weight she can't seem to lose
despite strict dieting and concerns of menstrual irregularity. She states she also has developed
multiple abdominal stretch marks and has noticed a deepening of her voice and appearance of facial
hair. As part of her initial work up, you obtain a 24-hour urine cortisol level, which is returned as
120 ng/ml (N 20 100 ng/ml). What is the most likely diagnosis in this patient?
A. Metabolic syndrome.
B. Diabetes mellitus type II.
C. * Cushing's syndrom
D. Addison's diseas
E. Conn's syndrom
54. A 47-year-old female presents to the health clinic for symptoms of weight she can't seem to lose
despite strict dieting and concerns of menstrual irregularity. She states she also has developed
multiple abdominal stretch marks and has noticed a deepening of her voice and appearance of facial
hair. As part of her initial work up, you obtain a 24-hour urine cortisol level, which is returned as
120 ng/ml (N 20 100 ng/ml). The most likely diagnosis in this patient is Cushing's syndrome.What
is the most appropriate next step in diagnosis of this patient?
A. Cosyntropin (ACTH) stimulation test.
B. * Dexamethasone suppression test.
C. Cosyntropin (ACTH) suppression test.
D. Dexamethasone stimulation test.
E. Urine catecholamine collection over a 24-hour perio
55. A 47-yr-old agricultural worker complains of a chronic cough, purulent sputum and abdominal
distention. He has just arrived in England from Spain where he was picking grapes. Choose the
single most likely diagnosis from the list of options above.
A. * Tuberculosis
B. Cirrhosis
C. Malabsorption
D. Pancreatitis
E. Peptic ulcer
56. A 48 years old patient, complaints on weakness, dyspnea, pain in the left half of thorax, permanent
cough with viscid sputum, in which particles of blood are sometimes determined. For the last 3
months lost 5 kg of body mass. On the X-ray of lungs there is total homogeneous shade determined
from the left side. Organs of mediastinum are displaced to the left. What diagnosis is possible?
A. * Lung athelectasis
B. Lung gangrene
C. Total exudative pleurisy
D. Pneumonia
E. Empyema of pleura
57. A 48-year-old patient complains of fatigue and easy bruising of 3 weeks’ duration. Physical
findings included pale, scattered ecchymoses and petechiae. RBC – 1.95 x 1012/L; Hb – 73 g/L;
HCT 20%; PLT – 23 x 109/L; and WBC – 182 x 109/L with 84% blasts, that contained Auric rods;
peroxidase stain was positive; What is the most probable diagnosis?
A. Megaloblastic anemia
B. Hemolytic anemia
C. Thrombocytopenia
D. * Acute myeloid leukemia
E. Chronic leukemia

58. A 48-year-old woman develops fevers, chills, and icteric sclera. In addition to a fever of 39.2_C,
the physical examination is remarkable for an ill-appearing jaundiced female with right upper
quadrant pain. Ultrasonography reveals a dilated common bile duct with stones in the gallbladder
and in the duct itself. The patient is placed on broad-spectrum antibiotics to cover organisms known
to infect the biliary tract. The procedure most appropriate now is
A. * endoscopic retrograde cholangiopancreatography
B. laparoscopic cholecystectomy
C. placement of an external stent for bilary drainage
D. laparotomy to canulate the common bile duct, remove the stone, and perform a cholecystectomy
E. antibiotics for several days
59. A 49 years old woman complains of weakness, malaise, anorexia, fever, dental bleeding. Petechiae
on a skin. Laboratory findings: Hb of 70 g/L, RBC of 2.2 x 1012/L, the platelet count is 30 x 109/L,
the WBC 100,5 x 109/L, blasts in peripheral blood smear. What is the most probable diagnosis?
A. Megaloblastic anemia
B. * Acute leukemia
C. Hemolytic anemia
D. Thrombocytopenia
E. Chronic leukemia
60. A 49-yr-old man who enjoys drinking presents with pallor, epistaxis and bleeding. On physical
examination you find the spleen to be enlarged and the liver to be slightly enlarged. Choose the
single most likely investigation from the list of options above.
A. * Liver function test
B. Abdominal USG
C. Bone marrow biopsy
D. Blood culture
E. White cell count
61. A 49-yr-old woman presents with goiter. On examination, the thyroid is firm and rubbery. Thyroid
microsomal antibodies are positive in high titer. Choose the most likely diagnosis
A. Thyroglossal cyst
B. * Hashimoto's thyroiditis
C. Hypothyroidism
D. Graves' disease
E. Simple goiter
62. A 50 years old woman complains of weakness, anorexia, fever, gums bleeding. Spleen, liver, and
lymphatic nodes are enlarged, petechiae on a skin. Laboratory findings: the platelet count is 90 x
109/L, WBC 100 x 109/L, blasts 40%. ? What is the most probable diagnosis?
A. * Acute leukemia
B. Megaloblastic anemia
C. Hemolytic anemia
D. Thrombocytopenia
E. Chronic leukemia
63. A 50-year-old man without significant past medical history or recent exposure to alcohol presents
with midepigastric abdominal pain, nausea, and vomiting. The physical examination is remarkable
for the absence of jaundice and any other specific physical findings. Which of the following is the
best strategy for screening for acute pancreatitis? Measurement of both serum amylase and serum
A. * Lipase
B. Measurement of serum lipase
C. Measurement of serum amylase
D. Isoamylase level analysis

E. None of above
64. A 50-yr-old patient complains of burning pain in the feet, worse at night or on walking. He
describes the sensation as like walking on hot coals. Choose the most likely diagnosis
A. Coronary artery disease
B. Cerebrovascular disease
C. Nephropathy
D. Retinopathy
E. * Polyneuropathy
65. A 50-yr-old woman presents to her GP for fatigue, depression and weight gain. She also complains
of constipation and poor memory. On examination, she has puffy face and coarse facial features,
thin eyebrows and a large tongue. Choose the most likely management
A. Propanolol
B. Calciferol
C. Carbimazole
D. * Thyroxine
E. Octreotide (somatostatin)
66. A 50-yr-old woman presents with a thyroid goiter. A core biopsy reveals evidence of lymphoma.
Choose the most likely management.
A. Total thyroid lobectomy
B. Thyroxine
C. * External beam radiation
D. Propranolol
E. Reassure and repeat FNAC in 1 yr
67. A 50-yr-old woman presents with fever, tachycardia restlessness, hypertension and vomiting. On
examination she has diffuse swelling of the thyroid gland and strabismus with diplopia. Choose the
most likely diagnosis.
A. Thyroglossal cyst
B. Hashimoto's thyroiditis
C. Toxic multinodular goiter
D. * Thyroid storm
E. Graves' disease
68. A 51-year-old female comes to the emergency department complaining of left lower quadrant
abdominal pain. She describes an acute illness accompanying the pain with subjective fever and
diarrhea over the last 8 hours. Abdominal exam shows tenderness in the LLQ of the abdomen, no
rebound tenderness at McBurney's point, and negative Murphy's sign. What imaging modality is
most appropriate for this patient?
A. Ultrasound of the abdomen.
B. * CT scan with and without contras
C. Colonoscopy.
D. Barium enema.
E. Plain upright abdominal x-ray.
69. A 51-year-old woman undergoes colorectal cancer screening. She feels well, has no significant
medical history, takes no medications, and has no family history of colorectal cancer. Physical
examination and complete blood count are normal. Which of the following is the most appropriate
screening program for colorectal cancer in this patient?
A. * Colonoscopy every 10 years
B. Flexible sigmoidoscopy every 2 to 3 years
C. Barium enema examination every 3 years

D. Fecal occult blood testing every 2 to 3 years


E. CT colonography (virtual colonoscopy) every 10 years
70. A 52-yr-old man has been gaining weight. He complains of a chronic cough, acne and bruising. On
physical examination you find his legs and arms to be abnormally thin. Choose the most likely
investigation
A. Chest X-ray
B. Serum cortisol
C. * Dexamethasone suppression test
D. Water deprivation test
E. T3, T4 and TSH
71. A 52-yr-old man presents with painless lump in the neck and a chronic cough. Physical
examination finds tachycardia and pallor. He feels that he has lost weight, but he is not certain. He
does not smoke or drink. Choose the most likely diagnosis.
A. Toxic adenoma
B. Adrenal hyperplasia
C. Hyperthyroidism
D. Hypothyroidism
E. * Follicular carcinoma
72. A 53-year-old woman with a history of mild persistent asthma is evaluated because of a recent
increase in her symptoms, with dyspnea and cough occurring daily and a cough that awakens her
once a week. She is currently using low-dose inhaled corticosteroids. She has no symptoms of
rhinitis or gastroesophageal reflux. On physical examination, she has intermittent wheezing
bilaterally. Which of the following is the most appropriate change in her therapy?
A. * Add a long-acting ?2-agonist
B. Initiate azithromycin therapy
C. Add a nebulized short-acting ?2-agonist
D. Add inhaled ipratropium bromide
E. Add a leukotriene inhibitor
73. A 55-year-old male alcoholic has recurrent attacks of severe mid-epigastric pain after eating. Serum
amylase determinations after such attacks have been in the normal range. The examination reveals
mild cachexia but is otherwise unremarkable. On further questioning, the patient states that he has
been sober for the past 10 years but prior to that time had multiple episodes of alcohol-induced
pancreatitis. He is currently taking pancreatic replacement enzymes by mouth. An ERCP reveals a
stricture of the pancreatic duct but is otherwise unremarkable. Computed tomography of the
abdomen reveals calcifications in the pancreas but does not show any evidence of malignancy. The
patient is taking 30 mg of continuous-release morphine sulfate twice a day. The best strategy at this
point would be to
A. * resect the head of the pancreas
B. double the dose of pancreatic replacement enzymes
C. double the dose of morphine
D. institute a low-fat diet
E. begin a continuous search for other causes ofabdominal pain
74. A 55-year-old man is evaluated in the emergency department because of an acute, severe asthma
attack; he is hospitalized in the intensive care unit for aggressive medical therapy and monitoring.
He is expectorating thick greenish sputum. His medical history includes hypertension,
cholecystectomy and glaucoma. Chest radiograph reveals hyperinflation only. Medical therapy in
the emergency department included repeated doses of aerosolized albuterol and ipratropium, as
well as methylprednisolone, 125mg administered intravenously. Peak expiratory flow rate is
unimproved at 80 l/min. Which of the following is the most appropriate next step in this patient’s
A. * management?
Intravenous magnesium sulfate

B. Nebulized ipratropium bromide administered by face mask


C. Broad-spectrum antibiotics targeting community-acquired respiratory pathogens
D. Inhaled corticosteroids
75. A 55-year-old man with no prior medical history presents with right flank pain. During an
evaluation for kidney stones, he has a computed tomography scan of the abdomen that reveals a 2 ?
2-cm mass in the left adrenal gland. The patient’s vital signs, kidney function, and electrolyte levels
are within normal limits. Which of the following tests should be ordered next to evaluate the
A. adrenal mass?and renin levels
Aldosterone
B. Random cortisol levels
C. Testosterone levels
D. * 24-Hour urine collection for metanephrines
E. No further testing
76. A 55-yr-old man presents with fever, sweats and weight loss. He also suffers from gout and
enlargement of lymphatic nodes, which are nontender, symmetrical. On examination he has an
enlarged spleen and liver. Blood tests reveal a lymphocytosis and anaemia. The Philadelphia
chromosome is negative. What is the most probable diagnosis?
A. ALL
B. * CLL
C. Multiple myeloma
D. Polycythemia vera
E. CML
77. A 55-yr-old presented with following reports on a routine screen: Calcium-2.85 mmol/l,
phosphate—0.8, ALP—110, PTH—raised, 25-OH vitamin D—low-normal. Choose the most likely
A. * diagnosis
Primary hyperparathyroidism
B. Tertiary hyperparathyroidism
C. Hypoparathyroidism
D. Hyperthyroidism
E. Paget's disease of bone
78. A 56-year-old patient with cirrhosis of the liver presents with massive hemetemesis. Two large-bore
intravenous lines are placed; somatostatin, fluids, and blood products are administered; and the
patient is intubated. Emergency endoscopy reveals bleeding esophageal varices. The patient
becomes stable hemodynamically but is still bleeding. The most appropriate next step is
A. * endoscopic variceal band ligation
B. intravenous vasopressin
C. balloon tamponade
D. endoscopic injection sclerotherapy
E. intravenous propranolol
79. A 57-year-old man with severe persistent asthma is evaluated on routine follow-up. He states that
his asthma has been under good control for the last 3 months on high-dose inhaled corticosteroids
and a long-acting ?2-agonist. He uses a short-acting ?2-agonist only three times per week as a
rescue medication, and he has nocturnal symptoms very rarely. Peak expiratory flows have been
stable. His physical examination is normal, including clear breath sounds, and spirometry is
normal. It is decided that he will keep using the short-acting ?2-agonist as a rescue medication.
Which of the following is the best next step in this patient’s management?
A. * Continue the long-acting ?2-agonist and decrease the dose of inhaled corticosteroid
B. Stop the long-acting ?2-agonist and decrease the dose of inhaled corticosteroid
C. Continue current therapy and have the patient return in 6 months
D. Stop the long-acting ?2-agonist and maintain the dose of inhaled corticosteroid

80. A 58-year-old patient complains of weight loss, night sweating, sensation of fullness in left
hypochondrium. CBC reveals: RBC 3.9х 1012/L, Hb 118g/L, Ht 41%, MCV 88 fl, Pl 645 х 109/L,
WBC 89,4х 109/L, Segmented Neutrophils 62%, Band Neutrophils 10%, Metamyelocytes 3%,
Myelocytes 5%, Monocytes 4%, Eosinophils 8%, Basophils 6%, Lymphocytes 2%, ESR 27mm/h.
What is the most likely diagnosis?
A. Acute leukemia
B. Chronic myeloid leukemia, accelerated phase
C. Chronic lymphocytic leukemia
D. * Chronic myeloid leukemia, chronic phase
E. Multiple myeloma
81. A 58-year-old woman complains of increasing fatigue and easy bruising of 3 weeks duration.
Physical findings included pale, scattered ecchymoses and petechiae and mild hepatosplenomegaly.
CBC: RBC – 2.55 x 1012/L; Hb – 73 g/L; HCT 20%; PLT – 23 x 109/L; and WBC – 162 x 109/L
with 82% blasts; peroxidase stain is positive. What is the most probable diagnosis?
A. Megaloblastic anemia
B. Hemolytic anemia
C. Thrombocytopenia
D. * Acute leukemia
E. Chronic leukemia
F. Chronic hemolytic anemia
82. A 58-yr-old man was diagnosed with diabetes at a routine medical examination 3 months ago. His
BMI is 32 despite losing 5 kg by following the dietician's advice. His home blood glucose readings
range from 7 to 11 and his HbAlc is 10%. Choose the most likely management.
A. * Metformin
B. No change in treatment required
C. IV insulin sliding scale
D. Gliclazide
E. Dietary adjustment
83. A 58-yr-old woman presents with fever, sweats and weight loss. On examination she has an
enlarged spleen. Blood tests reveal a lymphocytosis and anaemia. The Philadelphia chromosome is
negative. What is the diagnosis?
A. ALL
B. Multiple myeloma
C. * CLL
D. Polycythemia rubra vera
E. CML
84. A 59-yr-old man presents with obstructive jaundice. USG shows no gallstones. The liver appears
normal and the common bile duct measures 12 mm in diameter. His past medical history includes
partial gastrectomy 15 yr ago for peptic ulcer. Choose the single most likely investigation from the
list of options above.
A. Percutaneous transhepatic
B. * Cholangiography
C. MRI scan
D. Barium follow through
E. CTscan

85. A 60-year-old alcoholic man is admitted to the emergency department with hematemesis. His pulse
is 110/min, blood pressure is 100/60 mm Hg, and respirations are 19/min. He has multiple spider
angiomata on his back and chest, with bilateral gynecomastia. Abdominal examination is
significant for hepatosplenomegaly, and his abdomen is distended and tympanic on percussion; a
fluid level is easily detectable. His testicles are small, and a rectal examination produces
guaiac-negative stool. His hematocritis 23%. After placement of a nasogastric tube, 400 mL of
bright red blood is evacuated. After initial fluid resuscitation, which of the following is the most
appropriate next step in management?
A. Barium swallow
B. Esophageal balloon tamponade
C. * Esophagogastroscopy
D. Exploratory celiotomy
E. Selective angiography
86. A 60-year-old man with biopsy-proven hepatic cirrhosis is hospitalized because of massive ascites
and pedal edema. There is no evidence of respiratory compromise or hepatic encephalopathy. Bed
rest, sodium and water restriction, and the administration of spironolactone produce no significant
weight change after 5 days. Which of the following therapeutic measures would be most
appropriate at this time?
A. * Therapeutic paracentesis
B. None of them
C. Oral acetazolamide, 250 mg/d
D. Placement of a peritoneovenous shunt
E. Intravenous furosemide, 80 mg now
87. A 60-yr-old man is brought to A&E in an unconscious state. His glucose is 35 mmol/l. His arterial
blood gas shows a pH of 7.2 and a PaCO2 of 28 mm Hg. Serum Na is 140, K is 3.0, Cl is 100 and
HCO3" is – 18 mEq/L. Choose the most likely management.
A. Sugary drink
B. Insulin sliding scale, Heparin and 0.45% saline
C. * Insulin sliding scale, 0.9% NS and potassium replacement
D. Chest X-ray
E. 50 ml of 50% dextrose IV
88. A 61-year-old patient complains of weight loss, night sweating, general weakness, fatigue,
sensation of fullness in left hypochondrium. CBC reveals: RBC 4.0х 1012/L, Hb 88g/L, Hematocrit
34%, MCV 88 fl, Pl 98 х 109/L, WBC 125,4х 109/L, Segmented Neutrophils 35%, Band
Neutrophils 8%, Metamyelocytes 6%, Myelocytes 5%, Myeloblasts 10%, Monocytes
4%,Eosinophils 8% Basophils 22%, Lymphocytes 2%, ESR mm/h 27mm/h. What is the most likely
A. diagnosis?
Acute leukemia
B. * Chronic myeloid leukemia, accelerated phase
C. Chronic lymphocytic leukemia
D. Chronic myeloid leukemia, chronic phase
E. Multiple myeloma
89. A 61-year-old white man has a 2-month history of dysphagia for solid foods. He has lost 6.7 kg
during this time. The patient has chronic heartburn that is relieved by antacids. He also has
hypertension for which he takes atenolol and diltiazem. Which of the following is the most likely
diagnosis
A. * Esophageal adenocarcinoma
B. Pill-induced esophagitis
C. Esophageal web
D. diffuse esophageal spasm
E. None of above

90. A 64-year-old patient complains of weight loss, night sweating, general weakness, fatigue, nose
bleeding. Physical examination reveals lymphadenopathy, ecchymosis. CBC reveals: RBC 3.00 х
1012/L, Hb 98 g/L, Ht 40 %, MCV 98 fl, Reticulocyte Count 1.0 %, Pl 45 х 109/L, WBC 61.9 х
109/L, Segmented Neutrophils 2 %, Band Neutrophils 1 %, Monocytes -0% Eosinophils -0 %,
Basophils -0%, Lymphocytes 97%, ESR 24mm/h, Smudge cells ++. What is the most likely
diagnosis?
A. Acute leukemia
B. Chronic lymphocytic leukemia III st
C. Chronic lymphocytic leukemia I st
D. Chronic myeloid leukemia
E. * Chronic lymphocytic leukemia IV st
91. A 65-yr-old man has had type 2 diabetes for 4 years, for which he was taking Chlorpropamide. He
presents with an acute MI and his laboratory blood glucose is 11 mmol/l. Choose the most likely
management.
A. Metformin
B. No change in treatment required
C. * IV insulin sliding scale
D. Gliclazide
E. Dietary adjustment
92. A 65-yr-old patient is becoming increasingly confused. She has periods where her confusion seems
to be stable and then seems to rapidly deteriorate in a stepwise progression. On examination there
are extensor plantars but leg reflexes are diminished. Choose the most likely diagnosis
A. Coronary artery disease
B. * Cerebrovascular disease
C. Nephropathy
D. Retinopathy
E. Polyneuropathy
93. A 66-year-old man complains of fever, significant weight loss, bone and joint pain, and bleeding
gums. On examination - paleness, lymphadenopathy, hepato- and splenomegaly. WBC – 170 x
109/L with 13% lymphocytes, 1% monocytes, 21% basophiles, 29% neutrophils, 10% blasts, 12%
promyelocytes, 12% myelocytes, 2% metamyelocytes. ESR – 22 mm/h. The Philadelphia
chromosome positive. Which agent is used for induction or consolidation therapy?
A. Prednisolone
B. * Сytosar
C. Gatifloxacin
D. Vinblastine
E. Alendronic acid
94. A 67-year- male presents with a complaint of fatigue. There is no history of alcohol abuse or liver
disease. Scleral icterus is noted on physical exam. The liver and spleen are nonpalpable. The patient
is noted to have a normocytic, normochromic anemia. The first step in evaluation of this patient is
A. CT scan of the abdomen
B. Hepatitis profile
C. * Liver function tests, including direct versus indirect bilirubin and urine bilirubin
D. Abdominal ultrasound
E. Percutaneous transhepatic Cholangiography

95. A 67-year-old alcoholic man is admitted to the emergency department with hematemesis. His pulse
is 100/min, blood pressure is 100/60 mm Hg, and respirations are 19/min. Abdominal examination
is significant for hepatosplenomegaly, and his abdomen is distended and tympanic on percussion; a
fluid level is easily detectable. His testicles are small, and a rectal examination produces
guaiac-negative stool. His hematocritis 24%. After placement of a nasogastric tube, 420 mL of
bright red blood is evacuated. After initial fluid resuscitation, which of the following is the most
appropriate next step in management?
A. Barium swallow
B. Esophageal balloon tamponade
C. * Esophagogastroscopy
D. Exploratory celiotomy
E. Transjugular intrahepatic portosystemic shunt
96. A 67-year-old male presents with conjugated hyperbilirubinema, with bilirubin detected in the
urine. Serum bilirubin is 12 mg/dL, AST and ALT are in normal range, and alkaline phosphatase is
300 U/L (3 times normal). The next step in evaluation is
A. * Ultrasound or CT scan
B. Hepatitis profile
C. Reticulocyte count
D. Family history for hemochromatosis
E. Colonoscopy
97. A 67-year-old woman had her first colonoscopy 1 month ago for routine colorectal cancer
screening. A 6-mm tubular adenoma of the sigmoid colon was removed. She has no family history
of colorectal cancer. She asks what can be done to decrease her risk of developing colorectal
cancer. Which of the following is the most appropriate surveillance for this patient
A. * Repeat colonoscopy in 5 year
B. Repeat colonoscopy in 1 years
C. Aspirin, 81 mg daily
D. A high-fiber, low-fat diet
E. None of above
98. A 67-yr-old man is noted to have a glucose level of 37 mmol/l and a Na+ of 163 mmol/l. He has no
prior history of diabetes and has been on IV fluids for a week. His other medications include IV
Cefuroxime, Metronidazole, and Dexamethasone. Choose the most likely management.
A. Insulin sliding scale, Heparin and 0.9% saline
B. * Insulin sliding scale, Heparin and 0.45% saline
C. Sugary drink
D. Chest X-ray
E. 50 ml of 50% dextrose IV
99. A 68-year-old man is evaluated because of nausea, vomiting, and upper abdominal pain and
distention of 2 days duration. He has no fever, chills, or jaundice. On physical examination, he
appears uncomfortable and has orthostatic hypotension. Abdominal examination discloses
distention, tympany on percussion, and rushes on auscultation. Serum aspartate aminotransferase is
0,34 U/L, serum alanine aminotransferase is 0,61 U/L, and serum total bilirubin is 19,6 mcmol/L.
Plain radiographs of the abdomen show pneumobilia with multiple air-fluid levels in the jejunum.
No free air is seen. Abdominal ultrasonography shows four gallstones measuring 2 to 4 cm.
Because of pneumobilia, the biliary tree cannot be further visualized. Which of the following is the
most appropriate next step in this patient’s management?
A. * Exploratory laparotomy for bowel obstruction
B. Cholecystectomy
C. Endoscopic retrograde cholangiopancreatography
D. CT scan of the abdomen

E. Magnetic resonance cholangiopancreatography


100. A 68-year-old patient complains of weight loss, night sweating, sensation of fullness in left
hypochondrium. CBC reveals: RBC 3.9х 1012/L, Hb 128g/L, Ht 41%, MCV 78fl, Reticulocyte
Count 1.0 %, Pl 945 х 109/L, WBC 125,4х 109/L, Segmented Neutrophils 62%, Band Neutrophils
10%, Metamyelocytes 3%, Myelocytes 5%, Monocytes 4%, Eosinophils 8%, Basophils 6%,
Lymphocytes 2%, ESR 27mm/h. What is the most likely diagnosis?
A. Acute leukemia
B. Erythremia
C. Chronic lymphocytic leukemia
D. * Chronic myeloid leukemia
E. Multiple myeloma
101. A 68-yr-old patient presents with bone pain, anaemia and renal failure. Her bone marrow reveals
32 % of plasma cells. What is the most probable diagnosis?
A. * Multiple myeloma
B. Myeloid metaplasia
C. AML
D. CLL
E. ALL
102. A 70-year-old male presents with a complaint of fatigue. There is no history of alcohol abuse or
liver disease; the patient is on no medication. Scleral icterus is noted on physical exam. There is no
evidence for chronic liver disease on physical exam, and the liver and spleen are nonpalpable. The
patient is noted to have a normocytic, normochromic anemia. The first step in evaluation of this
patient is
A. CT scan of the abdomen
B. Hepatitis profile
C. * Liver function tests, including direct versus indirect bilirubin and urine bilirubin
D. Abdominal ultrasound
E. Esophagogastroduodenoscopy
103. A 70-yr-old man presents with bone pain, anaemia. His bone marrow reveals plasma cells 30 %.
What is the most probable diagnosis?
A. Myeloid metaplasia
B. AML
C. * Multiple myeloma
D. CLL
E. Megaloblastic anaemia
104. A 70-yr-old woman develops severe hip pain while gardening. Hip joint X-ray shows fracture neck
of femur. She gives a history of lower back pain and malaise. She is tender over the lumbar spine.
ESR is 110 mm/hr. What is your diagnosis?
A. * Multiple myeloma
B. Waldenstorm's macroglobulinaemia
C. CML
D. CLL
E. Acute leukemia
105. A 70-yr-old woman presents to her GP with weight loss and depression. On examination she is
noted to have buccal pigmentation and pigmented scars. She appears dehydrated. Her BP is 100/60
mm Hg. Choose the most likely management.
A. * Long-term replacement with glucocorticoids and mineralocorticoids
B. Calciferol
C. Carbimazole

D. Thyroxine
E. Octreotide (somatostatin)
106. A 71-year-old male presents with a complaint of fatigue. There is no history of alcohol abuse or
liver disease; the patient is on no medication. Scleral icterus is noted on physical exam. The patient
is noted to have conjugated hyperbilirubinema, with bilirubin detected in the urine. Serum bilirubin
is 12 mg/dL, AST and ALT are in normal range, and alkaline phosphatase is 300 U/L (3 times
normal). The next step in evaluation is
A. * Ultrasound or CT scan
B. Hepatitis profile
C. Reticulocyte count
D. Family history for hemochromatosis
E. Esophagogastroduodenoscopy
107. A 75-year-old man has been deferring colon cancer screening because she is afraid to undergo
colonoscopy. She learned of a new technique called virtual colonoscopy that she thinks may be
more tolerable and asks you about the relative merits of this procedure. Which of the following
statements is true regarding virtual colonoscopy?
A. * It detects colorectal cancers and large adenomas quite well, but may miss small polyps
B. It is more acceptable to patients because it does not require any bowel preparation
C. It is a noninvasive procedure that images the colon using ultrasound
D. Its sensitivity and specificity for detecting colon cancers and polyps is similar to that of
conventional colonoscopy
E. It does not require any instrumentation of the bowel
108. A 76-year-old man, who is in a rehabilitation facility after fracturing a leg, develops acute diarrhea.
He has no history of intestinal disorders. The patient has regular colonoscopic screenings. His last
colonoscopy, done 2 years ago, was normal. He has not received antibiotics in the past year. His
roommate is taking oral metronidazole for a diarrheal syndrome that he developed while in the
rehabilitation facility. Use of which of the following would most likely have prevented
development of the patients diarrheal syndrome?
A. * Good hand-washing technique
B. Prophylactic antibiotics
C. Prophylactic probiotic agents
D. Prophylactic loperamid
E. None of above
109. A man of 59 years old complaints of abdominal discomfort, gum bleeding, large ecchymosis,
weakness, sternal tenderness, fever, skin nodules. Laboratory findings: the white cell count 540 x
109/L, levels of basophils, eosinophils and platelets are increased; and a few normoblasts are seen;
Er – 3,1 x 1012/L, blast 48%. What is the most probable diagnosis?
A. Megaloblastic anemia
B. Hemolytic anemia
C. * Acute leukemia
D. Thrombocytopenia
E. Chronic leukemia
110. A man, 35years old, complains of weakness, palpitation, tinnitus dizziness. Data of anamnesis:
peptic gastric ulcer, repeated bleeding. Physical examination: skin is pale. Systolic murmur is heard
at the apex, HR 100 per min, BP 100/70 mm of Hg. CBC: ESR – 2.8 x 1012/L, haemoglobin
content - 69 gr/l, colour index is 0,7. What is your treatment?
A. Iron parenteral
B. Folic acid
C. * iron supplementation oral

D. Vit B12
E. Antibiotics
111. A middle-aged woman complains of irritability and weight loss. She says she has palpitations. On
physical examination you find mild tachycardia and goiter. There are no eye changes. A thyroid
scan determines a single hot nodule. Choose the most likely diagnosis.
A. * Toxic adenoma
B. Adrenal hyperplasia
C. Hyperthyroidism
D. Hypothyroidism
E. Follicular carcinoma
112. A nursing student has just completed her hepatitis B vaccine series. On reviewing her laboratory
studies (assuming she has no prior exposure to hepatitis B), you expect
A. Positive test for hepatitis B surface antigen
B. * Antibody against hepatitis B surface antigen (anti-HBS) alone
C. Antibody against hepatitis core antigen (anti-HBC)
D. Antibody against both surface and core antigen
E. Antibody against hepatitis E antigen
113. A paracentesis is performed on 43-year-old patient with long-standing alcohol abuse. On physical
exam, there are spider angiomas and palmar erythema. Abdominal collateral vessels are seen
around the umbilicus. The serum albumin minus ascitic fluid albumin equals 1.4 g/dL. The most
likely diagnosis is
A. * Portal hypertension
B. Pancreatitis
C. Tuberculous peritonitis
D. Hepatoma
E. No ascitis
114. A patient presents to a physician with severe jaundice. Physical examination reveals a nodular,
enlarged liver. In addition to the generalized nodularity of the liver, the physician can feel one
nodule that is much larger than the others. CT of the abdomen confirms multinodular cirrhosis and
demonstrates a 7-cm mass near the lower border of the liver. CT-guided biopsy of this mass shows
a malignant tumor derived from hepatic parenchymal cells. Which of the following risk factors is
most strongly associated with the development of this tumor?
A. Anatoxin exposure
B. Hemochromatosis
C. * Hepatitis В virus infection
D. Opistharchis infection
E. Thorotrast exposure
115. A patient who has long-standing diabetes mellitus and severe, burning pain in the feet and hands as
a result of peripheral neuropathy asks the physician why an antidepressant has been prescribed.
What is the physician’s best response?
A. “Many people experiencing chronic pain become depressed.”
B. * “The antidepressants may counteract the chemicals causing your pain.”
C. “You are less likely to become addicted from using antidepressants than you are from using other
types of pain killers.”
D. “The antidepressants also have strong anti-inflammatory properties and can reduce the pain you
have from inflammation.”
E. None of above

116. A patient who is 2 days postoperative from a bowel resection tells her physician that she is having a
hard time “catching her breath,” feels nauseated, and has chest pains when she inhales. The
physician suspects that she is having a pulmonary embolism. What intervention should the
physician perform before notifying the physician?
A. Increase the IV flow rate
B. * Apply oxygen by mask or nasal cannula at 5 l/min
C. Assess the chest and axillary area for the presence of petechiae
D. Place the patient in shock position, with her head and neck flat and her legs elevated
E. Non of above
117. A woman 22 year-old, complains of general weakness, shortness of breath, brittle nails, hair loss.
She has menorrhagia. Physical examination reveals paleness of skin, systolic murmur on
auscultation. What changes do you expect in her blood analysis?
A. Increased level of serum iron
B. High colour index
C. * Decreased level of serum iron
D. Increased level of free bilirubin
E. Decreased amount of thrombocytes
118. A young man presents with a neck lump. It is painless and had been bothering him for the past 4
months. He has no other symptoms or signs. On palpation you find the lump to be single discrete
not particularly hard and confined to the thyroid gland itself. His cervical lymph nodes are
enlarged. Choose the most likely diagnosis.
A. Toxic adenoma
B. Adrenal hyperplasia
C. Hyperthyroidism
D. Hypothyroidism
E. * Papillary carcinoma
119. A young woman complains of wheeze, dyspnoea and cough. She cannot sleep at night because of a
chronic cough. She and her mother love animals and together they have 14 cats. Her PEFR is
normal but her CXR suggests hyperinflation. What is the previous diagnosis?
A. * Bronchial asthma
B. Bronchogenic carcinoma
C. Emphysema
D. Respiratory failure
E. Bronchitis
120. After emotional exertion patient of 24 y.o. developed dyspnea with prolonged expiration, distant
wheezes, frequent night symptoms. Such changes limit his physical activity. PEV and FEV1 <
60 %, daily variability -30 %. What diagnosis is possible?
A. * Severe persistent bronchial asthma
B. Mild persistent bronchial asthma
C. Persistent bronchial asthma of moderate severity
D. Intermittent bronchial asthma
E. Bronchospastic syndrome of allergic origin
121. An 14-yr-old boy presents with painful bones, jaundice and anaemia. He is noted to have
splenomegaly. His blood film reveals target cells. What is your diagnosis?
A. Hemolytic anemia.
B. * Sickle cell anaemia
C. Iron-deficiency anemia.
D. B12-deficiency anemia

E. Aplastic anemia.
122. An 18-yr-old girl complains of her appearance. She is much too fat, she says. She also complains of
missed periods and hairiness. On physical examination you find her to be 10 kg overweight.
Choose the most likely investigation
A. Abdominal ultrasonography
B. Serum cortisol
C. * Dexamethasone suppression test
D. Water deprivation test
E. T3, T4 and TSH
123. An 69 year old asymptomatic woman was detected to have a monoclonal spike on serum
electrophoresis (IgG levels 1.5 g/dl). Bone marrow revealed plasma cells of 30%. What is the
possible diagnosis?
A. * Multiple myeloma
B. Indolent myeloma
C. Monoclonal gammopathy of unknown significance
D. Waldenstorm's macroglobulinemia
E. Amyloidosis
124. Drug abuser, a 41-yr-old man, presents with fever, cough and breathlessness. This was preceded by
viral influenza. Chest radiograph shows multiple abscesses. What is the most possible etiology of
disease?
A. * Staphylococcus aureus
B. Cryptococcus
C. Streptococcus pneumoniae
D. Legionella pneumonia
E. Mycobacterium avium
125. Female B., 44 years old, complains on cough with mucous sputum, increase of temperature to
39 °С, weakness, dyspnea, sweating. Breathing rate - 26/min, skin is moist. Below left scapula
there is shortening of percussion sound. Breathing during auscultation is weakened, moist rales.
Blood test: L - 11х109/l, ESR - 29 mm/h. Your previous diagnosis?
A. * Left-side lower lobe pneumonia
B. Gangrene of lungs
C. Left-side exudative pleurisy
D. Cancer of left side lower lobe
E. Pulmonary abscess
126. Female patient K., 46 years old, has flue with fever. After a while she noticesthoracic pain, cough
with yellow-green sputum (amount-150 ml a day), sometimes with some blood. Objectively:
breathing rate - 36/min. In lungs from the right side lower scapula there is dull sound during
percussion, hard breathing, and moist rales. Blood test: L - 18,6х109/l, ESR -64 mm/h. Analysis of
sputum: L -80-100 , Er - 40-50, elastic fibers, cocci. X-ray: lung roots are enlarged, from the right
side lower lobe is heterogeneously infiltrated with two lighter areas. What is the most possible
previous diagnosis?
A. * Right-side pneumonia with abscesses
B. Peripheral cancer
C. Infiltrative tuberculosis in the phase of disintegration
D. Exudative pleurisy
E. Infarction-pneumonia

127. Female, 34 years old, has an increase of body temperature to 38 °С, cough with purulent sputum,
weakness, dyspnea, pain in a thorax during breathing. During percussion there is shortening of
sound in the lower part of left lung, during auscultation – moist rales. What method of investigation
is the decisive one to confirm diagnosis?
A. * X-ray examination
B. Bacteriological analysis of sputum
C. Spirometry
D. Pneumotachometry
E. Bronchography
128. Girl, 18 y.o., pets seller, complaints mainly during working time on the attacks of dry cough,
running nose. She often suffers from acute viral respiratory infections. Her mother is ill with
bronchial asthma. Objectively: breathing rate - 18/min. Heart rate - 80/min, BP - 110/70. In lungs
vesicular breathing, dry wheezes are heard in distance. Tones of heart are weaker than normally.
Test with berotec showed reversibility of bronchial obstruction. What tactic will be the best for the
A. * patient?
To change job
B. To use intal
C. To use monteleucast
D. To use berotec constantly
E. To use antihystaminic preparations
129. Girl, 23 y.o., for 2 years is ill with bronchial asthma. Recently attacks of dyspnea became more
frequent and started to arise 4-5 times a week, night attacks - 2-3 times a month. She used
salbutamol to remove the symptoms. Test with the antigen of home dust is positive. Objectively:
condition is satisfactory. Breathing rate - 20/min. Heart rate - 76/min, BP -120/80. In lungs
breathing is vesicular. Tones of heart are a little weak, rhythm is normal. What mechanism is
desicive in development of bronchial obstruction in this case?
A. * Hyperreactivity of bronchi
B. Тrachео-bronchial dyskinesia
C. Violation of metabolism of arachidonic acid
D. Adrenergic disorders
E. Activity of the parasympathetic nervous system is increased
130. In the biochemical profile of patient B., 29 yr-old, it is elevated serum T4 and low radioactive
iodine uptake. Choose the most likely diagnosis.
A. Non-toxic goitre
B. Hashimoto's thyroiditis
C. * Subacute thyroiditis
D. Hypothyroidism
E. Graves' disease
131. In the biochemical profile of patient J., 38 yr-old, who has a neck mass, it is normal T3 and T4.
Choose the most likely diagnosis.
A. * Non-toxic goitre
B. Hashimoto's thyroiditis
C. Subacute thyroiditis
D. Hypothyroidism
E. Graves' disease
132. In the biochemical profile of patient L., 56 yr-old., it is normal TSH, free T4 and T3. Decreased
serum total T4. Choose the most likely diagnosis.
A. Non-toxic goitre
B. Hashimoto's thyroiditis
C. * Thyroid binding globulin deficiency

D. Hypothyroidism
E. Graves' disease
133. In the biochemical profile of patient S., 43 yr-old, it is elevated serum T4 and increased radioactive
iodine uptake. Choose the most likely diagnosis.
A. Non-toxic goitre
B. Hashimoto's thyroiditis
C. Subacute thyroiditis
D. Hypothyroidism
E. * Graves' disease
134. Male patient F., 48 years old, during a week stayed at home with diagnosis of respiratory viral
infection. Doctor noticed complaints on cough with small amount of mucus-purulent sputum,
weakness. Objectively: condition is relatively satisfactory. T - 37,2 °С. Breathing rate - 18/min.,
pulse - 80/min., BP - 110/70. In lungs there is vesicular breathing, with a hard tint, single dry
wheezes. Tones of heart are a little dull, rhythm is correct. What is the treatment tactic?
A. * To prescribe antibacterial therapy
B. To stay at home for some more days
C. To go to work
D. To send patient to pulmonologist
E. To hospitalize patient to the pulmonological department
135. Male patient G., 56 years old, complaints on permanent pain in a thorax which disturbs for last 2
months. Pain is not connected with breathing. There is also cough with particles of blood in
sputum. Weakness, fatigue are present. On the chest X-ray in the lower lobe of right lung there is
spherical shadow, size 4x6 cm, related to the lung root. What is the most possible diagnosis?
A. * Perypheral lung cancer
B. Tuberculoma
C. Metastasis
D. Pulmonary abscess
E. Pneumonia
136. Man, 32 y.o., complaints on attack of expiratory dyspnea, which lasts for 48 hours, cough with
small amount of sputum. He is ill with bronchial asthma for 5 years, was treated with
glucocorticosteroids, used inhalers. Objectively: condition is severe, patient sits. Diffuse cyanosis,
pulse -110/min, BP - 110/70. Tones of heart are weak, II tone is louder above the pulmonary artery.
During percussion in lungs there is “bang-box” sound, large amount of dry wheezes. In blood there
is eosinophylia - 18 %. What medicines are drugs of choice for this patient?
A. * Corticosteroids
B. ?2-agonists
C. Theophyllin
D. Cholynolytics
E. Antihystamines
137. Man, 39 y.o., 8 last years is ill with bronchial asthma. Rapidly during physical work he felt
worsening of breathing, cough, distance wheezes appeared and dyspnea began to increase.
Medicine of what pharmacological group is it better to recommend for the patient to remove such
attacks of dyspnea?
A. * Agonists of ?2-adrenoreceptors
B. Methylxantines
C. ?2-adrenoblockers
D. Inhalated glucocorticoids
E. Oral glucocorticoids

138. Man, 43 y.o., complaints on dyspnea during physical activity. Objectively: temperature 36,4 °С,
breathing rate - 20/min, pulse - 78/min, BP-125/80. Emphysematous form of thorax. In lungs –
weak vesicular breathing. What test should be passed by patient at home to decide question about
efficiency of prescribed broncholytics?
A. * Peakflowmetry
B. Spirography
C. ECG-control of overload of right chambers of heart
D. Bronchoscopy
E. Analysis of sputum (amount and microscopy)
139. Man, 46 y.o., suffers for the last 10 years from bronchial asthma. Rapidly during physical work he
felt worsening of breathing, cough, distance wheezes appeared and dyspnea began to increase.
Medicine of what pharmacological group is it better to recommend for the patient to remove such
attacks of dyspnea?
A. * Agonists of ?2-adrenoreceptors
B. Atropine
C. Intal
D. Epinephrine
E. Monteleucast
140. On a routine blood examination a 43-yr-old woman is found to have very high serum calcium level.
She has complained recently of bouts of abdominal pain and recurrent UTI. On physical
examination you find an enlarged thyroid gland. Choose the most likely diagnosis.
A. Toxic adenoma
B. Adrenal hyperplasia
C. Hyperthyroidism
D. Hypothyroidism
E. * Parathyroid carcinoma
141. Over a 2-month period, a 50-year-old woman with a history of polycythemia vera develops
abdominal pain and gross ascites. Physical examination demonstrates smooth hepatomegaly and
mild jaundice. Pressure applied over the liver fails to distend the jugular veins. The abdominal wall
is grossly edematous and shows a tortuous venous pattern. Edema of the legs is prominent. Which
of the following is the most likely diagnosis?
A. Hepatocellular carcinoma
B. Primary sclerosing cholangitis
C. Steatosis
D. * Budd-Chiari syndrome
E. Hepatic cirrhosis
142. Patient A., 35 y.o., noticed infrequent (rarer than 1 time a week) attacks of dyspnea, which are
easily removed with inhalations of ?2-agonists of short action. During attack in lungs are heard dry
wheezes, between attacks FEV1 is more than 80 % from normal. What is the diagnosis?
A. * Intermittent bronchial asthma
B. Persistent bronchial asthma of moderate severity
C. Mild persistent bronchial asthma
D. Severe persistent bronchial asthma
E. Given information is not enough for determination of severity of bronchial asthma
143. Patient A., 45 years old, suffers from chronic cholecystits during last 5 years. She was admitted to
the emergency department with acute pain in right hypochondriac area and high temperature.
Laboratory findings reveal leucocytosis, high ESR. Put preliminary diagnosis.
A. * Chronic cholecystitis, acute phase.
B. Chronic cholecystitis, subacute phase.

C. Chronic cholecystitis, phase of remission.


D. Dyskinezia of bile ducts.
E. Rotor’s syndrome.
144. Patient B., 25 years old engineer, appeared during a fire in the area of high concentration of CO (an
industrial accident). In hospital delivered in the irresponsible state. What laboratory tests are the
early criteria of estimation of severity of the state?
A. Estimation of blood viscosity
B. Anemia
C. Leucocytosis
D. * Carboxihemoglobinemia
E. Methemoglobinemia
145. Patient complaints on attacks of dyspnea, which arise 1-2 times a week, night symptoms - 2 times a
month and even more frequently. For a patient night sleep is violated as a result of attacks of
dyspnea. FEV1 > 80 % from normal. What diagnosis would you suspect?
A. * Mild persistent BA
B. Severe persistent BA
C. Intermittent BA
D. Moderate persistent BA
E. Status asthmaticus
146. Patient D., 56 yr-old, complains on muscle weakness, bradycardia and hypotension. ECG shows
tall peaked T waves. Choose the most likely diagnosis.
A. Hypokalaemia
B. Hyponatraemia
C. Hypervitaminosis A
D. * Hyperkalaemia
E. Hypoglycaemia
147. Patient F., 46 years old, was hospitalized urgently with acute attack of dyspnea. Last 5 years he has
been working on poultry farm. During examination bronchial asthma was diagnosed. What
additional diagnostic methods are necessary to confirm the professional genesis of asthma?
A. sanitary-hygienic characteristics of the work conditions
B. echocardioscopy
C. * allergic and immunological tests
D. investigation of the function of external breath
E. roentgenography of pulmonary system
148. Patient G., 36 years old, works on a poultry factory. She was emergently hospitalized with acute
attack of dyspnea. During observation bronchial asthma was diagnosed. What additional methods
of research must be conducted above all things to confirm the professional genesis of bronchial
A. asthma?
roentgenologic research of breathing organs
B. professional route of patient
C. sanitary-hygienic characteristic of work conditions
D. research of external breathing function
E. * allergic and immunological tests
149. Patient G., 38 yr-old, complains on muscle weakness and ectopic beats. ECG shows flattened or
inverted T waves. Choose the most likely diagnosis.
A. * Hypokalaemia
B. Hyponatraemia
C. Hypervitaminosis A
D. Hyperkalaemia

E. Hypoglycaemia
150. Patient G., 47 y.o., with long history of bronchial asthma, has developed more frequent attacks of
dyspnea. Inhalations of astmopent and berotec are not effective. From prescription of what
medicine is it better to begin the intensive treatment?
A. * Glucocorticoids
B. Oxygen therapy
C. Bronchodylators
D. Infusion therapy
E. Heart glycosides
151. Patient has severe attack of bronchial asthma which lasts more than 1 hour. Usage of beta-agonists
in inhalation, euphylline intravenously and cholynolytics was not effective. What medicines are
necessary for emergency therapy?
A. * Glucocorticosteroids intravenously
B. Beta-agonists intravenously
C. Inhaled glucocorticosteroids
D. Antihystaminic
E. Nonsteroid anti-inflammatory medicines
152. Patient J., 36 y.o., complains for fever (39 C), pain in the left part of the chest. Pleuropneumonia
was diagnosed in the patient. What onset is typical for pleuropneumonia?
A. * Acute
B. Latent
C. Fulminant
D. Gradual
E. Non of the above
153. Patient J., 38 yr-old, complains on sweating, palpitations, tremors, drowsiness and fatigue. Choose
the most likely diagnosis.
A. Hypokalaemia
B. Hyponatraemia
C. Hypervitaminosis A
D. Hyperkalaemia
E. * Hypoglycaemia
154. Patient J., 45 y.o., complaints on dyspnea during insignificant physical exertion, cough with
minimal amount of “glass-like” sputum, attacks of dyspnea up to 3 times a day, more often at night,
sweating. She is ill for more than 5 years. Has an allergy on dust, cockroaches. For treatment uses
bekotid for near the year. Diagnosis?
A. * Bronchial asthma
B. Eosinophylic pulmonary infiltrate
C. COPD
D. Bronchiectasis with bronchial spasm
E. Pulmonary vasculitis (syndrome of Charg - Stross)
155. Patient L, 41 years is suffering from diabetes type 1 for 16 years, receiving insulin therapy. He
complaints of swelling and shortness of breath. On examination: glycemia - 7.1 mmol / l,
cholesterol - 6.2 mmol / l, Creatinine - 0.21 mmol / l. What will you recommend the patient first of
A. all?
Increase the dose of insulin
B. ACE inhibitors
C. Statins
D. Lipoic acid
E. * Enterosorbents

156. Patient M., 30 years old, during last 3 years works as a nurse in manipulations cabinet. Last year
during the contact with penicilline she started to complain on discomfort in throat, sneezing, attack
of cough and dyspnea which disappear after inhalation of salbutamol. During last months attacks of
dyspnea became more severe and occurred only at contact with penicilline. During the life she had
not any diseases including allergic. She hadn’t received antibiotics. Can we consider the bronchial
asthma is professional in this patient?
A. no, we can’t
B. yes, we can if we have conclusion about attacks of bronchial asthma
C. yes, we can if we have conclusion about appearance of bronchial asthma attacks after contact with
penicilline
D. yes, we can
E. * yes, we can, if allergic and immunological tests are positive
157. Patient S., 43 yr-old, has perioral paraesthesia, carpopedal spasm and generalised seizures. Choose
the most likely diagnosis.
A. Hypokalaemia
B. Hyponatraemia
C. * Hypocalcaemia
D. Hyperkalaemia
E. Hypoglycaemia
158. Patient U., 44 yr-old, complains on severe abdominal pain, nausea, vomiting, constipation, polyuria
and polydipsia. Choose the most likely diagnosis.
A. * Hypercalcaemia
B. Hyponatraemia
C. Hypervitaminosis A
D. Hyperkalaemia
E. Hypoglycaemia
159. Patient Y.,49 y.o., complaints on dyspnea, cough. Sputum is absent. Used many puffs of
salbutamol, intal, but without any efficacy. Objectively: sits, leaning on a table. Total cyanosis of
the body. Peripheral edema is absent. Breathing is superficial, dyspnea, during auscultation
breathing cannot be heard in some areas of lungs; wheezes are diffuse, expiration is considerably
prolonged. Tones of heart are weak, tachycardia. Pulse - 112/min, BP - 110/70. Liver is near the
edge of costal arch. What is the previous diagnosis?
A. * Status asthmaticus
B. Bronchial asthma of moderate severity
C. COPD
D. Aspiration of foreign body
E. Heart asthma
160. Patient, 28 y.o., has running nose, attacks of dyspnea at night once a week. Felt ill after viral
respiratory infection which was treated with acetilsalicylic acid. Eosynophylia was found in blood
and sputum. What disease may be suspected?
A. * Aspirin bronchial asthma
B. Eosinophylic infiltrate of lungs
C. Bronchial asthma of physical exertion
D. Allergic rhinitis
E. Bronchial asthma, exogenous form
161. Patient, 30 y.o., after a viral infection has daily symptoms of dyspnea, which causes lowering of
activity and bad sleep; night symptoms are more frequent than once a week. PEV and FEV1 -
60-80 %, daily variability > 30 %. There is a necessity of daily usage of ?2-agonists of short action.
What is the diagnosis?

A. * Persistent bronchial asthma of moderate severity


B. Mild persistent bronchial asthma
C. Intermittent bronchial asthma
D. Severe persistent bronchial asthma
E. Status asthmaticus
162. Patient, 42 years old, complaints on attacks of dyspnea, every time uses 1-2 doses of salbutamol.
These attacks are accompanied with cough and minimal amount of viscid glassy sputum. He is ill
for 8 years. Objectively: temperature - 36,7 C; breathing rate – 21/min.; pulse-90/min.; BP -
130/80; FEV1 - 77 %. In lungs – solitary dry wheezes. Blood test: eosinophyles - 6 %. What
medicines are “basic” in the treatment of this patient?
A. * Antiinflammatory
B. Cholynolytics
C. Mucolytics
D. Antihystaminic
E. ?2-agonists
163. Patient, 44 y.o., complaints on attack of dyspnea, which arises suddenly at night. Connects this
attack with overcooling. He is ill for more than 10 years. Thorax of emphysematous form. During
percussion in lungs – “bang-box” sound. During auscultation there is plenty of dry wheezes. In
blood: moderate leucocytosis, eosinophylia - 10 %. On the chest X-ray film – increased
pneumatization of pulmonary tissue. What diagnosis is the most possible?
A. * Bronchial asthma, exacerbation phase
B. Bronchiectasis, exacerbation phase
C. COPD, exacerbation phase
D. Chronic bronchitis
E. Eosinophylic pulmonary vasculitis
164. PatientI., a 50 years old man, is evaluated in the emergency department because of fever,
nonproductive cough and 2-day history of myalgia and headache. He has also had nausea and
diarrhea. He is a heavy smoker. On physical examination, he is slightly disoriented. Temperature is
38.9 C, pulse rate is 110/min, respiration rate is 32/min. Chest radiograph shows fluffy infiltrates to
the right upper and lower lobes. Results of laboratory testing show serum sodium of 128 meq/L,
blood urea nitrogen of 42 mg/dL, serum creatinine of 2.2 mg/dL, and serum creatine kinase of 250
U/L. Which one of the following is best next step in the management of this patient’s pneumonia?
A. * Initiate empiric antibiotic therapy for Legionella
B. Order direct fluorescent antibody testing of the sputum for Legionella
C. Order serologic testing for Legionella
D. Send a urine specimen for measurement of Legionella antigen
E. All of the above
165. Physical examination of patient revealed lymphadenopathy, splenomegaly. CBC: RBCs - 3,6 x
1012/l, Hb- 87 g/l, Pl – 45 x 109/l, WBCs – 13 x 109/l, blasts - 87%, band neutrophils - 1%,
segmented neutrophils - 7%, lymphocytes - 5%, ESR - 55 mm/h. What is the most likely diagnosis?
A. * Acute leukemia
B. Erythremia
C. Chronic lymphocytic leukemia
D. Chronic myeloid leukemia
E. Multiple myeloma
166. The 30-year-old woman whose father has type 1 diabetes mellitus asks the physician what her
chances are of developing diabetes because of her father's disease. The risk for becoming diabetic
A. * is:20-50%
B. 0%

C. 50%
D. 50-80%
E. None of above
167. The 45-year-old diabetic patient has proliferative retinopathy, nephropathy, and peripheral
neuropathy. What should the physician teach this patient about exercise?
A. “The type of exercise that would most efficiently help you to lose weight, decrease insulin
requirements, and maintain cardiovascular health would be jogging for 20 minutes 4 to 7 days each
week.”
B. “Considering the complications you already have, vigorous exercise for an hour each day is
needed to prevent progression of disease.”
C. “Considering the complications you already have, you should avoid engaging in any form of
exercise.”
D. * “Swimming or water aerobics 30 minutes each day would be the safest exercise routine for you.”
E. None of above
168. The patient diabetic patient asks the physician why it is necessary to maintain blood glucose levels
no lower than about 74 mg/dL. What is the physician’s best response?
A. “Glucose is the only fuel form used by body cells to produce energy needed for physiologic
activity.”
B. * “The central nervous system, which cannot store glucose, requires a continuous supply of glucose
for fuel.”
C. “Without a minimum level of glucose circulating in the blood, erythrocytes cannot produce ATP.”
D. “The presence of glucose in the blood counteracts the formation of lactic acid and prevents
acidosis.”
E. None of above
169. The patient getting ready to engage in a 30-minute, moderate-intensity exercise program performs a
self-assessment. Which data indicate that exercise should be avoided at this time?
A. * Ketone bodies in the urine
B. Weight 1 pound higher than the week before
C. Blood sugar level of 155 mg/dL
D. Pulse rate of 66 beats/min
E. None of above
170. The patient has clinical signs which may suggest acromegaly. Which of the following is the most
specific test to diagnose acromegaly?
A. Magnetic resonance imaging of the pituitary gland
B. Measurement of growth hormone (GH) levels during an oral glucose tolerance test
C. * Measurement of insulin-like growth factor I serum levels
D. Measurement of random GH blood levels
E. An octreotide scan
171. The patient newly diagnosed with type 2 diabetes tells the physician that since he has increased his
intake of fiber, he is having loose stools, flatulence, and abdominal cramping. What is the
physician’s best response?
A. “Decrease your intake of water and other fluids.”
B. * “Decrease your intake of fiber now and gradually add high-fiber foods back into your diet.”
C. “You must have allergies to high-fiber foods and will need to avoid them in the future.”
D. “Taking an antacid 1 hour before meals or 2 hours after meals should reduce the intensity of your
bowel problems.”
E. None of above

172. The patient on an intensified insulin regimen consistently has a fasting blood glucose between 270
and 280 mg/dL, a postprandial blood glucose level below 200 mg/dL, ketones in urine ++and a
hemoglobin A1c level of 8.5%. What is the physician’s interpretation of these findings?
A. * The patient has developing night hypoglycemia.
B. The patient is at increased risk for developing hyperglycemia.
C. The patient is demonstrating signs of insulin resistance.
D. The patient is demonstrating good control of blood glucose.
E. None of above
173. The patient tells the physician that he enjoys having a glass of wine on Saturdays when dining out
with friends. He asks if having type 1 diabetes will prohibit him from this activity. What is the
physician’s best response?
A. “Insulin activity is dramatically reduced under the influence of alcohol and drinking even one
glass of wine will increase your insulin requirements.”
B. “Diabetics have decreased kidney function and should avoid ingesting alcohol in all forms at all
times.”
C. “You shouldn't drink any alcohol because it is likely to increase your sense of hunger and make
you overeat.”
D. * “One glass of wine can be ingested with a meal and is counted as two fat exchanges.”
E. None of above
174. The patient with diabetes is visually impaired and wants to know if syringes can be prefilled and
stored for use later. What is the physician’s best response?
A. * “Yes, prefilled syringes can be stored for up to 3 weeks in the refrigerator in a vertical position
with the needle pointing up.”
B. “Yes, prefilled syringes can be stored for up to 3 weeks in the refrigerator, placed in a horizontal
position.”
C. “Insulin reacts with plastic, so prefilled syringes must be made of glass.”
D. “No, insulin cannot be stored for any length of time outside of the container.”
E. None of above
175. The patient with hyperthyroid symptoms is having hormone studies done to confirm the diagnosis.
Which set of values indicates Graves’ disease hyperthyroidism?
A. Elevated T3, elevated T4, high TSH levels
B. * Elevated T3, normal T4, low TSH levels
C. Elevated T3, low T4, high TSH levels
D. Low T3, normal T4, high TSH levels
E. None of above
176. The patient with hypothyroidism as a result of Hashimoto’s thyroiditis asks the physician how long
she will have to take thyroid medication. What is the physician’s best response?
A. “You will need to take the thyroid medication until the goiter is completely gone.”
B. None of above
C. “The thyroiditis will be cured with antibiotics, and then you will no longer need the thyroid
medication.”
D. * “You will need thyroid replacement hormone therapy for the rest of your life because the thyroid
gland function will not return.”
E. “When your thyroid function studies indicate a normal blood level of thyroid hormones, you will
be able to discontinue the medication.”
177. The patient, 51 years old, suffers from diabetes 7 years, gets glibenclamide 15 mg a day. Complains
of shortness of breath and pitting edema. On examination of blood pressure - 180/110 mm Hg.
Laboratory: glycemia - 6.2 mmol / L, creatinine - 0.15 mmol / L, glomerular filtration - 62 ml /
min. What medication will you recommend to the patient?

A. Gliclazide
B. * Gliquidone
C. Glimepiride
D. Pioglitazone
E. Repaglinide
178. When taking the blood pressure of a patient after a parathyroidectomy, the physician notes that the
patient's hand has gone into flexion contractions. What is the physician’s interpretation of this
observation?
A. Hypokalemia
B. Hyperkalemia
C. Hyponatremia
D. * Hypocalcemia
E. None of above
179. Which the following type of anemia can be normocytic normochromic anemia with reticulocytosis?
A. * Haemolytic anaemia
B. Vitamin B12 deficiency anaemia
C. Acute posthemorrhagic anaemia
D. Chronic iron deficiency anaemia
E. Aplastic anaemia
180. Which the following type of anemia can be with Cabot rings?
A. Vitamin B12 deficiency anaemia
B. Acute posthemorrhagic anaemiaChronic iron deficiency anaemia
C. Aplastic anaemia
D. * Haemolytic anaemia
181. A 27-year-old male presents to the health center for right upper quadrant abdominal pain,
generalized pruritus, and jaundice for 3 days. He states that the pain came on gradually and awoke
him early on the morning of presentation. His past medical history is pertinent for ulcerative colitis,
although he has not taken any medication in 4 years. His temperature is (39.2°C) and physical
exam shows pain in the right subcostal region with deep inspiration, and generalized jaundice.
What is the most serious complication of the most likely diagnosis?
A. Perforation of rectum.
B. Perforation of stomach wall.
C. Infarction of small bowel.
D. * Cholangiocarcinoma.
E. Pancreatic pseudocyst formation.
182. A 27-year-old man complains to a physician of chronic gastric pain. The pain is sometimes relieved
by food. EGDS shows antral erosive gastritis, biopsy of antral mucous presents Hеlicobacter
Pylori. What can be diagnosed in this case?
A. * Gastritis of type B
B. Menetrier’s disease
C. Rigid antral gastritis
D. Gastritis of A type
E. Reflux – gastritis
183. A 27-year-old man is evaluated because of two episodes of painless melena, the last of which
occurred 6 hours ago. He has been taking naproxen for the past 6 weeks for a sports-related
shoulder injury. Medical history is otherwise noncontributory. On physical examination,
temperature is 36.0 C (96.8 F), pulse rate is 98/min and regular, respiration rate is 18/min, and
blood pressure is 104/89 mm Hg without orthostatic changes. Digital rectal examination discloses
very dark stool that is positive for occult blood. Hemoglobin is 9.3 g/dL, leukocyte count is
10,800/?L, platelet count is 250,000/?L, and other routine laboratory studies are normal.
Nasogastric aspirate is negative. Which of the following should be done next?
A. Order red blood cell transfusion to achieve a hemoglobin value above 10 g/dL
B. Establish access with two intravenous lines and infuse isotonic saline, 500 mL/h for 6 to 8 hours;
then reassess the hemoglobin value
C. Transfuse 6 units of platelets to correct the platelet dysfunction caused by naproxen
D. * Perform upper endoscopy
E. Order an upper gastrointestinal series
184. A 27-yr-old pregnant woman is found to have thyrotoxicosis due to Grave's disease during second
trimester of her pregnancy. Choose the most likely management.
A. Radioactive iodine
B. Subtotal thyroidectomy
C. Propanolol
D. * Carbimazole
E. Thyroxine
185. A 27-yr-old woman presents with fever, sore throat and dysphagia. On examination she has a fine
tremor and a diffusely tender thyroid. Radioisotope scan shows no uptake. Choose the most likely
diagnosis
A. Thyroglossal cyst
B. * De Quervain's cyst
C. Hypothyroidism
D. Lymphoma
E. Simple goiter
186. A 27-yr-old woman was found to have glycosuria at a routine antenatal clinic visit. A glucose
tolerance test confirmed diagnosis of gestational diabetes. Choose the most likely management.
A. Metformin
B. No change in treatment required
C. IV insulin sliding scale
D. Gliclazide
E. * Dietary adjustment
187. A 28 years old patient, complaints on cough with small amount of colourless sputum, pain in the
right half of thorax during breathing, shortness of breath, increase of temperature to 39 °С. Felt ill
rapidly. Used aspirin. Objectively: herpes on lips. In lower lobe of right lung there is dull
percussion sound, bronchial breathing. X-ray: there is homogeneous infiltration of right lower lobe.
What is the most possible etiology of pneumonia?
A. * Pneumococcus
B. Staphylococcus
C. Mycoplasma
D. Legionella
E. Klebsiella
188. A 29 year-old man has Crohns ileocolitis complicated by perianal fistulae. Ileocecal resection was
performed 5 years ago. Approximately 4 months ago, he developed abdominal pain, cramping, and
a new enterocutaneous fistula. An upper gastrointestinal series with small bowel follow-through
showed 4 cm of inflammatory changes at the neoterminal ileum plus an enterocutaneous fistula
originating from this area. After treatment with metronidazole and mesalamine, his abdominal pain
improved and the fistula appeared to resolve. Which of the following is most appropriate for
maintaining remission in this patient?
A. * Start 6-mercaptopurine
B. Continue metronidazole
C. Start budesonide
D. Start prednisone
E. tart oral cyclosporine
189. A 29 year-old woman has hemoglobin level 7.8 g/dl, with a reticulocyte count of 1 %. The
peripheral blood smear showed microcytic hypochromic anemia. The serum iron was low and the
total iron binding capacity was high. The most likely cause of anemia is
A. * Iron deficiency anemia.
B. Beta-thalassemia minor.
C. Sideroblastic anemia.
D. Anemia due to chronic infection.
E. Porphyria
190. A 29-year-old patient complains of weight loss, night sweating, general weakness, fatigue, fever.
CBC reveals: RBC 1.1 х 1012/L, Hb 48 g/L, Pl 40 х 109/L, WBC 138,6 х 109/L. In bone marrow -
Blasts with Auer rods 70 %. What is the most likely diagnosis?
A. * Acute nonlymphoblastic (myeloid) leukemia
B. Chronic lymphocytic leukemia III st
C. Chronic myeloid leukemia, accelerated phase
D. Chronic myeloid leukemia, chronic phase
E. Chronic lymphocytic leukemia IV st
191. A 30-yr-old woman presents with a 2 cm thyroid nodule. FNAC suggests a colloid nodule. Choose
the most likely management.
A. Total thyroid lobectomy
B. Ablative dose of radioactive iodine
C. External beam radiation
D. Chemotherapy
E. * Reassure and repeat FNAC in 1 yr
192. A 30-yr-old woman presents with weight gain, constipation, lethargy and flaky rash. Choose the
most likely diagnosis
A. Thyroglossal cyst
B. De Quervain's cyst
C. * Hypothyroidism
D. Lymphoma
E. Simple goiter
193. A 31-year-old man has a 2-month history of abdominal pain and bloating. An upper gastrointestinal
series with small bowel follow-through shows ulcerations and inflammatory changes in the distal
12 cm of the terminal ileum. Colonoscopy is normal except for erythema and small linear
ulcerations seen on cannulation of the terminal ileum. Which of the following is the most
appropriate therapy for this patient at this time?
A. * pH-release mesalamine
B. Mesalamine enemas
C. Balsalazide
D. Olsalazine
E. Sulfasalazine
194. A 31-year-old woman with ulcerative colitis has been taking prednisone for the past year. Each time
the prednisone is tapered below 20 mg/d, her symptoms return. She is subsequently started on
6-mercaptopurine, 50 mg/d. Three days after beginning the new drug, she develops worsening
abdominal pain with radiation to her back. She does not have a rash. Her leukocyte count is
3200/?L. Which of the following is the most likely cause of this patients new symptoms?
A. * Pancreatitis due to initiation of 6-mercaptopurine
B. Pancreatitis due to continuation of prednisone
C. Flare of ulcerative colitis
D. An abdominal and psoas abscess secondary to 6-mercaptopurine-induced neutropenia
E. None of above
195. A 32 -year-old woman complained on epigastric pain just after meal, heartburn, and nausea.
Stomach endoscopy revealed a large amount of mucus, hyperemia, erosions of mucous membrane
of stomach. Helicobacter infection test is positive.
A. Chronic type A gastritis
B. Peptic ulcer of the stomach
C. Chronic type C gastritis
D. Menetrier’s disease
E. * Chronic type B gastritis
196. A 32-year-old father of two is involved in a head-on motor vehicle accident and is admitted to your
hospital. He has sustained a blunt force trauma to the upper thorax but injuries appear only to be
several cracked ribs and a minor concussion. On hospital day 3, he reports the uncontrollable urge
to drink copious amounts of water, states that he has been urinating much more frequently and that
his urine is very thin in appearance. The blood and a urin analysis are normal except for high
plasma osmolarity and low urine osmolarity. You decide to do a water deprivation test. Thus for 24
hours, the patient is restricted to a sip of water every hour, and on day 2 of the test is clinically
judged to be dehydrated. Vasopressin (ADH) is then given by IV and his urine osmolarity is seen to
rise above a baseline dehydrated state reading. The most likely diagnosis is central diabetes
insipidus. What is the next step in the management of this patient?
A. Hydrochlorothiazide (HCTZ).
B. Furosemide (Lasix).
C. Hydrocortisone orally.
D. Aggressive fluid replacement.
E. * Desmopressin (DDAVP).
197. A 32-year-old woman has a 4-month history of hoarseness and throat clearing. Evaluation by an
otolaryngologist disclosed laryngeal inflammation suggestive of gastroesophageal reflux disease,
and the patient is referred to you. She is otherwise asymptomatic and denies heartburn,
regurgitation, dysphagia, and weight loss. The patient maintains an active lifestyle and currently
takes no medications. Physical examination and routine laboratory studies are normal. Which of the
following should be done next?
A. Upper endoscopy
B. Esophageal manometry
C. Ambulatory 24-hour esophageal pH monitoring
D. Barium swallow
E. * Trial of acid suppressive therapy
198. A 32-yr-old male presents with nausea and acute abdominal pain boring through to the back. The
epigastrium is very tender. He has had three similar bouts in the past 18 months. A barium meal is
normal. Choose the single most likely diagnosis

A. Gastric atrophy
B. Gastric ulcer
C. Basal pneumonia
D. Chronic cholecystitis
E. * Non-ulcer dyspepsia
199. A 32-yr-old man presents with spontaneous bruising and recurrent infections with marked lethargy.
There is no recent treatment history. His Hb is 56 g/l, RBC 2,8 x 1012/L, WBC 2.8 platelets 48,5.
What is the diagnosis?
A. * Aplastic anaemia
B. Autoimmune hemolytic anaemia
C. Anaemia of chronic disease
D. Iron deficiency
E. Pernicious anaemia
200. A 32-yr-old woman has had type 1 diabetes for 15 yr. She injects Isophane insulin twice a day and
rarely tests her blood glucose at home. She attends the diabetic clinic for the first time in over a
year and informs you that she is 12 weeks pregnant. Choose the most likely management.
A. Metformin
B. No change in treatment required
C. IV insulin sliding scale
D. * One long-acting and 3 short- acting insulin
E. Dietary adjustment
201. A 33-year-old woman presents to the clinic with a positive home pregnancy test. The patient has a
history of Hashimoto’s thyroiditis, which has been successfully managed with levothyroxine 125
µg daily for the past 4 years; serum thyroid-stimulating hormone (TSH) levels have been between
0.5 and 1.5 µIU/mL (normal, 0.3-5.5 µIU/mL). She has a family history of thyroid disease, and her
mother also takes levothyroxine. In the office, the pregnancy test is confirmed; this is her first
pregnancy. To remain biochemically and clinically euthyroid, how should this patient be managed?
A. * Levothyroxine should be increased
B. Levothyroxine should be decreased
C. Triiodothyronine (T3) should be taken in addition to levothyroxine
D. The pregnancy should be terminated
E. None of above
202. A 34-year-old male presents with substernal discomfort. The symptoms are worse after meals,
particularly a heavy evening meal, and are sometimes associated with hot/sour fluid in the back of
the throat and nocturnal awakening. The patient denies difficulty swallowing, pain on swallowing,
or weight loss. The symptoms have been present for 6 weeks; the patient has gained 20 lb in the
past 2 years. Your initial approach is
A. * therapeutic trial of ranitidine
B. Exercise test with thallium imaging
C. Esophagogastroduodenoscopy
D. CT scan of the chest
E. Barium contrast study is indicated
203. A 34-year-old woman is evaluated because of a 1-year history of increased dyspnea on exertion.
She has no symptoms at rest but has to stop about 15 minutes into her aerobics class because of
dyspnea and occasional cough. She usually recovers fully in about an hour. One year ago she was
able to do aerobics for 45 minutes without difficulty. Her vital signs are normal, and her physical
examination is normal, including clear breath sounds. Baseline spirometry is also normal. Which of
the following would be best next step in the management of this patient?
A. * Inhaled albuterol prior to exercise

B. Oral leukotriene inhibitors


C. Long-acting theophylline
D. Inhaled ipratropium bromide prior to exercise
E. Inhaled corticosteroids
204. A 34-year-old woman is hospitalized after a minor episode of hematemesis. She denies current use
of nonsteroidal anti-inflammatory drugs. Two years ago, a duodenal ulcer and Helicobacter pylori
infection were diagnosed by upper endoscopy, and bismuth subsalicylate, metronidazole,
tetracycline, and ranitidine were prescribed for 14 days. However, the patient took the medications
for only 8 days because of nausea. Upper endoscopy performed during the current admission shows
a clean-based duodenal ulcer, and rapid urease testing of an endoscopic mucosal biopsy specimen is
positive for H. pylori. Which of the following is the most appropriate management at this time?
A. Bismuth subsalicylate, metronidazole, tetracycline, and ranitidine for 4 days
B. Bismuth subsalicylate, metronidazole, amoxicillin, and ranitidine for 14 days
C. A proton pump inhibitor for 14 days
D. * A proton pump inhibitor, clarithromycin, and amoxicillin for 14 days
E. Ranitidini (H2-histaminoblokers)
205. A 34-year-old woman is scheduled to undergo flexible sigmoidoscopy because of bloody bowel
movements of 4 days duration. She has had diarrhea for about 10 days, but 4 days ago she
developed frequent, low-volume bloody stools that are associated with urgency. The patient has had
episodic diarrhea for about 3 months, during which time she noted and ignored bloody bowel
movements on four occasions. Stool specimens obtained 3 days ago are negative for pathogens.
Sigmoidoscopic findings include inflammation from the rectum to 20 cm with no areas of normal
intervening mucosa. Which of the following is the most likely diagnosis?
A. * Ulcerative proctocolitis
B. Crohn’s disease
C. Irritable bowel syndrome
D. lschemic colitis
E. None of above
206. A 34-yr-old man presents with insidious onset weakness and weight loss. On examination, he has
hyperpigmentation of the palmar creases and postural hypotension. Choose the most likely
investigation
A. Dexamethasone suppression test
B. * ACTH stimulation test
C. Fasting blood glucose
D. T3, T4 and TSH levels
E. Water deprivation test
207. A 34-yr-old woman points to an area of acute epigastric pain with his right index finger. The pain is
worse at night and taking food relieves it. Taking antacids also relieves it. Choose the single most
likely diagnosis
A. Gastric atrophy
B. Acute pancreatitis
C. Basal pneumonia
D. Chronic cholecystitis
E. * Esophagitis
208. A 35 -year-old man complained of squeezed epigastric pain 1 hour after meal and heartburn. She
has been ill for 2 years. On palpation, there was moderate tenderness in pyloroduodenal area. Antral
gastritis was revealed on gastroscopy. Which study can establish genesis of the disease?
A. USD of abdomen
B. Gastrin level in blood

C. Examination of stomach secretion


D. Examination of stomach motor function
E. * Revealing of Helicobacter infection in gastric mucosa
209. A 35 -year-old patient complains on severe weakness and fever. Disease started with pain in the
throat and fever. Physical examination: skin and mucous are pale, ecchymosis, purpura of skin and
subcutaneous tissue. Peripheral blood smear: RBC - 1,5 х 1012 /L, Нв - 50 g/L. Blast – 90 %,.
What is the most likely diagnosis?
A. * Acute leukemia
B. Thrombocytopenic Purpura
C. Hypoplastic Anemia
D. AIDS
E. Leukemoid reaction
210. A 35-year-old man complains to a physician of chronic vague gastric pain of several years'
duration. The pain is sometimes relieved by food. Which of the following is the most appropriate
test to noninvasively determine the H. pylori?
A. Culture of gastric biopsy
B. Rapid urease test
C. Repeat qualitative IgA and IgG antibodies against H. pylori
D. Repeat quantitative IgA and IgG antibodies against H. pylori
E. * Urea breath test
211. A 35-year-old man consults a gastroenterologist because of chronic heartburn for several years. The
heartburn tends to be worse at night, and he frequently tastes refluxed gastric contents when he
goes to bed. He found that his symptoms were a little better when he avoided his customary late
evening alcoholic drink; however, this modest improvement has subsequently deteriorated. The
gastroenterologist performs esophageal manometry with pH monitoring, which demonstrates
decreased pressure of the lower esophageal sphincter and the presence of acid in the esophagus.
Biopsy of the proximal end of one of these fingers shows surface epithelium with regular columnar
cells with small, ovoid nuclei admixed with goblet cells. Which of the following is the most likely
diagnosis?
A. Achalasia
B. * Barrett esophagus
C. Corrosive esophagitis
D. Esophageal adenocarcinoma
E. Zenker diverticulum
212. A 35-year-old patient with alcohol abuse and abnormal diet complains of pain in epigastrium that
occurs in 1-1,5 hours after having meal. Esophagogastroduodeno-scopy: marked hyperemia, small
defects, easy appearance of sores on the mucous membrane in the antral section of the stomach.
What is the most probable reason for this pathology?
A. * Helicobacter pуlori infection
B. Presence of antibodies to parietal cells
C. Alimentary factor
D. Nervous overstrain
E. Toxic action of alcohol
213. A 35-year-old woman who underwent successful therapy for acute leukemia 10 years ago is
seropositive for hepatitis C. Polymerase chain reaction testing confirms the presence of virus in a
blood sample. She has mildly elevated serum transaminases. A liver biopsy discloses moderately
severe chronic hepatitis without fibrosis. The most appropriate therapy is
A. * interferon plus ribavirin
B. acyclovir

C. ribavirin
D. propranolol
E. no therapy is indicated
214. A 35-year-old woman with amenorrhea is found to have an enlarged pituitary glan. Her prolactin
level is 80 ng/L (normal less than 20 ng/L), and her thyrotropin level is 100 mlU/ml (normal, 0.5 to
4.5 mlU/ml). Which of the following is the best treatment option for this patient?
A. Administration of bromocriptine
B. * Administration of L-thyroxine
C. Irradiation of the pituitary gland
D. Resection of the pituitary gland
E. Use of oral contraceptives
215. A 35-yr-old woman with bone pain, drowsiness and thirst: Calcium—3.3, phosphate—0.75,
ALP—190, PTH—low-normal, PTH—activity high, glucose—6 mmol/l. Choose the most likely
A. diagnosis
Primary hyperparathyroidism
B. Tertiary hyperparathyroidism
C. Hypoparathyroidism
D. Hyperthyroidism
E. * Hyperparathyroidism with ectopic PTH
216. A 36-yr-old woman presented with generalised pruritus for 5 months. On examination she was
tanned and there were spider naevi on her chest. The liver was palpable one finger breadth below
the costal margin as well as the tip of the spleen. Choose the single most likely diagnosis from the
list of options above.
A. Wilson's disease
B. * Primary biliary cirrhosis
C. Cholecystitis
D. Haemochromatosis
E. Alcoholic liver cirrhosis
217. A 37-year-old man complains of pains in epigastrium which are relieved by food intake. EGDS
shows antral erosive gastritis, biopsy of antral mucous presents Hеlicobacter Pylori. Diagnosis is:
A. Gastritis of type A
B. Reflux - gastritis
C. Menetrier's gastritis
D. Rigid antral gastritis
E. * Gastritis of type B
218. A 37-year-old man with asthma is evaluated because he continues to have frequent attacks and now
feels his short-acting ?2-agonist is not providing relief. He states he is using his medications,
including a long-acting ?2-agonist inhaler, inhaled high-dose corticosteroids, and a
short-acting ?2-agonist inhaler as rescue medication. He has symptoms daily and nocturnal
symptoms about twice per week. On physical examination, he is in mild respiratory distress. He is
afebrile. Pulse rate is 90/min and regular, respiration rate is 18/min, and blood pressure is 140/85
mm Hg. He has bilateral wheezing. Spirometry shows a forced expiratory volume in 1 sec (FEV1)
65% of predicted; it improves with bronchodilators to 85% of predicted. He has no history of recent
viral upper respiratory infections or rhinitis or symptoms of gastroesophageal reflux disease. Which
of the following is the best next step in this patient’s management?
A. * Observe the patient using the metered-dose inhaler
B. Add a leukotriene inhibitor
C. Switch to an oral ?2-agonist and have the patient return for a pill count
D. Initiate oral prednisone therapy and have the patient return for a pill count
E. Have the patient return with a symptom and treatment log.

219. A 37-year-old patient complains of epigastric pain that occurs in 1-1,5 hours after having meal.
Esophagogastroduodeno-scopy: marked hyperemia, small defects, easy appearance of sores on the
mucous membrane in the antral section of the stomach. What is the most probable reason for this
pathology?
A. * Helicobacter pуlori infection
B. Presence of antibodies to parietal cells
C. Alimentary factor
D. Nervous overstrain
E. Toxic action of alcohol
220. A 37-year-old woman has a 4-month history of epigastric discomfort and heartburn. Symptoms are
usually exacerbated postprandially, especially if she eats spicy foods. The patient denies dysphagia,
weight loss, and decreased appetite. Treatment with a proton pump inhibitor, once daily for 4
weeks, resulted in only minimal improvement. Increasing the medication to twice daily for an
additional 4 weeks did not improve her symptoms, and the patient wants to know what other
management options are available. Referral for which of the following procedures is most
A. * appropriate
Ambulatoryat24-hour
this time?
esophageal pH monitoring
B. Upper endoscopy with esophageal dilation
C. Barium swallow
D. Surgical fundoplication
E. USD
221. A 37-year-old woman has a 4-month history of epigastric discomfort and heartburn. Symptoms are
usually exacerbated postprandially, especially if she eats spicy foods. The patient denies dysphagia,
weight loss, and decreased appetite. Treatment with a proton pump inhibitor, once daily for 4
weeks, resulted in only minimal improvement. Increasing the medication to twice daily for an
additional 4 weeks did not improve her symptoms, and the patient wants to know what other
management options are available. Referral for which of the following procedures is most
A. appropriate at this time?
Upper endoscopy with esophageal dilation
B. Barium swallow
C. Surgical fundoplication
D. USD
E. * Ambulatory 24-hour esophageal pH monitoring
222. A 37-year-old woman sees her physician because of gum bleeding, menorrhagia. Physical
examination reveals petechiae, bruises on her legs. CBC: RBC 2.5 х 1012/L, Hemoglobin 67 g/L,
MCV 64 fl, CI – 0,7, Pl 41 х 109/L, Giant platelets +, WBС 7.9 х 109/L, ESR 7 mm/h, Bleeding
time 17 min. What is the most probably diagnosis?
A. Henoch–Schonlein purpura
B. Thrombotic thrombocytopenic purpura
C. Chronic lymphocytic leukemia
D. Macrocytic hyperchromic anemia
E. * Idiopathic thrombocytic purpura
223. A 37-yr-old woman presents with weight loss, muscular weakness, oligomenorrhea, diarrhea and
blurring of vision. On examination, there is exophthalmos and proximal myopathy. Choose the
most likely diagnosis
A. Thyroglossal cyst
B. * Graves' disease
C. Hypothyroidism
D. Lymphoma
E. Simple goiter

224. A 38-year-old former hemodialysis nurse is seen because of a 6-month history of fatigue and
amenorrhea. On examination she has scleral icterus, a mildly tender liver, and a tibial rash
consistent with erythema nodosum. ALT and AST levels are both in the range increased, while
alkaline phosphatase and serum albumin levels are normal. Hepatitis serologic testing detects
HBsAg and IgG anti-HBcAg. Liver biopsy discloses a mononuclear cell portal infiltrate and
hepatocyte destruction at the periphery of lobules. Which of the following therapeutic strategies is
A. * best?
Administration of interferon, 10 million units three times per week for 4 months
B. Administration of prednisone, 20 to 40 mg/d for 2 months and then a taper based on the response
C. Administration of prednisone, 10 mg every other day for 3 months
D. Administration of acyclovir, 400 mg every 6 h for 2 weeks
E. Administration of low-dose cyclophosphamide, 50 mg/d for 2 months
225. A 38-year-old male insurance agent with a benign past medical history presents to his primary care
physician complaining of indigestion. He notes that “heartburn” has occurred weekly for about 1
year, especially after eating a heavy meal. He has no risk factors for coronary artery disease and
does not complain of weight loss, vomiting, dysphagia, or bleeding. Physical and routine laboratory
examinations are unrevealing. Which of the following is the most appropriate next step?
A. Upper gastrointestinal barium radiography
B. Upper gastrointestinal endoscopy
C. Ambulatory esophageal pH testing
D. * Serology for H. Pylori
E. Prescribe omeprazole
226. A 38-year-old male insurance agent with a benign past medical history presents to his primary care
physician complaining of indigestion. He notes that “heartburn” has occurred weekly for about 1
year, especially after eating a heavy meal. He has no risk factors for coronary artery disease and
does not complain of weight loss, vomiting, dysphagia, or bleeding. Physical and routine laboratory
examinations are unrevealing. Which of the following is the most appropriate next step?
A. Upper gastrointestinal endoscopy
B. Ambulatory esophageal pH testing
C. Upper gastrointestinal barium radiography
D. Prescribe omeprazole
E. * Serology for H. Pylori
227. A 38-year-old man complains of chronic heartburn for several years. The heartburn tends to be
worse at night, and he frequently tastes refluxed gastric contents when he goes to bed. He found
that his symptoms were a little better when he avoided his customary late evening alcoholic drink;
however, this modest improvement has subsequently deteriorated. The esophageal manometry with
pH monitoring demonstrates decreased pressure of the lower esophageal sphincter and the presence
of acid in the esophagus. Biopsy of the proximal end of esophagus shows surface epithelium with
regular columnar cells with small, ovoid nuclei admixed with goblet cells. Which of the following
is the most likely diagnosis?
A. Achalasia
B. * Barrett esophagus
C. Corrosive esophagitis
D. Esophageal adenocarcinoma
E. Zenker diverticulum

228. A 38-year-old man who works as a reporter for a travel magazine comes to his physician because of
the acute onset of jaundice, malaise, and temperatures to 38.5 С .He had returned from Burma 2
weeks ago, where he spent 4 weeks. He says that he abstains from alcohol beverages and does not
take any medications. Laboratory studies show elevated serum aminotransferases, high bilirubin
(both total and direct), and negative serology for hepatitis A virus (HAV) and С virus (HCV)
infection. He was vaccinated for hepatitis В virus (HBV) 3 years ago and is now positive for
anti-HBsAg antibodies. Which of the following serologic markers should be tested as the most
appropriate next step in diagnosis?
A. Anti-HCV IgG antibodies by RIBA
B. Anti-HDV IgG antibodies
C. * Anti-HEV IgM antibodies
D. Anti-HGV IgG antibodies
E. HBsAg
229. A 39-year-old patient complains of weight loss, night sweating, general weakness, fatigue, fever,
gum bleeding. Bone marrow examination reveals blasts with Auer rods 87 %. What is the most
likely diagnosis?
A. * Acute nonlymphoblastic (myeloid) leukemia
B. Erythremia
C. Chronic lymphocytic leukemia
D. Chronic myeloid leukemia
E. Multiple myeloma
230. A 39-yr-old woman presented with jaundice and painless depigmented patches on her hands, neck
and face. On exami_nation multiple spider naevi were found. Choose the single most likely
diagnosis from the list of options above.
A. Haemochromatosis
B. Primary biliary cirrhosis
C. Wilson's disease
D. * Chronic active hepatitis
E. Pancreatic carcinoma
231. A 40-year-old male with long-standing alcohol abuse complains of abdominal swelling, which has
been progressive over several months. He has a history of gastrointestinal bleeding. On physical
exam, there are spider angiomas and palmar erythema. Abdominal collateral vessels are seen
around the umbilicus. There is shifting dullness, and bulging flanks are noted. An important first
step in the patient’s evaluation is
A. * Diagnostic paracentesis
B. UGI series
C. Ethanol level
D. CT scan
E. Esophagogastroduodenoscopy
232. A 40-year-old white female complains of pruritus. She has an elevated alkaline phosphatase and
positive antimitochondrial antibody test. What is the most likely disease?
A. * Primary biliary cirrhosis
B. Sclerosing cholangitis
C. Anaerobic liver abscess
D. Hepatoma
E. Hepatitis C

233. A 40-year-old woman, a nurse, is evaluated because of worsening asthma symptoms. She has had
mild, intermittent asthma since college, for which she has been using an albuterol inhaler as
needed, usually less than once a month. During the past 3 months, she has experienced cough,
tightness of the chest, and wheezing, which improve after the use of inhaled albuterol. She uses the
inhaler twice a day on average and has awakened at least twice a week with nocturnal cough. She
works three consecutive 12-hour day shifts, and the cough is regularly worse at the end of each
shift. During her days off, she has fewer asthma symptoms and feels significantly better by the time
she returns to work. She has a history of allergic rhinitis that has also recently become more
symptomatic. Approximately 6 months ago, she acquired a kitten that sleeps in the bedroom. She
has lived in her home for 6 years, and it is carpeted and has heavy draperies. Chest examination is
notable for good air entry. There are scattered end-expiratory wheezes. In addition to treatment with
inhaled corticosteroids, which of the following interventions is most likely to benefit this patient?
A. * Avoiding exposure to latex products
B. Treatment with an oral antihistamine
C. Getting rid of the kitten
D. Removing the carpets and draperies from her home
E. Treatment with a leukotriene-modifying drug
234. A 40-yr-old diabetic actor is started on Propanolol for stage fright. He collapses after a day
shooting. He has not changed his insulin regimen. Choose the most likely management.
A. Sugary drink
B. Insulin sliding scale, Heparin and 0.45% saline
C. Insulin sliding scale, 0.9% NS and potassium replacement
D. Chest X-ray
E. * 50 ml of 50% dextrose IV
235. A 40-yr-old man presents to his GP complaining of change in appearance and headaches. His brow
is more prominent and his nose had broadened. He states that his shoes are too small and he has
tingling in certain fingers worse at night. Choose the most likely management.
A. Propanolol
B. Calciferol
C. Carbimazole
D. Thyroxine
E. * Octreotide (somatostatin)
236. A 40-yr-old woman has a long history of pruritus, arthralgia and mild jaundice. She presents with
haematemesis and is found to have splenomegaly. Endoscopy shows oesophageal varices. Choose
the single most likely diagnosis from the list of options above. Choose the single most likely
diagnosis from the list of options above.
A. * Portal hypertension
B. Myelofibrosis
C. Infective endocarditis
D. None of above
E. Gilbert's syndrome
237. A 40-yr-old woman presents with a hard, nodular midline neck mass. Blood tests reveal the
presence of antibodies to thyroglobulin. Choose the most likely diagnosis.
A. Thyroglossal cyst
B. * Hashimoto's thyroiditis
C. Toxic multinodular goiter
D. Thyroid storm
E. Graves' disease

238. A 40-yr-old woman presents with a solitary nodule in the right thyroid lob. FNAC suggests
follicular adenoma. Choose the most likely management.
A. * Total thyroid lobectomy
B. Ablative dose of radioactive iodine
C. External beam radiation
D. Chemotherapy
E. Reassure and repeat FNAC in 1 yr
239. A 40-yr-old woman presents with fatigue, dyspnoea, paraesthesiae and a sore red tongue. Her blood
film shows hypersegmented polymorphs, an MCV of > 110 fl and a low Hb. What is your
A. * diagnosis?
Pernicious anaemia
B. Iron-deficiency anemia.
C. Sickle cell anaemia
D. Hemolytic anemia.
E. Aplastic anemia.
240. A 41-year-old morbidly obese female comes to the emergency department with colicky abdominal
pain in her right upper abdomen. She complains that this is similar to, yet more severe than, the
pain that often occurs after meals for the past 4 months. Her past medical history is positive for
diabetes mellitus type 2, hypertension, hyperlipidemia, and smoking. On physical exam, her
temperature is 100.5°F (38.1°C) and her sclera appear mildly icteric. What imaging modality may
be limited in this patient?
A. MRCP.
B. CT scan.
C. * Ultrasonography.
D. Esophogastroduodenoscopy (EGD).
E. IDA scan.
241. A 42-year-old patient complains of weight loss, night sweating, general weakness, fatigue. Physical
examination reveals lymphadenopathy, splenomegaly. CBC reveals: RBC 2.34 х 1012/L,
Hemoglobin 78 g/L, Hematocrit 40 %, MCV 90 fl, Reticulocyte Count 1.0 %, Pl 245 х 109/L,
WBC 71.9 х 109/L, Segmented Neutrophils 2 %, Band Neutrophils 0 %, Monocytes -0%
Eosinophils -0 %, Basophils -0%, Lymphocytes 98%, ESR 24mm/h,Smudge cells +. What is the
A. most
Acutelikely diagnosis?
leukemia
B. * Chronic lymphocytic leukemia III st
C. Chronic lymphocytic leukemia I st
D. Chronic myeloid leukemia
E. Chronic lymphocytic leukemia IV st
242. A 42-year-old patient complains of weight loss, night sweating. CBC reveals: RBC 3.75 х 1012/L,
Hb 125 g/L, Ht 42 %, MCV 91 fl, -Reticulocyte Count 1.0 %, Pl 280 х 109/L, WBC 47.9 х 109/L,
Segmented Neutrophils 3 %, Band Neutrophils 1 %, Monocytes 2 %, Eosinophils 1 %, Basophils
0 %, Lymphocytes 93 %, ESR 7 mm/h, Smudge cells +. What is the most likely diagnosis?
A. Acute leukemia
B. Erythremia
C. * Chronic lymphocytic leukemia
D. Chronic myeloid leukemia
E. Multiple myeloma
243. A 42-year-old patient complains of weight loss, night sweating. Physical examination reveals
lymphadenopathy, splenomegaly. CBC reveals: RBC 3.75 х 1012/L, Hb 125 g/L, Ht 42 %, MCV
91 fl, -Reticulocyte Count 1.0 %, Pl 280 х 109/L, WBC 47.9 х 109/L, Segmented Neutrophils 3 %,
Band Neutrophils 1 %, Monocytes 2 %, Eosinophils 1 %, Basophils 0 %, Lymphocytes 93 %, ESR
7 mm/h, Smudge cells +. What is the most likely diagnosis?

A. Acute leukemia
B. Chronic lymphocytic leukemia III st
C. * Chronic lymphocytic leukemia II st
D. Chronic myeloid leukemia
E. Chronic lymphocytic leukemia IV st
244. A 42-year-old patient suffering from alco_holism has advanced liver disease with ascites.
Examination reveals asterixis of the hands, ankle clonus, and spider angiomas on the face and
chest. Precipitat_ing factors to look for include all of the fol_lowing EXCEPT:
A. * heart insufficiency
B. bleeding esophageal varices
C. excessive diuretic therapy
D. non-compliance with lactulose therapy
E. spontaneous bacterial peritonitis
245. A 42-yr-old man has hypertension, hyperglycaemia, myopathy, thinning of the skin, buffalo hump
and truncal obesity. Choose the most likely investigation
A. * Dexamethasone suppression test
B. ACTH stimulation test
C. Fasting blood glucose
D. T3, T4 and TSH levels
E. Water deprivation test
246. A 44-yr-old mother of five children complained of yellow skin and abdominal pain, especially after
meals. She was overweight and she said that she did not like going out to restaurants because of
embarrassing flatulence. Choose the single most likely diagnosis from the list of options above.
A. * Gallstones
B. Hepatitis
C. Carcinoma of bile duct
D. Alcoholism
E. Carcinoma pancreas
247. A 44-yr-old woman complains of intense pruritus and yellowing of her skin. On physical
examination you notice xanthomata and skin pigmentation. Choose the single most likely diagnosis
from the list of options above.
A. * Primary biliary cirrhosis
B. Hepatitis
C. Carcinoma of bile duct
D. Alcoholism
E. Haemolytic anaemia
248. A 44-yr-old woman presents with tachycardia, atrial fibrillation, double vision and swelling above
her ankles. She has lid lag on examination. Choose the most likely investigation
A. Dexamethasone suppression test
B. ACTH stimulation test
C. Fasting blood glucose
D. * T3, T4 and TSH levels
E. Water deprivation test
249. A 45 year-old man undergoes a routine physical examination with screening blood studies. Physical
examination is notable for an increased liver diameter; the liver edge is palpable and without
irregularities. Blood studies show elevated liver enzymes. The clinician suspects alcoholic hepatitis.
Which of the following findings would tend to support this diagnosis?
A. Alanine aminotransferase = 2000 U/L

B. * Aspartate aminotransferase (AST)/alanine aminotransferase (ALT) ratio = 2.5


C. Gamma-glutamyl transferase (GGT) = 20 U/L (norm
D. Mean corpuscular volume (MCV) = 65 urn3
E. Platelet count = 600,000/mm3
250. A 45-year-old female with long-standing alcohol abuse complains of abdominal swelling, which
has been progressive over several months. On physical exam, there are spider angiomas and palmar
erythema. Abdominal collateral vessels are seen around the umbilicus. A paracentesis is performed.
The serum albumin minus ascitic fluid albumin equals 1.4 g/dL. The most likely diagnosis is
A. * Portal hypertension
B. Pancreatitis
C. Tuberculous peritonitis
D. Hepatoma
E. No ascitis
251. A 45-year-old man for 1 month has complained of epigastric and right subcostal aching pain,
pruritus, indigestion, dark color of the urine and acholic stool, fever, and significant weight loss. On
exam: jaundice, presence of Curvuasier’s sign. US scan did not reveal stones in the gallbladder and
choledochus. What is the most likely diagnosis?
A. * Cancer of the pancreas head
B. Gallbladder stones
C. Chronic pancreatitis
D. Chronic cholangitis
E. Chronic hepatitis
252. A 45-year-old man with history of high blood pressure that has been difficult to control with a
variety of antihypertensive medications presents with persistent headaches, excessive sweating, and
palpitations. Routine blood tests are normal. Electrocardiogram shows supraventricular tachycardia.
What is this patient’s most likely diagnosis?
A. * Pheochromocytoma
B. Primary hyperaldosteronism
C. Cushing’s disease
D. Hyperthyroidism
E. Malignant hypertension
253. A 45-year-old man with history of high blood pressure that has been difficult to control with a
variety of antihypertensive medications presents with persistent headaches, excessive sweating, and
palpitations. Routine blood tests are normal. Electrocardiogram shows supraventricular tachycardia.
It was suspected pheochromocytoma. What is this patient’s most likely test?
A. cortisol level
B. aldosteron level
C. 17-hydroxycorticosteroids
D. * urine metanephrines
E. TSH
254. A 45-year-old man with history of high blood pressure that has been difficult to control with a
variety of antihypertensive medications presents with persistent headaches, excessive sweating, and
palpitations. Routine blood tests are normal. Electrocardiogram shows supraventricular tachycardia.
Which of the following is the best initial pharmacologic therapy choice for this patient?
A. ?-Blockers
B. Diuretics
C. Steroids
D. Methimazole
E. * ?-Blockers

255. A 45-year-old woman complains of right upper quadrant pain, which occurs after she eats a large
meal. Occasionally the episodes are accompanied by nausea and vomiting. A plain x-ray of the
abdomen discloses gallstones. Ultrasonography reveals gallstones and a normalsized common bile
duct. The patient’s blood chemistry and CBC are normal. The most therapeutic maneuver at this
time would be
A. * laparoscopic cholecystectomy
B. observation
C. ursodeoxycholic acid
D. shock wave lithotripsy
E. ursodeoxycholic acid and shock wave lithotripsy
256. A 45-yr-old patient complains of giddiness and falls. She also suffers with intermittent vomiting
and sweating and occasional faecal incontinence at night. Choose the most likely diagnosis
A. Coronary artery disease
B. Cerebrovascular disease
C. Nephropathy
D. Retinopathy
E. * Autonomic neuropathy
257. A 45-yr-old woman presents with a diffuse swelling of the thyroid gland. On examination she has a
stare, lid lag, and lid retraction. On the dorsum of her legs she has areas of raised peau d'orange-like
thickened skin. Blood tests reveal thyroid-stimulating immunoglobulins against the TSH receptor
site. Choose the most likely diagnosis.
A. Thyroglossal cyst
B. Hashimoto's thyroiditis
C. Toxic multinodular goiter
D. Thyroid storm
E. * Graves' disease
258. A 46-year-old woman who works as a nurse is evaluated because of a 2-year history of episodic
wheezing and a squeaky voice. This past spring, her symptoms worsened, requiring her to seek
medical attention; she was placed on a short-acting ?2-agonist that did not provide much relief. She
has no history of wheezing and says that these changes began after a severe influenza infection 3
years ago. Currently she feels well and has had no symptoms for several months; she is not taking
any medications. Physical examination shows no abnormalities, and baseline spirometry is normal.
What is the best test to evaluate this patient’s condition?
A. * Methacholine challenge testing
B. Bronchoscopy to evaluate her trachea
C. Exercise echocardiogram
D. CT scan of the sinuses
E. Non of above
259. A 47-year-old female presents to the health clinic for symptoms of weight she can't seem to lose
despite strict dieting and concerns of menstrual irregularity. She states she also has developed
multiple abdominal stretch marks and has noticed a deepening of her voice and appearance of facial
hair. As part of her initial work up, you obtain a 24-hour urine cortisol level, which is returned as
120 ng/ml (N 20 100 ng/ml). What is the most likely diagnosis in this patient?
A. Metabolic syndrome.
B. Diabetes mellitus type II.
C. * Cushing's syndrom
D. Addison's diseas
E. Conn's syndrom

260. A 47-year-old female presents to the health clinic for symptoms of weight she can't seem to lose
despite strict dieting and concerns of menstrual irregularity. She states she also has developed
multiple abdominal stretch marks and has noticed a deepening of her voice and appearance of facial
hair. As part of her initial work up, you obtain a 24-hour urine cortisol level, which is returned as
120 ng/ml (N 20 100 ng/ml). The most likely diagnosis in this patient is Cushing's syndrome.What
is the most appropriate next step in diagnosis of this patient?
A. Cosyntropin (ACTH) stimulation test.
B. * Dexamethasone suppression test.
C. Cosyntropin (ACTH) suppression test.
D. Dexamethasone stimulation test.
E. Urine catecholamine collection over a 24-hour perio
261. A 47-yr-old agricultural worker complains of a chronic cough, purulent sputum and abdominal
distention. He has just arrived in England from Spain where he was picking grapes. Choose the
single most likely diagnosis from the list of options above.
A. * Tuberculosis
B. Cirrhosis
C. Malabsorption
D. Pancreatitis
E. Peptic ulcer
262. A 48 years old patient, complaints on weakness, dyspnea, pain in the left half of thorax, permanent
cough with viscid sputum, in which particles of blood are sometimes determined. For the last 3
months lost 5 kg of body mass. On the X-ray of lungs there is total homogeneous shade determined
from the left side. Organs of mediastinum are displaced to the left. What diagnosis is possible?
A. * Lung athelectasis
B. Lung gangrene
C. Total exudative pleurisy
D. Pneumonia
E. Empyema of pleura
263. A 48-year-old patient complains of fatigue and easy bruising of 3 weeks’ duration. Physical
findings included pale, scattered ecchymoses and petechiae. RBC – 1.95 x 1012/L; Hb – 73 g/L;
HCT 20%; PLT – 23 x 109/L; and WBC – 182 x 109/L with 84% blasts, that contained Auric rods;
peroxidase stain was positive; What is the most probable diagnosis?
A. Megaloblastic anemia
B. Hemolytic anemia
C. Thrombocytopenia
D. * Acute myeloid leukemia
E. Chronic leukemia
264. A 48-year-old woman develops fevers, chills, and icteric sclera. In addition to a fever of 39.2_C,
the physical examination is remarkable for an ill-appearing jaundiced female with right upper
quadrant pain. Ultrasonography reveals a dilated common bile duct with stones in the gallbladder
and in the duct itself. The patient is placed on broad-spectrum antibiotics to cover organisms known
to infect the biliary tract. The procedure most appropriate now is
A. * endoscopic retrograde cholangiopancreatography
B. laparoscopic cholecystectomy
C. placement of an external stent for bilary drainage
D. laparotomy to canulate the common bile duct, remove the stone, and perform a cholecystectomy
E. antibiotics for several days

265. A 49 years old woman complains of weakness, malaise, anorexia, fever, dental bleeding. Petechiae
on a skin. Laboratory findings: Hb of 70 g/L, RBC of 2.2 x 1012/L, the platelet count is 30 x 109/L,
the WBC 100,5 x 109/L, blasts in peripheral blood smear. What is the most probable diagnosis?
A. Megaloblastic anemia
B. * Acute leukemia
C. Hemolytic anemia
D. Thrombocytopenia
E. Chronic leukemia
266. A 49-yr-old man who enjoys drinking presents with pallor, epistaxis and bleeding. On physical
examination you find the spleen to be enlarged and the liver to be slightly enlarged. Choose the
single most likely investigation from the list of options above.
A. * Liver function test
B. Abdominal USG
C. Bone marrow biopsy
D. Blood culture
E. White cell count
267. A 49-yr-old woman presents with goiter. On examination, the thyroid is firm and rubbery. Thyroid
microsomal antibodies are positive in high titer. Choose the most likely diagnosis
A. Thyroglossal cyst
B. * Hashimoto's thyroiditis
C. Hypothyroidism
D. Graves' disease
E. Simple goiter
268. A 50 years old woman complains of weakness, anorexia, fever, gums bleeding. Spleen, liver, and
lymphatic nodes are enlarged, petechiae on a skin. Laboratory findings: the platelet count is 90 x
109/L, WBC 100 x 109/L, blasts 40%. ? What is the most probable diagnosis?
A. * Acute leukemia
B. Megaloblastic anemia
C. Hemolytic anemia
D. Thrombocytopenia
E. Chronic leukemia
269. A 50-year-old man without significant past medical history or recent exposure to alcohol presents
with midepigastric abdominal pain, nausea, and vomiting. The physical examination is remarkable
for the absence of jaundice and any other specific physical findings. Which of the following is the
best strategy for screening for acute pancreatitis? Measurement of both serum amylase and serum
A. * Lipase
B. Measurement of serum lipase
C. Measurement of serum amylase
D. Isoamylase level analysis
E. None of above
270. A 50-yr-old patient complains of burning pain in the feet, worse at night or on walking. He
describes the sensation as like walking on hot coals. Choose the most likely diagnosis
A. Coronary artery disease
B. Cerebrovascular disease
C. Nephropathy
D. Retinopathy
E. * Polyneuropathy

271. A 50-yr-old woman presents to her GP for fatigue, depression and weight gain. She also complains
of constipation and poor memory. On examination, she has puffy face and coarse facial features,
thin eyebrows and a large tongue. Choose the most likely management
A. Propanolol
B. Calciferol
C. Carbimazole
D. * Thyroxine
E. Octreotide (somatostatin)
272. A 50-yr-old woman presents with a thyroid goiter. A core biopsy reveals evidence of lymphoma.
Choose the most likely management.
A. Total thyroid lobectomy
B. Thyroxine
C. * External beam radiation
D. Propranolol
E. Reassure and repeat FNAC in 1 yr
273. A 50-yr-old woman presents with fever, tachycardia restlessness, hypertension and vomiting. On
examination she has diffuse swelling of the thyroid gland and strabismus with diplopia. Choose the
most likely diagnosis.
A. Thyroglossal cyst
B. Hashimoto's thyroiditis
C. Toxic multinodular goiter
D. * Thyroid storm
E. Graves' disease
274. A 51-year-old female comes to the emergency department complaining of left lower quadrant
abdominal pain. She describes an acute illness accompanying the pain with subjective fever and
diarrhea over the last 8 hours. Abdominal exam shows tenderness in the LLQ of the abdomen, no
rebound tenderness at McBurney's point, and negative Murphy's sign. What imaging modality is
most appropriate for this patient?
A. Ultrasound of the abdomen.
B. * CT scan with and without contras
C. Colonoscopy.
D. Barium enema.
E. Plain upright abdominal x-ray.
275. A 51-year-old woman undergoes colorectal cancer screening. She feels well, has no significant
medical history, takes no medications, and has no family history of colorectal cancer. Physical
examination and complete blood count are normal. Which of the following is the most appropriate
screening program for colorectal cancer in this patient?
A. * Colonoscopy every 10 years
B. Flexible sigmoidoscopy every 2 to 3 years
C. Barium enema examination every 3 years
D. Fecal occult blood testing every 2 to 3 years
E. CT colonography (virtual colonoscopy) every 10 years
276. A 52-yr-old man has been gaining weight. He complains of a chronic cough, acne and bruising. On
physical examination you find his legs and arms to be abnormally thin. Choose the most likely
investigation
A. Chest X-ray
B. Serum cortisol
C. * Dexamethasone suppression test
D. Water deprivation test

E. T3, T4 and TSH


277. A 52-yr-old man presents with painless lump in the neck and a chronic cough. Physical
examination finds tachycardia and pallor. He feels that he has lost weight, but he is not certain. He
does not smoke or drink. Choose the most likely diagnosis.
A. Toxic adenoma
B. Adrenal hyperplasia
C. Hyperthyroidism
D. Hypothyroidism
E. * Follicular carcinoma
278. A 53-year-old woman with a history of mild persistent asthma is evaluated because of a recent
increase in her symptoms, with dyspnea and cough occurring daily and a cough that awakens her
once a week. She is currently using low-dose inhaled corticosteroids. She has no symptoms of
rhinitis or gastroesophageal reflux. On physical examination, she has intermittent wheezing
bilaterally. Which of the following is the most appropriate change in her therapy?
A. * Add a long-acting ?2-agonist
B. Initiate azithromycin therapy
C. Add a nebulized short-acting ?2-agonist
D. Add inhaled ipratropium bromide
E. Add a leukotriene inhibitor
279. A 55-year-old male alcoholic has recurrent attacks of severe mid-epigastric pain after eating. Serum
amylase determinations after such attacks have been in the normal range. The examination reveals
mild cachexia but is otherwise unremarkable. On further questioning, the patient states that he has
been sober for the past 10 years but prior to that time had multiple episodes of alcohol-induced
pancreatitis. He is currently taking pancreatic replacement enzymes by mouth. An ERCP reveals a
stricture of the pancreatic duct but is otherwise unremarkable. Computed tomography of the
abdomen reveals calcifications in the pancreas but does not show any evidence of malignancy. The
patient is taking 30 mg of continuous-release morphine sulfate twice a day. The best strategy at this
point would be to
A. * resect the head of the pancreas
B. double the dose of pancreatic replacement enzymes
C. double the dose of morphine
D. institute a low-fat diet
E. begin a continuous search for other causes ofabdominal pain
280. A 55-year-old man is evaluated in the emergency department because of an acute, severe asthma
attack; he is hospitalized in the intensive care unit for aggressive medical therapy and monitoring.
He is expectorating thick greenish sputum. His medical history includes hypertension,
cholecystectomy and glaucoma. Chest radiograph reveals hyperinflation only. Medical therapy in
the emergency department included repeated doses of aerosolized albuterol and ipratropium, as
well as methylprednisolone, 125mg administered intravenously. Peak expiratory flow rate is
unimproved at 80 l/min. Which of the following is the most appropriate next step in this patient’s
A. * management?
Intravenous magnesium sulfate
B. Nebulized ipratropium bromide administered by face mask
C. Broad-spectrum antibiotics targeting community-acquired respiratory pathogens
D. Inhaled corticosteroids
281. A 55-year-old man with no prior medical history presents with right flank pain. During an
evaluation for kidney stones, he has a computed tomography scan of the abdomen that reveals a 2 ?
2-cm mass in the left adrenal gland. The patient’s vital signs, kidney function, and electrolyte levels
are within normal limits. Which of the following tests should be ordered next to evaluate the
A. adrenal mass?and renin levels
Aldosterone
B. Random cortisol levels

C. Testosterone levels
D. * 24-Hour urine collection for metanephrines
E. No further testing
282. A 55-yr-old man presents with fever, sweats and weight loss. He also suffers from gout and
enlargement of lymphatic nodes, which are nontender, symmetrical. On examination he has an
enlarged spleen and liver. Blood tests reveal a lymphocytosis and anaemia. The Philadelphia
chromosome is negative. What is the most probable diagnosis?
A. ALL
B. * CLL
C. Multiple myeloma
D. Polycythemia vera
E. CML
283. A 55-yr-old presented with following reports on a routine screen: Calcium-2.85 mmol/l,
phosphate—0.8, ALP—110, PTH—raised, 25-OH vitamin D—low-normal. Choose the most likely
A. * diagnosis
Primary hyperparathyroidism
B. Tertiary hyperparathyroidism
C. Hypoparathyroidism
D. Hyperthyroidism
E. Paget's disease of bone
284. A 56-year-old patient with cirrhosis of the liver presents with massive hemetemesis. Two large-bore
intravenous lines are placed; somatostatin, fluids, and blood products are administered; and the
patient is intubated. Emergency endoscopy reveals bleeding esophageal varices. The patient
becomes stable hemodynamically but is still bleeding. The most appropriate next step is
A. * endoscopic variceal band ligation
B. intravenous vasopressin
C. balloon tamponade
D. endoscopic injection sclerotherapy
E. intravenous propranolol
285. A 57-year-old man with severe persistent asthma is evaluated on routine follow-up. He states that
his asthma has been under good control for the last 3 months on high-dose inhaled corticosteroids
and a long-acting ?2-agonist. He uses a short-acting ?2-agonist only three times per week as a
rescue medication, and he has nocturnal symptoms very rarely. Peak expiratory flows have been
stable. His physical examination is normal, including clear breath sounds, and spirometry is
normal. It is decided that he will keep using the short-acting ?2-agonist as a rescue medication.
Which of the following is the best next step in this patient’s management?
A. * Continue the long-acting ?2-agonist and decrease the dose of inhaled corticosteroid
B. Stop the long-acting ?2-agonist and decrease the dose of inhaled corticosteroid
C. Continue current therapy and have the patient return in 6 months
D. Stop the long-acting ?2-agonist and maintain the dose of inhaled corticosteroid
286. A 58-year-old patient complains of weight loss, night sweating, sensation of fullness in left
hypochondrium. CBC reveals: RBC 3.9х 1012/L, Hb 118g/L, Ht 41%, MCV 88 fl, Pl 645 х 109/L,
WBC 89,4х 109/L, Segmented Neutrophils 62%, Band Neutrophils 10%, Metamyelocytes 3%,
Myelocytes 5%, Monocytes 4%, Eosinophils 8%, Basophils 6%, Lymphocytes 2%, ESR 27mm/h.
What is the most likely diagnosis?
A. Acute leukemia
B. Chronic myeloid leukemia, accelerated phase
C. Chronic lymphocytic leukemia
D. * Chronic myeloid leukemia, chronic phase
E. Multiple myeloma

287. A 58-year-old woman complains of increasing fatigue and easy bruising of 3 weeks duration.
Physical findings included pale, scattered ecchymoses and petechiae and mild hepatosplenomegaly.
CBC: RBC – 2.55 x 1012/L; Hb – 73 g/L; HCT 20%; PLT – 23 x 109/L; and WBC – 162 x 109/L
with 82% blasts; peroxidase stain is positive. What is the most probable diagnosis?
A. Megaloblastic anemia
B. Hemolytic anemia
C. Thrombocytopenia
D. * Acute leukemia
E. Chronic leukemia
F. Chronic hemolytic anemia
288. A 58-yr-old man was diagnosed with diabetes at a routine medical examination 3 months ago. His
BMI is 32 despite losing 5 kg by following the dietician's advice. His home blood glucose readings
range from 7 to 11 and his HbAlc is 10%. Choose the most likely management.
A. * Metformin
B. No change in treatment required
C. IV insulin sliding scale
D. Gliclazide
E. Dietary adjustment
289. A 58-yr-old woman presents with fever, sweats and weight loss. On examination she has an
enlarged spleen. Blood tests reveal a lymphocytosis and anaemia. The Philadelphia chromosome is
negative. What is the diagnosis?
A. ALL
B. Multiple myeloma
C. * CLL
D. Polycythemia rubra vera
E. CML
290. A 59-yr-old man presents with obstructive jaundice. USG shows no gallstones. The liver appears
normal and the common bile duct measures 12 mm in diameter. His past medical history includes
partial gastrectomy 15 yr ago for peptic ulcer. Choose the single most likely investigation from the
list of options above.
A. Percutaneous transhepatic
B. * Cholangiography
C. MRI scan
D. Barium follow through
E. CTscan
291. A 60-year-old alcoholic man is admitted to the emergency department with hematemesis. His pulse
is 110/min, blood pressure is 100/60 mm Hg, and respirations are 19/min. He has multiple spider
angiomata on his back and chest, with bilateral gynecomastia. Abdominal examination is
significant for hepatosplenomegaly, and his abdomen is distended and tympanic on percussion; a
fluid level is easily detectable. His testicles are small, and a rectal examination produces
guaiac-negative stool. His hematocritis 23%. After placement of a nasogastric tube, 400 mL of
bright red blood is evacuated. After initial fluid resuscitation, which of the following is the most
appropriate next step in management?
A. Barium swallow
B. Esophageal balloon tamponade
C. * Esophagogastroscopy
D. Exploratory celiotomy
E. Selective angiography

292. A 60-year-old man with biopsy-proven hepatic cirrhosis is hospitalized because of massive ascites
and pedal edema. There is no evidence of respiratory compromise or hepatic encephalopathy. Bed
rest, sodium and water restriction, and the administration of spironolactone produce no significant
weight change after 5 days. Which of the following therapeutic measures would be most
appropriate at this time?
A. * Therapeutic paracentesis
B. None of them
C. Oral acetazolamide, 250 mg/d
D. Placement of a peritoneovenous shunt
E. Intravenous furosemide, 80 mg now
293. A 60-yr-old man is brought to A&E in an unconscious state. His glucose is 35 mmol/l. His arterial
blood gas shows a pH of 7.2 and a PaCO2 of 28 mm Hg. Serum Na is 140, K is 3.0, Cl is 100 and
HCO3" is – 18 mEq/L. Choose the most likely management.
A. Sugary drink
B. Insulin sliding scale, Heparin and 0.45% saline
C. * Insulin sliding scale, 0.9% NS and potassium replacement
D. Chest X-ray
E. 50 ml of 50% dextrose IV
294. A 61-year-old patient complains of weight loss, night sweating, general weakness, fatigue,
sensation of fullness in left hypochondrium. CBC reveals: RBC 4.0х 1012/L, Hb 88g/L, Hematocrit
34%, MCV 88 fl, Pl 98 х 109/L, WBC 125,4х 109/L, Segmented Neutrophils 35%, Band
Neutrophils 8%, Metamyelocytes 6%, Myelocytes 5%, Myeloblasts 10%, Monocytes
4%,Eosinophils 8% Basophils 22%, Lymphocytes 2%, ESR mm/h 27mm/h. What is the most likely
A. diagnosis?
Acute leukemia
B. * Chronic myeloid leukemia, accelerated phase
C. Chronic lymphocytic leukemia
D. Chronic myeloid leukemia, chronic phase
E. Multiple myeloma
295. A 61-year-old white man has a 2-month history of dysphagia for solid foods. He has lost 6.7 kg
during this time. The patient has chronic heartburn that is relieved by antacids. He also has
hypertension for which he takes atenolol and diltiazem. Which of the following is the most likely
diagnosis
A. * Esophageal adenocarcinoma
B. Pill-induced esophagitis
C. Esophageal web
D. diffuse esophageal spasm
E. None of above
296. A 64-year-old patient complains of weight loss, night sweating, general weakness, fatigue, nose
bleeding. Physical examination reveals lymphadenopathy, ecchymosis. CBC reveals: RBC 3.00 х
1012/L, Hb 98 g/L, Ht 40 %, MCV 98 fl, Reticulocyte Count 1.0 %, Pl 45 х 109/L, WBC 61.9 х
109/L, Segmented Neutrophils 2 %, Band Neutrophils 1 %, Monocytes -0% Eosinophils -0 %,
Basophils -0%, Lymphocytes 97%, ESR 24mm/h, Smudge cells ++. What is the most likely
diagnosis?
A. Acute leukemia
B. Chronic lymphocytic leukemia III st
C. Chronic lymphocytic leukemia I st
D. Chronic myeloid leukemia
E. * Chronic lymphocytic leukemia IV st

297. A 65-yr-old man has had type 2 diabetes for 4 years, for which he was taking Chlorpropamide. He
presents with an acute MI and his laboratory blood glucose is 11 mmol/l. Choose the most likely
management.
A. Metformin
B. No change in treatment required
C. * IV insulin sliding scale
D. Gliclazide
E. Dietary adjustment
298. A 65-yr-old patient is becoming increasingly confused. She has periods where her confusion seems
to be stable and then seems to rapidly deteriorate in a stepwise progression. On examination there
are extensor plantars but leg reflexes are diminished. Choose the most likely diagnosis
A. Coronary artery disease
B. * Cerebrovascular disease
C. Nephropathy
D. Retinopathy
E. Polyneuropathy
299. A 66-year-old man complains of fever, significant weight loss, bone and joint pain, and bleeding
gums. On examination - paleness, lymphadenopathy, hepato- and splenomegaly. WBC – 170 x
109/L with 13% lymphocytes, 1% monocytes, 21% basophiles, 29% neutrophils, 10% blasts, 12%
promyelocytes, 12% myelocytes, 2% metamyelocytes. ESR – 22 mm/h. The Philadelphia
chromosome positive. Which agent is used for induction or consolidation therapy?
A. Prednisolone
B. * Сytosar
C. Gatifloxacin
D. Vinblastine
E. Alendronic acid
300. A 67-year- male presents with a complaint of fatigue. There is no history of alcohol abuse or liver
disease. Scleral icterus is noted on physical exam. The liver and spleen are nonpalpable. The patient
is noted to have a normocytic, normochromic anemia. The first step in evaluation of this patient is
A. CT scan of the abdomen
B. Hepatitis profile
C. * Liver function tests, including direct versus indirect bilirubin and urine bilirubin
D. Abdominal ultrasound
E. Percutaneous transhepatic Cholangiography
301. A 67-year-old alcoholic man is admitted to the emergency department with hematemesis. His pulse
is 100/min, blood pressure is 100/60 mm Hg, and respirations are 19/min. Abdominal examination
is significant for hepatosplenomegaly, and his abdomen is distended and tympanic on percussion; a
fluid level is easily detectable. His testicles are small, and a rectal examination produces
guaiac-negative stool. His hematocritis 24%. After placement of a nasogastric tube, 420 mL of
bright red blood is evacuated. After initial fluid resuscitation, which of the following is the most
appropriate next step in management?
A. Barium swallow
B. Esophageal balloon tamponade
C. * Esophagogastroscopy
D. Exploratory celiotomy
E. Transjugular intrahepatic portosystemic shunt
302. A 67-year-old male presents with conjugated hyperbilirubinema, with bilirubin detected in the
urine. Serum bilirubin is 12 mg/dL, AST and ALT are in normal range, and alkaline phosphatase is
300 U/L (3 times normal). The next step in evaluation is

A. * Ultrasound or CT scan
B. Hepatitis profile
C. Reticulocyte count
D. Family history for hemochromatosis
E. Colonoscopy
303. A 67-year-old woman had her first colonoscopy 1 month ago for routine colorectal cancer
screening. A 6-mm tubular adenoma of the sigmoid colon was removed. She has no family history
of colorectal cancer. She asks what can be done to decrease her risk of developing colorectal
cancer. Which of the following is the most appropriate surveillance for this patient
A. * Repeat colonoscopy in 5 year
B. Repeat colonoscopy in 1 years
C. Aspirin, 81 mg daily
D. A high-fiber, low-fat diet
E. None of above
304. A 67-yr-old man is noted to have a glucose level of 37 mmol/l and a Na+ of 163 mmol/l. He has no
prior history of diabetes and has been on IV fluids for a week. His other medications include IV
Cefuroxime, Metronidazole, and Dexamethasone. Choose the most likely management.
A. Insulin sliding scale, Heparin and 0.9% saline
B. * Insulin sliding scale, Heparin and 0.45% saline
C. Sugary drink
D. Chest X-ray
E. 50 ml of 50% dextrose IV
305. A 68-year-old man is evaluated because of nausea, vomiting, and upper abdominal pain and
distention of 2 days duration. He has no fever, chills, or jaundice. On physical examination, he
appears uncomfortable and has orthostatic hypotension. Abdominal examination discloses
distention, tympany on percussion, and rushes on auscultation. Serum aspartate aminotransferase is
0,34 U/L, serum alanine aminotransferase is 0,61 U/L, and serum total bilirubin is 19,6 mcmol/L.
Plain radiographs of the abdomen show pneumobilia with multiple air-fluid levels in the jejunum.
No free air is seen. Abdominal ultrasonography shows four gallstones measuring 2 to 4 cm.
Because of pneumobilia, the biliary tree cannot be further visualized. Which of the following is the
most appropriate next step in this patient’s management?
A. * Exploratory laparotomy for bowel obstruction
B. Cholecystectomy
C. Endoscopic retrograde cholangiopancreatography
D. CT scan of the abdomen
E. Magnetic resonance cholangiopancreatography
306. A 68-year-old patient complains of weight loss, night sweating, sensation of fullness in left
hypochondrium. CBC reveals: RBC 3.9х 1012/L, Hb 128g/L, Ht 41%, MCV 78fl, Reticulocyte
Count 1.0 %, Pl 945 х 109/L, WBC 125,4х 109/L, Segmented Neutrophils 62%, Band Neutrophils
10%, Metamyelocytes 3%, Myelocytes 5%, Monocytes 4%, Eosinophils 8%, Basophils 6%,
Lymphocytes 2%, ESR 27mm/h. What is the most likely diagnosis?
A. Acute leukemia
B. Erythremia
C. Chronic lymphocytic leukemia
D. * Chronic myeloid leukemia
E. Multiple myeloma
307. A 68-yr-old patient presents with bone pain, anaemia and renal failure. Her bone marrow reveals
32 % of plasma cells. What is the most probable diagnosis?
A. * Multiple myeloma

B. Myeloid metaplasia
C. AML
D. CLL
E. ALL
308. A 70-year-old male presents with a complaint of fatigue. There is no history of alcohol abuse or
liver disease; the patient is on no medication. Scleral icterus is noted on physical exam. There is no
evidence for chronic liver disease on physical exam, and the liver and spleen are nonpalpable. The
patient is noted to have a normocytic, normochromic anemia. The first step in evaluation of this
patient is
A. CT scan of the abdomen
B. Hepatitis profile
C. * Liver function tests, including direct versus indirect bilirubin and urine bilirubin
D. Abdominal ultrasound
E. Esophagogastroduodenoscopy
309. A 70-yr-old man presents with bone pain, anaemia. His bone marrow reveals plasma cells 30 %.
What is the most probable diagnosis?
A. Myeloid metaplasia
B. AML
C. * Multiple myeloma
D. CLL
E. Megaloblastic anaemia
310. A 70-yr-old woman develops severe hip pain while gardening. Hip joint X-ray shows fracture neck
of femur. She gives a history of lower back pain and malaise. She is tender over the lumbar spine.
ESR is 110 mm/hr. What is your diagnosis?
A. * Multiple myeloma
B. Waldenstorm's macroglobulinaemia
C. CML
D. CLL
E. Acute leukemia
311. A 70-yr-old woman presents to her GP with weight loss and depression. On examination she is
noted to have buccal pigmentation and pigmented scars. She appears dehydrated. Her BP is 100/60
mm Hg. Choose the most likely management.
A. * Long-term replacement with glucocorticoids and mineralocorticoids
B. Calciferol
C. Carbimazole
D. Thyroxine
E. Octreotide (somatostatin)
312. A 71-year-old male presents with a complaint of fatigue. There is no history of alcohol abuse or
liver disease; the patient is on no medication. Scleral icterus is noted on physical exam. The patient
is noted to have conjugated hyperbilirubinema, with bilirubin detected in the urine. Serum bilirubin
is 12 mg/dL, AST and ALT are in normal range, and alkaline phosphatase is 300 U/L (3 times
normal). The next step in evaluation is
A. * Ultrasound or CT scan
B. Hepatitis profile
C. Reticulocyte count
D. Family history for hemochromatosis
E. Esophagogastroduodenoscopy

313. A 75-year-old man has been deferring colon cancer screening because she is afraid to undergo
colonoscopy. She learned of a new technique called virtual colonoscopy that she thinks may be
more tolerable and asks you about the relative merits of this procedure. Which of the following
statements is true regarding virtual colonoscopy?
A. * It detects colorectal cancers and large adenomas quite well, but may miss small polyps
B. It is more acceptable to patients because it does not require any bowel preparation
C. It is a noninvasive procedure that images the colon using ultrasound
D. Its sensitivity and specificity for detecting colon cancers and polyps is similar to that of
conventional colonoscopy
E. It does not require any instrumentation of the bowel
314. A 76-year-old man, who is in a rehabilitation facility after fracturing a leg, develops acute diarrhea.
He has no history of intestinal disorders. The patient has regular colonoscopic screenings. His last
colonoscopy, done 2 years ago, was normal. He has not received antibiotics in the past year. His
roommate is taking oral metronidazole for a diarrheal syndrome that he developed while in the
rehabilitation facility. Use of which of the following would most likely have prevented
development of the patients diarrheal syndrome?
A. * Good hand-washing technique
B. Prophylactic antibiotics
C. Prophylactic probiotic agents
D. Prophylactic loperamid
E. None of above
315. A man of 59 years old complaints of abdominal discomfort, gum bleeding, large ecchymosis,
weakness, sternal tenderness, fever, skin nodules. Laboratory findings: the white cell count 540 x
109/L, levels of basophils, eosinophils and platelets are increased; and a few normoblasts are seen;
Er – 3,1 x 1012/L, blast 48%. What is the most probable diagnosis?
A. Megaloblastic anemia
B. Hemolytic anemia
C. * Acute leukemia
D. Thrombocytopenia
E. Chronic leukemia
316. A man, 35years old, complains of weakness, palpitation, tinnitus dizziness. Data of anamnesis:
peptic gastric ulcer, repeated bleeding. Physical examination: skin is pale. Systolic murmur is heard
at the apex, HR 100 per min, BP 100/70 mm of Hg. CBC: ESR – 2.8 x 1012/L, haemoglobin
content - 69 gr/l, colour index is 0,7. What is your treatment?
A. Iron parenteral
B. Folic acid
C. * iron supplementation oral
D. Vit B12
E. Antibiotics
317. A middle-aged woman complains of irritability and weight loss. She says she has palpitations. On
physical examination you find mild tachycardia and goiter. There are no eye changes. A thyroid
scan determines a single hot nodule. Choose the most likely diagnosis.
A. * Toxic adenoma
B. Adrenal hyperplasia
C. Hyperthyroidism
D. Hypothyroidism
E. Follicular carcinoma
318. A nursing student has just completed her hepatitis B vaccine series. On reviewing her laboratory
studies (assuming she has no prior exposure to hepatitis B), you expect

A. Positive test for hepatitis B surface antigen


B. * Antibody against hepatitis B surface antigen (anti-HBS) alone
C. Antibody against hepatitis core antigen (anti-HBC)
D. Antibody against both surface and core antigen
E. Antibody against hepatitis E antigen
319. A paracentesis is performed on 43-year-old patient with long-standing alcohol abuse. On physical
exam, there are spider angiomas and palmar erythema. Abdominal collateral vessels are seen
around the umbilicus. The serum albumin minus ascitic fluid albumin equals 1.4 g/dL. The most
likely diagnosis is
A. * Portal hypertension
B. Pancreatitis
C. Tuberculous peritonitis
D. Hepatoma
E. No ascitis
320. A patient presents to a physician with severe jaundice. Physical examination reveals a nodular,
enlarged liver. In addition to the generalized nodularity of the liver, the physician can feel one
nodule that is much larger than the others. CT of the abdomen confirms multinodular cirrhosis and
demonstrates a 7-cm mass near the lower border of the liver. CT-guided biopsy of this mass shows
a malignant tumor derived from hepatic parenchymal cells. Which of the following risk factors is
most strongly associated with the development of this tumor?
A. Anatoxin exposure
B. Hemochromatosis
C. * Hepatitis В virus infection
D. Opistharchis infection
E. Thorotrast exposure
321. A patient who has long-standing diabetes mellitus and severe, burning pain in the feet and hands as
a result of peripheral neuropathy asks the physician why an antidepressant has been prescribed.
What is the physician’s best response?
A. “Many people experiencing chronic pain become depressed.”
B. * “The antidepressants may counteract the chemicals causing your pain.”
C. “You are less likely to become addicted from using antidepressants than you are from using other
types of pain killers.”
D. “The antidepressants also have strong anti-inflammatory properties and can reduce the pain you
have from inflammation.”
E. None of above
322. A patient who is 2 days postoperative from a bowel resection tells her physician that she is having a
hard time “catching her breath,” feels nauseated, and has chest pains when she inhales. The
physician suspects that she is having a pulmonary embolism. What intervention should the
physician perform before notifying the physician?
A. Increase the IV flow rate
B. * Apply oxygen by mask or nasal cannula at 5 l/min
C. Assess the chest and axillary area for the presence of petechiae
D. Place the patient in shock position, with her head and neck flat and her legs elevated
E. Non of above
323. A woman 22 year-old, complains of general weakness, shortness of breath, brittle nails, hair loss.
She has menorrhagia. Physical examination reveals paleness of skin, systolic murmur on
auscultation. What changes do you expect in her blood analysis?
A. Increased level of serum iron
B. High colour index

C. * Decreased level of serum iron


D. Increased level of free bilirubin
E. Decreased amount of thrombocytes
324. A young man presents with a neck lump. It is painless and had been bothering him for the past 4
months. He has no other symptoms or signs. On palpation you find the lump to be single discrete
not particularly hard and confined to the thyroid gland itself. His cervical lymph nodes are
enlarged. Choose the most likely diagnosis.
A. Toxic adenoma
B. Adrenal hyperplasia
C. Hyperthyroidism
D. Hypothyroidism
E. * Papillary carcinoma
325. A young woman complains of wheeze, dyspnoea and cough. She cannot sleep at night because of a
chronic cough. She and her mother love animals and together they have 14 cats. Her PEFR is
normal but her CXR suggests hyperinflation. What is the previous diagnosis?
A. * Bronchial asthma
B. Bronchogenic carcinoma
C. Emphysema
D. Respiratory failure
E. Bronchitis
326. After emotional exertion patient of 24 y.o. developed dyspnea with prolonged expiration, distant
wheezes, frequent night symptoms. Such changes limit his physical activity. PEV and FEV1 <
60 %, daily variability -30 %. What diagnosis is possible?
A. * Severe persistent bronchial asthma
B. Mild persistent bronchial asthma
C. Persistent bronchial asthma of moderate severity
D. Intermittent bronchial asthma
E. Bronchospastic syndrome of allergic origin
327. An 14-yr-old boy presents with painful bones, jaundice and anaemia. He is noted to have
splenomegaly. His blood film reveals target cells. What is your diagnosis?
A. Hemolytic anemia.
B. * Sickle cell anaemia
C. Iron-deficiency anemia.
D. B12-deficiency anemia
E. Aplastic anemia.
328. An 18-yr-old girl complains of her appearance. She is much too fat, she says. She also complains of
missed periods and hairiness. On physical examination you find her to be 10 kg overweight.
Choose the most likely investigation
A. Abdominal ultrasonography
B. Serum cortisol
C. * Dexamethasone suppression test
D. Water deprivation test
E. T3, T4 and TSH
329. An 69 year old asymptomatic woman was detected to have a monoclonal spike on serum
electrophoresis (IgG levels 1.5 g/dl). Bone marrow revealed plasma cells of 30%. What is the
possible diagnosis?
A. * Multiple myeloma
B. Indolent myeloma

C. Monoclonal gammopathy of unknown significance


D. Waldenstorm's macroglobulinemia
E. Amyloidosis
330. Drug abuser, a 41-yr-old man, presents with fever, cough and breathlessness. This was preceded by
viral influenza. Chest radiograph shows multiple abscesses. What is the most possible etiology of
disease?
A. * Staphylococcus aureus
B. Cryptococcus
C. Streptococcus pneumoniae
D. Legionella pneumonia
E. Mycobacterium avium
331. Female B., 44 years old, complains on cough with mucous sputum, increase of temperature to
39 °С, weakness, dyspnea, sweating. Breathing rate - 26/min, skin is moist. Below left scapula
there is shortening of percussion sound. Breathing during auscultation is weakened, moist rales.
Blood test: L - 11х109/l, ESR - 29 mm/h. Your previous diagnosis?
A. * Left-side lower lobe pneumonia
B. Gangrene of lungs
C. Left-side exudative pleurisy
D. Cancer of left side lower lobe
E. Pulmonary abscess
332. Female patient K., 46 years old, has flue with fever. After a while she noticesthoracic pain, cough
with yellow-green sputum (amount-150 ml a day), sometimes with some blood. Objectively:
breathing rate - 36/min. In lungs from the right side lower scapula there is dull sound during
percussion, hard breathing, and moist rales. Blood test: L - 18,6х109/l, ESR -64 mm/h. Analysis of
sputum: L -80-100 , Er - 40-50, elastic fibers, cocci. X-ray: lung roots are enlarged, from the right
side lower lobe is heterogeneously infiltrated with two lighter areas. What is the most possible
previous diagnosis?
A. * Right-side pneumonia with abscesses
B. Peripheral cancer
C. Infiltrative tuberculosis in the phase of disintegration
D. Exudative pleurisy
E. Infarction-pneumonia
333. Female, 34 years old, has an increase of body temperature to 38 °С, cough with purulent sputum,
weakness, dyspnea, pain in a thorax during breathing. During percussion there is shortening of
sound in the lower part of left lung, during auscultation – moist rales. What method of investigation
is the decisive one to confirm diagnosis?
A. * X-ray examination
B. Bacteriological analysis of sputum
C. Spirometry
D. Pneumotachometry
E. Bronchography
334. Girl, 18 y.o., pets seller, complaints mainly during working time on the attacks of dry cough,
running nose. She often suffers from acute viral respiratory infections. Her mother is ill with
bronchial asthma. Objectively: breathing rate - 18/min. Heart rate - 80/min, BP - 110/70. In lungs
vesicular breathing, dry wheezes are heard in distance. Tones of heart are weaker than normally.
Test with berotec showed reversibility of bronchial obstruction. What tactic will be the best for the
A. * patient?
To change job
B. To use intal
C. To use monteleucast

D. To use berotec constantly


E. To use antihystaminic preparations
335. Girl, 23 y.o., for 2 years is ill with bronchial asthma. Recently attacks of dyspnea became more
frequent and started to arise 4-5 times a week, night attacks - 2-3 times a month. She used
salbutamol to remove the symptoms. Test with the antigen of home dust is positive. Objectively:
condition is satisfactory. Breathing rate - 20/min. Heart rate - 76/min, BP -120/80. In lungs
breathing is vesicular. Tones of heart are a little weak, rhythm is normal. What mechanism is
desicive in development of bronchial obstruction in this case?
A. * Hyperreactivity of bronchi
B. Тrachео-bronchial dyskinesia
C. Violation of metabolism of arachidonic acid
D. Adrenergic disorders
E. Activity of the parasympathetic nervous system is increased
336. In the biochemical profile of patient B., 29 yr-old, it is elevated serum T4 and low radioactive
iodine uptake. Choose the most likely diagnosis.
A. Non-toxic goitre
B. Hashimoto's thyroiditis
C. * Subacute thyroiditis
D. Hypothyroidism
E. Graves' disease
337. In the biochemical profile of patient J., 38 yr-old, who has a neck mass, it is normal T3 and T4.
Choose the most likely diagnosis.
A. * Non-toxic goitre
B. Hashimoto's thyroiditis
C. Subacute thyroiditis
D. Hypothyroidism
E. Graves' disease
338. In the biochemical profile of patient L., 56 yr-old., it is normal TSH, free T4 and T3. Decreased
serum total T4. Choose the most likely diagnosis.
A. Non-toxic goitre
B. Hashimoto's thyroiditis
C. * Thyroid binding globulin deficiency
D. Hypothyroidism
E. Graves' disease
339. In the biochemical profile of patient S., 43 yr-old, it is elevated serum T4 and increased radioactive
iodine uptake. Choose the most likely diagnosis.
A. Non-toxic goitre
B. Hashimoto's thyroiditis
C. Subacute thyroiditis
D. Hypothyroidism
E. * Graves' disease
340. Male patient F., 48 years old, during a week stayed at home with diagnosis of respiratory viral
infection. Doctor noticed complaints on cough with small amount of mucus-purulent sputum,
weakness. Objectively: condition is relatively satisfactory. T - 37,2 °С. Breathing rate - 18/min.,
pulse - 80/min., BP - 110/70. In lungs there is vesicular breathing, with a hard tint, single dry
wheezes. Tones of heart are a little dull, rhythm is correct. What is the treatment tactic?
A. * To prescribe antibacterial therapy
B. To stay at home for some more days
C. To go to work
D. To send patient to pulmonologist
E. To hospitalize patient to the pulmonological department
341. Male patient G., 56 years old, complaints on permanent pain in a thorax which disturbs for last 2
months. Pain is not connected with breathing. There is also cough with particles of blood in
sputum. Weakness, fatigue are present. On the chest X-ray in the lower lobe of right lung there is
spherical shadow, size 4x6 cm, related to the lung root. What is the most possible diagnosis?
A. * Perypheral lung cancer
B. Tuberculoma
C. Metastasis
D. Pulmonary abscess
E. Pneumonia
342. Man, 32 y.o., complaints on attack of expiratory dyspnea, which lasts for 48 hours, cough with
small amount of sputum. He is ill with bronchial asthma for 5 years, was treated with
glucocorticosteroids, used inhalers. Objectively: condition is severe, patient sits. Diffuse cyanosis,
pulse -110/min, BP - 110/70. Tones of heart are weak, II tone is louder above the pulmonary artery.
During percussion in lungs there is “bang-box” sound, large amount of dry wheezes. In blood there
is eosinophylia - 18 %. What medicines are drugs of choice for this patient?
A. * Corticosteroids
B. ?2-agonists
C. Theophyllin
D. Cholynolytics
E. Antihystamines
343. Man, 39 y.o., 8 last years is ill with bronchial asthma. Rapidly during physical work he felt
worsening of breathing, cough, distance wheezes appeared and dyspnea began to increase.
Medicine of what pharmacological group is it better to recommend for the patient to remove such
attacks of dyspnea?
A. * Agonists of ?2-adrenoreceptors
B. Methylxantines
C. ?2-adrenoblockers
D. Inhalated glucocorticoids
E. Oral glucocorticoids
344. Man, 43 y.o., complaints on dyspnea during physical activity. Objectively: temperature 36,4 °С,
breathing rate - 20/min, pulse - 78/min, BP-125/80. Emphysematous form of thorax. In lungs –
weak vesicular breathing. What test should be passed by patient at home to decide question about
efficiency of prescribed broncholytics?
A. * Peakflowmetry
B. Spirography
C. ECG-control of overload of right chambers of heart
D. Bronchoscopy
E. Analysis of sputum (amount and microscopy)
345. Man, 46 y.o., suffers for the last 10 years from bronchial asthma. Rapidly during physical work he
felt worsening of breathing, cough, distance wheezes appeared and dyspnea began to increase.
Medicine of what pharmacological group is it better to recommend for the patient to remove such
attacks of dyspnea?
A. * Agonists of ?2-adrenoreceptors
B. Atropine
C. Intal
D. Epinephrine
E. Monteleucast

346. On a routine blood examination a 43-yr-old woman is found to have very high serum calcium level.
She has complained recently of bouts of abdominal pain and recurrent UTI. On physical
examination you find an enlarged thyroid gland. Choose the most likely diagnosis.
A. Toxic adenoma
B. Adrenal hyperplasia
C. Hyperthyroidism
D. Hypothyroidism
E. * Parathyroid carcinoma
347. Over a 2-month period, a 50-year-old woman with a history of polycythemia vera develops
abdominal pain and gross ascites. Physical examination demonstrates smooth hepatomegaly and
mild jaundice. Pressure applied over the liver fails to distend the jugular veins. The abdominal wall
is grossly edematous and shows a tortuous venous pattern. Edema of the legs is prominent. Which
of the following is the most likely diagnosis?
A. Hepatocellular carcinoma
B. Primary sclerosing cholangitis
C. Steatosis
D. * Budd-Chiari syndrome
E. Hepatic cirrhosis
348. Patient A., 35 y.o., noticed infrequent (rarer than 1 time a week) attacks of dyspnea, which are
easily removed with inhalations of ?2-agonists of short action. During attack in lungs are heard dry
wheezes, between attacks FEV1 is more than 80 % from normal. What is the diagnosis?
A. * Intermittent bronchial asthma
B. Persistent bronchial asthma of moderate severity
C. Mild persistent bronchial asthma
D. Severe persistent bronchial asthma
E. Given information is not enough for determination of severity of bronchial asthma
349. Patient A., 45 years old, suffers from chronic cholecystits during last 5 years. She was admitted to
the emergency department with acute pain in right hypochondriac area and high temperature.
Laboratory findings reveal leucocytosis, high ESR. Put preliminary diagnosis.
A. * Chronic cholecystitis, acute phase.
B. Chronic cholecystitis, subacute phase.
C. Chronic cholecystitis, phase of remission.
D. Dyskinezia of bile ducts.
E. Rotor’s syndrome.
350. Patient B., 25 years old engineer, appeared during a fire in the area of high concentration of CO (an
industrial accident). In hospital delivered in the irresponsible state. What laboratory tests are the
early criteria of estimation of severity of the state?
A. Estimation of blood viscosity
B. Anemia
C. Leucocytosis
D. * Carboxihemoglobinemia
E. Methemoglobinemia
351. Patient complaints on attacks of dyspnea, which arise 1-2 times a week, night symptoms - 2 times a
month and even more frequently. For a patient night sleep is violated as a result of attacks of
dyspnea. FEV1 > 80 % from normal. What diagnosis would you suspect?
A. * Mild persistent BA
B. Severe persistent BA
C. Intermittent BA
D. Moderate persistent BA

E. Status asthmaticus
352. Patient D., 56 yr-old, complains on muscle weakness, bradycardia and hypotension. ECG shows
tall peaked T waves. Choose the most likely diagnosis.
A. Hypokalaemia
B. Hyponatraemia
C. Hypervitaminosis A
D. * Hyperkalaemia
E. Hypoglycaemia
353. Patient F., 46 years old, was hospitalized urgently with acute attack of dyspnea. Last 5 years he has
been working on poultry farm. During examination bronchial asthma was diagnosed. What
additional diagnostic methods are necessary to confirm the professional genesis of asthma?
A. sanitary-hygienic characteristics of the work conditions
B. echocardioscopy
C. * allergic and immunological tests
D. investigation of the function of external breath
E. roentgenography of pulmonary system
354. Patient G., 36 years old, works on a poultry factory. She was emergently hospitalized with acute
attack of dyspnea. During observation bronchial asthma was diagnosed. What additional methods
of research must be conducted above all things to confirm the professional genesis of bronchial
A. asthma?
roentgenologic research of breathing organs
B. professional route of patient
C. sanitary-hygienic characteristic of work conditions
D. research of external breathing function
E. * allergic and immunological tests
355. Patient G., 38 yr-old, complains on muscle weakness and ectopic beats. ECG shows flattened or
inverted T waves. Choose the most likely diagnosis.
A. * Hypokalaemia
B. Hyponatraemia
C. Hypervitaminosis A
D. Hyperkalaemia
E. Hypoglycaemia
356. Patient G., 47 y.o., with long history of bronchial asthma, has developed more frequent attacks of
dyspnea. Inhalations of astmopent and berotec are not effective. From prescription of what
medicine is it better to begin the intensive treatment?
A. * Glucocorticoids
B. Oxygen therapy
C. Bronchodylators
D. Infusion therapy
E. Heart glycosides
357. Patient has severe attack of bronchial asthma which lasts more than 1 hour. Usage of beta-agonists
in inhalation, euphylline intravenously and cholynolytics was not effective. What medicines are
necessary for emergency therapy?
A. * Glucocorticosteroids intravenously
B. Beta-agonists intravenously
C. Inhaled glucocorticosteroids
D. Antihystaminic
E. Nonsteroid anti-inflammatory medicines

358. Patient J., 36 y.o., complains for fever (39 C), pain in the left part of the chest. Pleuropneumonia
was diagnosed in the patient. What onset is typical for pleuropneumonia?
A. * Acute
B. Latent
C. Fulminant
D. Gradual
E. Non of the above
359. Patient J., 38 yr-old, complains on sweating, palpitations, tremors, drowsiness and fatigue. Choose
the most likely diagnosis.
A. Hypokalaemia
B. Hyponatraemia
C. Hypervitaminosis A
D. Hyperkalaemia
E. * Hypoglycaemia
360. Patient J., 45 y.o., complaints on dyspnea during insignificant physical exertion, cough with
minimal amount of “glass-like” sputum, attacks of dyspnea up to 3 times a day, more often at night,
sweating. She is ill for more than 5 years. Has an allergy on dust, cockroaches. For treatment uses
bekotid for near the year. Diagnosis?
A. * Bronchial asthma
B. Eosinophylic pulmonary infiltrate
C. COPD
D. Bronchiectasis with bronchial spasm
E. Pulmonary vasculitis (syndrome of Charg - Stross)
361. Patient L, 41 years is suffering from diabetes type 1 for 16 years, receiving insulin therapy. He
complaints of swelling and shortness of breath. On examination: glycemia - 7.1 mmol / l,
cholesterol - 6.2 mmol / l, Creatinine - 0.21 mmol / l. What will you recommend the patient first of
A. all?
Increase the dose of insulin
B. ACE inhibitors
C. Statins
D. Lipoic acid
E. * Enterosorbents
362. Patient M., 30 years old, during last 3 years works as a nurse in manipulations cabinet. Last year
during the contact with penicilline she started to complain on discomfort in throat, sneezing, attack
of cough and dyspnea which disappear after inhalation of salbutamol. During last months attacks of
dyspnea became more severe and occurred only at contact with penicilline. During the life she had
not any diseases including allergic. She hadn’t received antibiotics. Can we consider the bronchial
asthma is professional in this patient?
A. no, we can’t
B. yes, we can if we have conclusion about attacks of bronchial asthma
C. yes, we can if we have conclusion about appearance of bronchial asthma attacks after contact with
penicilline
D. yes, we can
E. * yes, we can, if allergic and immunological tests are positive
363. Patient S., 43 yr-old, has perioral paraesthesia, carpopedal spasm and generalised seizures. Choose
the most likely diagnosis.
A. Hypokalaemia
B. Hyponatraemia
C. * Hypocalcaemia
D. Hyperkalaemia

E. Hypoglycaemia
364. Patient U., 44 yr-old, complains on severe abdominal pain, nausea, vomiting, constipation, polyuria
and polydipsia. Choose the most likely diagnosis.
A. * Hypercalcaemia
B. Hyponatraemia
C. Hypervitaminosis A
D. Hyperkalaemia
E. Hypoglycaemia
365. Patient Y.,49 y.o., complaints on dyspnea, cough. Sputum is absent. Used many puffs of
salbutamol, intal, but without any efficacy. Objectively: sits, leaning on a table. Total cyanosis of
the body. Peripheral edema is absent. Breathing is superficial, dyspnea, during auscultation
breathing cannot be heard in some areas of lungs; wheezes are diffuse, expiration is considerably
prolonged. Tones of heart are weak, tachycardia. Pulse - 112/min, BP - 110/70. Liver is near the
edge of costal arch. What is the previous diagnosis?
A. * Status asthmaticus
B. Bronchial asthma of moderate severity
C. COPD
D. Aspiration of foreign body
E. Heart asthma
366. Patient, 28 y.o., has running nose, attacks of dyspnea at night once a week. Felt ill after viral
respiratory infection which was treated with acetilsalicylic acid. Eosynophylia was found in blood
and sputum. What disease may be suspected?
A. * Aspirin bronchial asthma
B. Eosinophylic infiltrate of lungs
C. Bronchial asthma of physical exertion
D. Allergic rhinitis
E. Bronchial asthma, exogenous form
367. Patient, 30 y.o., after a viral infection has daily symptoms of dyspnea, which causes lowering of
activity and bad sleep; night symptoms are more frequent than once a week. PEV and FEV1 -
60-80 %, daily variability > 30 %. There is a necessity of daily usage of ?2-agonists of short action.
What is the diagnosis?
A. * Persistent bronchial asthma of moderate severity
B. Mild persistent bronchial asthma
C. Intermittent bronchial asthma
D. Severe persistent bronchial asthma
E. Status asthmaticus
368. Patient, 42 years old, complaints on attacks of dyspnea, every time uses 1-2 doses of salbutamol.
These attacks are accompanied with cough and minimal amount of viscid glassy sputum. He is ill
for 8 years. Objectively: temperature - 36,7 C; breathing rate – 21/min.; pulse-90/min.; BP -
130/80; FEV1 - 77 %. In lungs – solitary dry wheezes. Blood test: eosinophyles - 6 %. What
medicines are “basic” in the treatment of this patient?
A. * Antiinflammatory
B. Cholynolytics
C. Mucolytics
D. Antihystaminic
E. ?2-agonists

369. Patient, 44 y.o., complaints on attack of dyspnea, which arises suddenly at night. Connects this
attack with overcooling. He is ill for more than 10 years. Thorax of emphysematous form. During
percussion in lungs – “bang-box” sound. During auscultation there is plenty of dry wheezes. In
blood: moderate leucocytosis, eosinophylia - 10 %. On the chest X-ray film – increased
pneumatization of pulmonary tissue. What diagnosis is the most possible?
A. * Bronchial asthma, exacerbation phase
B. Bronchiectasis, exacerbation phase
C. COPD, exacerbation phase
D. Chronic bronchitis
E. Eosinophylic pulmonary vasculitis
370. PatientI., a 50 years old man, is evaluated in the emergency department because of fever,
nonproductive cough and 2-day history of myalgia and headache. He has also had nausea and
diarrhea. He is a heavy smoker. On physical examination, he is slightly disoriented. Temperature is
38.9 C, pulse rate is 110/min, respiration rate is 32/min. Chest radiograph shows fluffy infiltrates to
the right upper and lower lobes. Results of laboratory testing show serum sodium of 128 meq/L,
blood urea nitrogen of 42 mg/dL, serum creatinine of 2.2 mg/dL, and serum creatine kinase of 250
U/L. Which one of the following is best next step in the management of this patient’s pneumonia?
A. * Initiate empiric antibiotic therapy for Legionella
B. Order direct fluorescent antibody testing of the sputum for Legionella
C. Order serologic testing for Legionella
D. Send a urine specimen for measurement of Legionella antigen
E. All of the above
371. Physical examination of patient revealed lymphadenopathy, splenomegaly. CBC: RBCs - 3,6 x
1012/l, Hb- 87 g/l, Pl – 45 x 109/l, WBCs – 13 x 109/l, blasts - 87%, band neutrophils - 1%,
segmented neutrophils - 7%, lymphocytes - 5%, ESR - 55 mm/h. What is the most likely diagnosis?
A. * Acute leukemia
B. Erythremia
C. Chronic lymphocytic leukemia
D. Chronic myeloid leukemia
E. Multiple myeloma
372. The 30-year-old woman whose father has type 1 diabetes mellitus asks the physician what her
chances are of developing diabetes because of her father's disease. The risk for becoming diabetic
A. * is:20-50%
B. 0%
C. 50%
D. 50-80%
E. None of above
373. The 45-year-old diabetic patient has proliferative retinopathy, nephropathy, and peripheral
neuropathy. What should the physician teach this patient about exercise?
A. “The type of exercise that would most efficiently help you to lose weight, decrease insulin
requirements, and maintain cardiovascular health would be jogging for 20 minutes 4 to 7 days each
week.”
B. “Considering the complications you already have, vigorous exercise for an hour each day is
needed to prevent progression of disease.”
C. “Considering the complications you already have, you should avoid engaging in any form of
exercise.”
D. * “Swimming or water aerobics 30 minutes each day would be the safest exercise routine for you.”
E. None of above

374. The patient diabetic patient asks the physician why it is necessary to maintain blood glucose levels
no lower than about 74 mg/dL. What is the physician’s best response?
A. “Glucose is the only fuel form used by body cells to produce energy needed for physiologic
activity.”
B. * “The central nervous system, which cannot store glucose, requires a continuous supply of glucose
for fuel.”
C. “Without a minimum level of glucose circulating in the blood, erythrocytes cannot produce ATP.”
D. “The presence of glucose in the blood counteracts the formation of lactic acid and prevents
acidosis.”
E. None of above
375. The patient getting ready to engage in a 30-minute, moderate-intensity exercise program performs a
self-assessment. Which data indicate that exercise should be avoided at this time?
A. * Ketone bodies in the urine
B. Weight 1 pound higher than the week before
C. Blood sugar level of 155 mg/dL
D. Pulse rate of 66 beats/min
E. None of above
376. The patient has clinical signs which may suggest acromegaly. Which of the following is the most
specific test to diagnose acromegaly?
A. Magnetic resonance imaging of the pituitary gland
B. Measurement of growth hormone (GH) levels during an oral glucose tolerance test
C. * Measurement of insulin-like growth factor I serum levels
D. Measurement of random GH blood levels
E. An octreotide scan
377. The patient newly diagnosed with type 2 diabetes tells the physician that since he has increased his
intake of fiber, he is having loose stools, flatulence, and abdominal cramping. What is the
physician’s best response?
A. “Decrease your intake of water and other fluids.”
B. * “Decrease your intake of fiber now and gradually add high-fiber foods back into your diet.”
C. “You must have allergies to high-fiber foods and will need to avoid them in the future.”
D. “Taking an antacid 1 hour before meals or 2 hours after meals should reduce the intensity of your
bowel problems.”
E. None of above
378. The patient on an intensified insulin regimen consistently has a fasting blood glucose between 270
and 280 mg/dL, a postprandial blood glucose level below 200 mg/dL, ketones in urine ++and a
hemoglobin A1c level of 8.5%. What is the physician’s interpretation of these findings?
A. * The patient has developing night hypoglycemia.
B. The patient is at increased risk for developing hyperglycemia.
C. The patient is demonstrating signs of insulin resistance.
D. The patient is demonstrating good control of blood glucose.
E. None of above
379. The patient tells the physician that he enjoys having a glass of wine on Saturdays when dining out
with friends. He asks if having type 1 diabetes will prohibit him from this activity. What is the
physician’s best response?
A. “Insulin activity is dramatically reduced under the influence of alcohol and drinking even one
glass of wine will increase your insulin requirements.”
B. “Diabetics have decreased kidney function and should avoid ingesting alcohol in all forms at all
times.”

C. “You shouldn't drink any alcohol because it is likely to increase your sense of hunger and make
you overeat.”
D. * “One glass of wine can be ingested with a meal and is counted as two fat exchanges.”
E. None of above
380. The patient with diabetes is visually impaired and wants to know if syringes can be prefilled and
stored for use later. What is the physician’s best response?
A. * “Yes, prefilled syringes can be stored for up to 3 weeks in the refrigerator in a vertical position
with the needle pointing up.”
B. “Yes, prefilled syringes can be stored for up to 3 weeks in the refrigerator, placed in a horizontal
position.”
C. “Insulin reacts with plastic, so prefilled syringes must be made of glass.”
D. “No, insulin cannot be stored for any length of time outside of the container.”
E. None of above
381. The patient with hyperthyroid symptoms is having hormone studies done to confirm the diagnosis.
Which set of values indicates Graves’ disease hyperthyroidism?
A. Elevated T3, elevated T4, high TSH levels
B. * Elevated T3, normal T4, low TSH levels
C. Elevated T3, low T4, high TSH levels
D. Low T3, normal T4, high TSH levels
E. None of above
382. The patient with hypothyroidism as a result of Hashimoto’s thyroiditis asks the physician how long
she will have to take thyroid medication. What is the physician’s best response?
A. “You will need to take the thyroid medication until the goiter is completely gone.”
B. None of above
C. “The thyroiditis will be cured with antibiotics, and then you will no longer need the thyroid
medication.”
D. * “You will need thyroid replacement hormone therapy for the rest of your life because the thyroid
gland function will not return.”
E. “When your thyroid function studies indicate a normal blood level of thyroid hormones, you will
be able to discontinue the medication.”
383. The patient, 51 years old, suffers from diabetes 7 years, gets glibenclamide 15 mg a day. Complains
of shortness of breath and pitting edema. On examination of blood pressure - 180/110 mm Hg.
Laboratory: glycemia - 6.2 mmol / L, creatinine - 0.15 mmol / L, glomerular filtration - 62 ml /
min. What medication will you recommend to the patient?
A. Gliclazide
B. * Gliquidone
C. Glimepiride
D. Pioglitazone
E. Repaglinide
384. When taking the blood pressure of a patient after a parathyroidectomy, the physician notes that the
patient's hand has gone into flexion contractions. What is the physician’s interpretation of this
observation?
A. Hypokalemia
B. Hyperkalemia
C. Hyponatremia
D. * Hypocalcemia
E. None of above
385. Which the following type of anemia can be normocytic normochromic anemia with reticulocytosis?
A. * Haemolytic anaemia

B. Vitamin B12 deficiency anaemia


C. Acute posthemorrhagic anaemia
D. Chronic iron deficiency anaemia
E. Aplastic anaemia
386. Which the following type of anemia can be with Cabot rings?
A. Vitamin B12 deficiency anaemia
B. Acute posthemorrhagic anaemiaChronic iron deficiency anaemia
C. Aplastic anaemia
D. * Haemolytic anaemia
Назва наукового напрямку (модуля): Семестр: 12
Внутрішня медицина 6 курс
Опис:
тести
Перелік питань:
1. Bilateral malignant pleural effusion is most often seen in:
A. * Cancer of lung
B. Cancer of breast
C. Mesothelioma
D. Lymphoma
E. Nothing of the above
2. Bilateral pleural effusion is seen in:
A. Congestive cardiac failure
B. Nephritic syndrome
C. Constrictive pericarditis
D. * All of the above
E. Nothing of the above
3. Bilateral subtotal of thyroidectomy for Graves’ disease most likely can cause hypoparathyroidism
as a result as:
A. Loss of thyrocalcitonin
B. Reduction in parathyroid stimulating hormone
C. Removal of the four parathyroids
D. * Infarction of the parathyroids
E. Secondary hypopituitarism
4. Blocking of these structures can worsen the asthma
A. * ?2 -adrenoreceptors
B. ?-adrenoreceptors
C. Mast cells
D. Neutrophils
E. Eosinophils
5. Bone marrow transplant is the treatment of choice in all, except:
A. Aplastic anemia
B. AML in 1st remission
C. ALL in second remission
D. * ITP
E. Lympoma
6. Bronchial asthma is associated with raised levels of
A. * Leukotrienes
B. Thromboxane
C. Antinuclear antibodies
D. Corticosteroids
E. All of the above
7. Bronchiectasis is the most common in such lung lobe:
A. * Left lower
B. Right middle
C. Right upper
D. Left upper
E. Nothing of the above
8. Causes of haemorrhagic pleural effusion are all except:
A. * Bronchial adenoma
B. Pulmonary infarction
C. Mesothelioma
D. Tuberculosis
E. Nothing of the above
9. CBC revealed a WBC of 72,000 and a differential that was 83% lymphocytes. What is the MOST
likely diagnosis?
A. Acute lymphocytic leukemia
B. * Chronic lymphocytic leukemia
C. Acute prolymphocytic leukemia
D. Chronic prolymphocytic leukaemia
E. Lymphoma
10. Characteristic(s) of gastric mucosal acid secretion:
A. * all of the above
B. from parietal cells founded in mucosal glands of fundusof the stomach
C. stimulated by muscarinic cholinergic system (parietal cell innervation)
D. oxidative dephosphorylation dependent
E. None of above
11. Charcot’s joint in diabetes is NOT seen in:
A. None of them
B. Knee
C. Hip
D. Tarsal
E. * Elbow
12. Chose correct answer. Zollinger-Ellison syndrome -
A. * gastrin-secreting islet cell tumor
B. it is associated with Helicobacter pylori
C. it is associated withLamblia
D. All of the above
E. None of above
13. Common sites for cushing ulcers include all of the following except:
A. * Distal duodenum
B. Stomach
C. Is1 part of duodenum
D. Esophagus
E. None of above
14. Common X-ray finding of Staphylococcal pneumonia:
A. * Bilateral hilar lymphadenopathy
B. Involvement of lower lobes
C. Dense homogenous consolidation
D. Visible air bronchogram
E. Nothing of the above
15. Commonest site of peptic ulcer is:
A. * 1st part of Duodenum

B. Middle part of duodenum


C. Distal 1/3 of stomach
D. Pylorus of the stomach
E. None of above
16. Consider the following: Serum iron (1), MCV (2), serum total iron binding capacity (3), serum
ferritin (4). Which of these decreased in iron deficiency anaemia?
A. 1 and 2
B. 1, 3 and 4
C. * 1, 2 and 4
D. 1, 2, 3 and 4
E. 2,3 and 4
17. Constipation is caused by
A. * aluminum hydroxide
B. magnesium hydroxide
C. All of above
D. None of above
E. sodium bicarbonate
18. Contraindication for glucocorticosteroid therapy of bronchial asthma is:
A. * Bleeding from gastric ulcer
B. Chronic persistant hepatitis
C. Exacerbation of respiratory-heart failure
D. Emphysema of lungs
E. Respiratory failure
19. Curschmann's spirals in sputum are seen in:
A. * Bronchial asthma
B. Bronchiectasis
C. Chronic bronchitis
D. Wegenersgranulomatosis
E. All of the above
20. Dawn phenomenon refers to:
A. Early morning hyperglycemia followed by hypoglycemia
B. Hypoglycaemia followed by hyperglycemia
C. Early morning hypoglycemia
D. * Early morning hyperglycemia
E. High insulin level
21. Deaths from lactic acidosis in diabetes mellitus are associated with therapy:
A. Glibenclamide
B. Gliquidone
C. Chlorpropamide
D. * Phenformin
E. Glipizide
22. Decreased maximum mid-expiratory flow rate indicates obstruction in:
A. * Small airways
B. Trachea
C. Large airways
D. Trachea &amp;amp;amp; bronchi both

E. Nothing of the above


23. Diabetic ketoacidosis characterized by:
A. Low serum potassium
B. * Increased anion gap
C. Metabolic alkalosis
D. Respiratory acidosis
E. Hypoglycemia
24. Diagnostic tests for H. Pylori include all of the following except:
A. * Nechyporenco test
B. Rapid urease test
C. Gastric biopsy &amp;amp;amp; Warthin-starry stain
D. Urea-breath test
E. None of above
25. Drug of choice for inhalation in acute asthma not responding to salbutamol is:
A. * Ipratropium bromide
B. Atropine sulfate
C. Isopropamide
D. Hyoscine methyl bromide
E. All of the above
26. Drug of choice in asthma with heart disease is
A. * Ipratropium
B. Rimiterol
C. Terbutaline
D. Cromolyn sodium
E. All of the above
27. Drug of choice in Zollinger Ellison syndrome is
A. * Omeprazole
B. Ranitidine
C. Antacids
D. B-blocker
E. Atropin
28. Drug(s) used in eradication of Helicobacter pylori infection:
A. * All of above
B. metronidazole (Flagyl)
C. clarithromycin (Biaxin)
D. bismuth compounds
E. amoxicillin (Amoxil Polymox)
29. During an epidemic of hepatitis E, Fatality is maximum in :
A. * Pregnant women
B. Infants
C. Malnourished male
D. Adolescents
E. None of above
30. Early gastric cancer generally indicates
A. * Gastric adenocarcinoma confined to the mucosa &amp;amp;amp; submucosa
B. Gastric adenocarcinoma confined to the mucosa

C. Gastric adencarcinoma detected early


D. Gastric adenocarcinoma less than 1 cm In size
E. None of above
31. Endoscopic biopsy from a case of H. pylori related duodenal ulcer is most likely to reveal
A. * Antral predominant gastritis
B. Multifocal atrophic gastritis
C. Acute erosive gastritis
D. Gastric atrophy
E. None of above
32. Epidemiological studies of H. pylori are done by using:
A. * Serological markers
B. Urea-breath test
C. Culture
D. Gastric-biopsy urease test
E. None of above
33. Eradication of H. pylori effectiveness for bismuth compounds when using combination with
antibiotics:
A. * 95%
B. 20%
C. 50%
D. 80%
E. 5%
34. Example(s) of "aggressive factors" in peptic ulcer disease:
A. * pepsin
B. bicarbonate
C. prostaglandins
D. neither
E. All of above
35. Example(s) of "agressive factor(s)" in peptic ulcer disease:
A. * Helicobacter pylori
B. prostaglandins
C. gastric mucus
D. bicarbonate
E. all of the above
36. Example(s) of "protective factor(s)" in peptic ulcer disease:
A. * all of the above
B. prostaglandins
C. gastric mucus
D. bicarbonate
E. None of above
37. Examples of H2 (histamine receptor Type II) antagonists:
A. * All of above
B. ranitidine (Zantac)
C. nizatidine (Axid)
D. famotidine (Pepcid)
E. cimetidine (Tagamet)

38. Exercise induced asthma is not precipitated by :


A. * Swimming in hot water
B. High altitude climb and exercises
C. Cycling in cold weather
D. Swimming in cold water
E. All of the above
39. FEV1/FVC is reduced in case of:
A. * Asthma
B. Pleural effusion
C. Lung fibrosis
D. All of the above
E. Nothing of the above
40. Findings in hemophilia A include:
A. * Partial thromboplastin time increased
B. Increased clotting time
C. Increased prothrombin time
D. Bleeding time increased
E. Thrombocytopenia
41. Following are causes of unconjugated hyperbilirubinemia, except:
A. * Rotor syndrome
B. Large hematoma
C. Hemolytic anemia
D. Megaloblastic anemia
E. Irondeficiency anemia
42. For the patient who has sustained tracheobronchial trauma, which assessment finding alerts the
physician to the possibility of tracheal lacerations?
A. Hypertympanic sound on affected side
B. * Subcutaneous emphysema over the trachea
C. Hypotension and decreased capillary refill
D. Deviation of the trachea to the affected side
E. Non of above
43. Haemophilia A is characterised by the presence of following features, except
A. Bleeding into soft tissues
B. Reduced VIII levels
C. * Prolonged bleeding time
D. Prolonged partial thromboplastin time
E. Bleeding into muscles &amp;amp;amp; joints
44. Helicobacter pylori is not associated with :
A. * Gastric leiomyoma
B. Gastric cancer
C. Peptic ulcer of stomach
D. Peptic ulcer of duodenum
E. None of above
45. Histamine and gastric acid secretion:
A. * all of the above
B. release enhanced by increased cholinergic activity

C. most important gastric acid secretion stimulant


D. released from enterochromaffin-like cells
E. None of above
46. How long is the usual course of drug treatment for a patient with active tuberculosis?
A. 7 to 10 days
B. 6 weeks
C. * 6 months
D. 2 years
E. Non of above
47. Hypercalcemia may be treated by:
A. IV pamidronate
B. IV fluids + furosemide
C. Glucocorticoids
D. None above them
E. * All of them
48. Hypergastrinemia with hypochlorhydria is seen in:
A. * Pernicious anemia
B. All of above
C. Zollinger Ellison Syndrome
D. Glucagonoma
E. None of above
49. Hyperosmolar hyperglycemic non-ketonic coma is characterized by such level of glycemia:
A. * 55 mmol/1
B. 20 mmol/1
C. 30 mmol/1
D. 5 mmol/1
E. 25 mmol/1
50. Hypoglycemic coma can be caused by:
A. Insulin deficiency
B. Dehydration
C. * Extreme activity
D. Metabolic acidosis
E. Hypokalemia
51. Hypoglycemic effect of intermediate acting insulin lasts for:
A. * 12-16 hours
B. 20 -30 hours
C. 0.5 – 1.0 hours
D. 4 – 6 hours
E. 6 – 8 hours
52. Hypoglycemic unawareness that occurs in diabetic patients when transferred from oral
hypoglycemic agents to insulin, is due to :
A. * Autonomic neuropathy
B. Insulin resistance
C. Allergy
D. Somogi phenomenon
E. Down phenomenon

53. In a case of hypertrophic pyloric stenosis, the metabolic disturbance is


A. * Metabolic alkalosis with paradoxical aciduria
B. Metabolic acidosis
C. Respiratory alkalosis
D. Metabolic alkalosis with alkaline urine
E. None of above
54. In a patient only Anti HBsAg is positive in serum, all other viral markers are negative. It indicates :
A. * Immunized person with hepatitis B vaccine
B. Chronic active hepatitis
C. Persistent carrier
D. Acute hepatitis
E. None of above
55. In bronchial asthma glucocorticoids
A. * Reduce airway inflammation
B. Act as potent bronchodilators
C. Inhibit degranulation of mast cells
D. Block the action of humoral mediators
E. Nothing of the above
56. In endemic goiter such statement will be true:
A. Investigations invariably show some degree of hypothyroidism
B. There is a clear evidence of an increased incidence of carcinoma of the thyroid
C. Administration of thyroxine will often cause the goitre to shrink
D. Administration of iodine occasionally precipitates hyperthyroidism
E. * All of them
57. In gastric outlet obstruction in a peptic ulcer patient, the site of obstruction is most likely to be
A. * Duodenum
B. Antrum
C. Pylorus
D. Pyloric canal
E. None of above
58. In patients with nonketotic hyperosmolar coma the level of glycemia is :
A. Slightly elevated
B. Mildly elevated
C. Moderately elevated
D. May be decreased
E. * Grossly elevated
59. In pneumonia due to Mycoplasma all of the following are true except:
A. * Bilateral infiltration on chest X-ray
B. Myalgia
C. Dry cough
D. Pleuritic chest pain
E. All of the above
60. In restrictive lung disease :
A. * FEV1/FVC is high
B. FVC is high
C. FEV1 is high

D. All of the above


E. Nothing of the above
61. In severe bronchial asthma, it is present:
A. * Hyperresonant chest with prolonged expiration
B. Infrequent rhonchi and absent breath sounds
C. Increased fremitus and crackles
D. Decreased fremitus and crepitation
E. All of the above
62. In the time of attack of bronchial asthma in lungs are heard
A. * Dry loud wheezes
B. Moist loud wheezes
C. Moist soundless wheezes
D. Voice of pleural friction
E. Crepitation
63. In what specific way does the respiratory system function contribute to acid-base balance?
A. Prevents excessive loss of hydrogen ions by evaporation
B. Increases the potassium ion content in bronchial secretions
C. * Removes carbon dioxide generated as a result of metabolism
D. Maintains body water levels through mucous membrane function
E. Non of above
64. In what specific way does the respiratory system function contribute to acid-base balance?
A. Prevents excessive loss of hydrogen ions by evaporation
B. Increases the potassium ion content in bronchial secretions
C. * Removes carbon dioxide generated as a result of metabolism
D. Maintains body water levels through mucous membrane function
E. Non of above
65. In which case the dose of insulin in stable diabetics can be reduced:
A. Thyrotoxicosis
B. Steroid therapy
C. Pregnancy
D. * Chronic renal failure
E. None above them
66. Increased amylase may be seen in all of the following except:
A. * Appendicitis
B. Pancreatic pseudocyst
C. Perforated peptic ulcer
D. Ruptured ectopic pregnancy
E. None of above
67. Increased incidence of duodenal ulcer associated with:
A. * All of above
B. alcoholic cirrhosis
C. renal transplantation
D. systemic mastocytosis
E. None of above
68. Increased incidence of duodenal ulcer associated with:
A. * All of above

B. None of above
C. liver cirrhosis
D. chronic renal failure
E. hyperparathyroidism
69. Increased RBC level may be seen in:
A. Glomerulonephritis
B. Vitamin D excess
C. * Cor pulmonale
D. Acute heart failure
E. Pneumonia
70. Insulin resistance is associated with all of the following status, EXCEPT:
A. Acanthosis nigricans
B. Lipodystrophy
C. * Gout
D. Obesity
E. Metabolic syndrome
71. Intestinal hypomotility is seen in all the following except:
A. * All of the above
B. Parkinsonism
C. Amyloidosis
D. Diabetes
E. None of above
72. Intrinsic factor for absorption of vitamin Bl2 is secreted by the :
A. * Parietal cells
B. Peptic cells
C. Chief cells
D. Goblet cells
E. Endothelial cells
73. It is not Antacid
A. * Histamine
B. magnesium hydroxide
C. calcium carbonate
D. sodium bicarbonate
E. All of above
74. Location(s) of histamine in gastric mucosa:
A. * enterochromaffin-like cells (ECL) + mast cell cytoplasmic granules
B. mast cell cytoplasmic granules
C. enterochromaffin-like cells (ECL)
D. neither
E. are not localized in gastric mucosa
75. Macrophages containing large quantities of undigested and partial digested bacteria in intestine are
seen in
A. * Whipple's disease
B. Amyloidosis
C. Immunoapproliferative small instetinal disease
D. Vibro cholerae infection

E. None of above
76. Massive bleeding per rectum in a 70 yr old patient is due to :
A. * Diverticulosis
B. Carcinoma colon
C. Colitis
D. Polyps
E. None of above
77. Mechanism of action: cytoprotection, binds to the ulcer base, antibacterial effect:
A. * bismuth compounds
B. tetracycline (Achromycin)
C. metronidazole (Flagyl)
D. ranitidine (Zantac)
E. omeprazole (Prilosec)
78. Mechanism of Idiopathic thrombocytopenic purpura is:
A. Vasculitis
B. Antibodies to vascular epithelium
C. * Antibodies to platelets
D. Antibody to clotting factors
E. Fungal infectious
79. Mechanism(s) by which somatostatin reduces gastrin release:
A. * inhibits parietal cells secretion+ inhibits histamine release by enterochromaffin-like cells
B. inhibits histamine release by enterochromaffin-like cells
C. inhibits parietal cells secretion
D. stimulates parietal cells secretion
E. None of above
80. Melphalan is used in:
A. Wilm's tumor
B. Lymphoma
C. * Multiple myeloma
D. Nephroblastoma
E. ALL
81. Metabolic changes associated with excessive vomiting includes the following:
A. * Hypokalemia
B. Hyperchloremia
C. Metabolic acidosis
D. Decreases bicarbonates
E. None of above
82. Microcytic anemia is seen in:
A. Sprue
B. Pernicious anemia
C. Latun infestation
D. * Iron deficiency anemia
E. Aplastic anemia
83. Monotherapy effectiveness in eradication of H. pylori-- bismuth compounds
A. * 20%
B. 5%

C. 50%
D. 90%
E. 100%
84. More likely to affect the cytochrome P450 drug metabolizing system:
A. * cimetidine (Tagamet)
B. All of above
C. nizatidine (Axid)
D. ranitidine (Zantac)
E. famotidine (Pepcid)
85. Most common CNS manifestation of Whipple's disease is :
A. * Dementia
B. Supranuclear ophthalmoplegia
C. Seizure
D. Cerebellar ataxia
E. None of above
86. Most common complication of acute pancreatitis is:
A. * Pseudocyst
B. Pancreatic abscess
C. Phlegmon
D. Pleural effusion
E. None of above
87. Most common extraintestinal manifestation of Crohn's disease of small bowel is.
A. * Erythema Nodosum
B. Ankylosing Spondylitis
C. Iritis
D. Ureteral Obstruction
E. None of above
88. Most common sign of aspiration pneumonitis:
A. * Tachypnoe
B. Bronchospasm
C. Cyanosis
D. Crepitations
E. Nothing of the above
89. Most sensitive and specific test for diagnosis of iron deficiency is:
A. * Serum ferritin levels
B. Serum iron levels
C. Serum transferrin receptor population
D. Transferrin saturation
E. Hb, Ht
90. Most successful protocol for eradication of H. pylori/treatment of peptic ulcer disease:
A. * therapy using bismuth compounds, Clarythromycin (Flagyl), and amoxicillin (Amoxil Polymox)
B. bismuth monotherapy
C. bismuth compounds and amoxicillin (Amoxil Polymox)
D. all of the above equally effective
E. None of above
91. Most widely used antacid in treating ulcer disease:

A. * combination of aluminum hydroxide and magnesium hydroxide


B. calcium carbonate
C. magnesium hydroxide
D. aluminum hydroxide
E. sodium bicarbonate
92. Mycoplasma infection simulates:
A. * Viral pneumonia
B. Pnemuococcal pneumonia
C. Pulmonary oedema
D. Pulmonary infarction
E. Nothing of the above
93. Non-parenteral hepatitis is :
A. * Hepatitis E
B. Hep B
C. Hep C
D. Hep D
E. All of above
94. Omeprazole (Prilosec) and lansoprazole approved for clinical treatment of:
A. * All of above
B. erosive gastritis
C. Zollinger-Ellison syndrome and other gastric acid hypersecretory states
D. None of above
E. duodenal ulcer
95. Only class of antiulcer drugs that can eradicate Helicobacter pylori and cure associated gastritis:
A. * colloidal bismuth
B. sucralfate (Carafate)
C. H2 blockers
D. Anticholinergic agents
E. None of above
96. Oral anticoagulant therapy is monitored with indexes
A. * INR
B. PTT
C. TT
D. Clot lysis time
E. Bleeding time
97. Pancreatic pseudocyst most commonly occurs after
A. * Pancreatitis
B. Trauma
C. Pancreatic surgery
D. Pancreatic malignancy
E. None of above
98. Paroxysmal nocturnal haemoglobinuria is due to:
A. Antibodies
B. Congenital membrane defect
C. Inherited autoimmune disorder
D. * Complement mediated RBC lysis

E. Lysis of RBC in spleen


99. Pathogenic factor(s) in duodenal ulcer:
A. * all of the above
B. genetic factor
C. cigarette smoking
D. alcoholic cirrhosis
E. COPD (chronic obstructive pulmonary disease)
100. Patient D. presents elevated serum T4 and increased radioactive iodine uptake. Put diagnosis,
A. please.
Non-toxic goiter
B. Hashimoto's thyroiditis
C. Subacute thyroiditis
D. Hypothyroidism
E. * Graves' disease
101. Patient F., 29 yr. old, presents with subacute thyroiditis. Treatment will include:
A. Antibiotics
B. Vitamins
C. Antithyroid drugs
D. Iodine preparations
E. * Corticosteroids
102. Patient on heparin therapy should be monitored with:
A. * Clotting time
B. Bleeding time
C. Prothrombin time
D. Fibrinogen
E. PTT
103. Patient presents with recurrent duodenal ulcer of 2.5 cm size; procedure of choice
A. * Truncal vagotomy and antrectomy
B. Truncal vagotomy and gastrojejunostomy
C. Highly selective vagotomy
D. Laparoscopic vagotomy and gastrojejunostomy
E. None of above
104. Peptic ulcer classification:
A. * duodenal+ gastric
B. gastric
C. duodenal
D. neither
E. cardiac
105. Peptic ulcer should be differentiated with
A. * All of above
B. Cholecystitis
C. Gastritis
D. Miocardial infarction
E. Pancreatitis
106. Pernicious anemia is seen in:
A. Iron deficiency
B. Chronic liver disease

C. Bleeding
D. * Atrophic gastritis
E. Hemolysis
107. Platelet count is decreased in all of the following condition except:
A. Idiopathic thrombocytopenic purpura
B. Thrombotic thrombocytopenic purpura
C. Systemic lupus erythematosus
D. * Haemophilia
E. Hemolytic uremic syndrom
108. Pneumothorax is a possible complication of
A. * Staphylococcal pneumonia
B. Pneumococcal pneumonia
C. Klebsiella pneumonia
D. Viral pneumonia
E. All of the above
109. Presentation of hypothyroidism includes:
A. * Menorrhagia
B. Oligomenorrhoea
C. Amenorrhea
D. All of them
E. None above them
110. Primary atypical pneumonia is caused by:
A. * Mycoplasma
B. Mycobateriumkansasii
C. Str. pneumoniae
D. Pneumocystis carinii
E. Nothing of the above
111. Primary hyperparathyroidism most likely caused by:
A. Multiple parathyroid adenomas
B. * Solitary parathyroid adenoma
C. Adrenal hyperplasia
D. Ectopic PTH production
E. None above them
112. Pseudopolyps are features of
A. * Ulcerative colitis
B. Crohn's disease
C. Celiac sprue
D. Whipple's disease
E. None of above
113. pylori is known to cause all of the following except:
A. * Fundal atrophic gastritis
B. Duodenal ulcer
C. Gastritis type B
D. Gastric ulcer
E. None of above
114. Raised serum amylase levels are used to diagnose

A. * Acute pancreatitis
B. Degenerative diseases
C. Acute cholecystitis
D. Autoimmune disease
E. None of above
115. Ranitidine (Zantac) and gastric acid secretion:
A. * inhibits basal acid secretion+ inhibits secretion in response to vagal stimulation or feeding
B. inhibits secretion in response to vagal stimulation or feeding
C. inhibits basal acid secretion
D. neither
E. stimulates basal acid secretion
116. Relapse rate for duodenal ulcer following H. pylori eradication:
A. * 15%
B. 75%
C. 50%
D. 90%
E. 5%
117. Reserve transcriptase of hepatitis B virus is coded on the following gene :
A. * P gene
B. S gene
C. C gene
D. X gene
E. All of above
118. Risk factor for development of gastric Carcinoma
A. * Intestinal metaplasia type III
B. Duodenal ulcer
C. Intestinal hyperplasia
D. Blood group O
E. None of above
119. Screening method for medullary carcinoma thyroid is:
A. * Serum calcitonin
B. Serum calcium
C. Serum alkaline phosphate
D. Serum acid phosphatase
E. None above them
120. Secondary hyperparathyroidism is associated with:
A. Parathyroid adenoma
B. Marked hypercalcaemia
C. * Chronic renal failure
D. Parathyroidectomy
E. All of them
121. Serum unconjugated bilirubin and urine urobilinogen concentration are elevated in
A. anemias of chronic disease
B. * hemolytic anemias
C. anemia of acute hemorrhage
D. aplastic anemias

E. iron deficiency anemia


122. Sharko-Leiden crystals are seen in:
A. * Bronchial asthma
B. Bronchiectasis
C. Chronic bronchitis
D. Wegenersgranulomatosis
E. All of the above
123. Side effect of antiulcer medication: urinary retention, blurred vision, xerostromia:
A. * atropine
B. ranitidine (Zantac)
C. nifedipine (Procardia, Adalat)
D. sucralfate (Carafate)
E. None of above
124. Significant high ESR is seen in :
A. Polycythemia vera
B. CHF
C. * Multiple myeloma
D. Sickle cell anemia
E. CLL
125. Skip granulomatous lesions are seen in
A. * Crohn's disease
B. Ulcerative colitis
C. Whipple's disease
D. Reiter's disease
E. None of above
126. Small capillaries bleeds in:
A. * Thrombocytopenia
B. Coagulation failure deficiency
C. Anemia
D. Agranulocytosis
E. Hemophilia
127. Somogi phenomenon characterized by :
A. * Early morning hyperglycemia
B. Early morning hypoglycemia
C. Hypoglycemia followed by hyperglycemia
D. High insulin levels
E. Night hyperglycemia
128. Spherocytosis is best diagnosed by:
A. Splenic puncture
B. BM aspiration
C. Plasma
D. * Peripheral blood smear
E. Phenotyping
129. Such radiological findings as "hour-glass" vertebrae and "triturated pelvis" are seen in:
A. Thyrotoxicosis
B. Myxedema

C. Cretinism
D. * Hyperparathyroidism
E. Hypothyroidism
130. Such results of investigations as positive Chvostek sign and elevated serum parathyroid hormone
(PTH) level are associated with:
A. * Pseudohypoparathyroidism
B. Primary hypoparathyroidism
C. Primary hyperparathyroidism
D. Osteoporosis
E. Hypercalcemia of malignancy
131. The actions of insulin includes which one of the following?
A. * Increased utilization of glucose by muscle
B. Increased hepatic output of glucose
C. Increased lipolysis
D. Increased amino acid flux from muscle due to protein breakdown
E. Activated glycogenolysis
132. The adenocarcinoma of esophagus-developed in
A. * Barret's esophagus
B. Long standing achalasia
C. Corrosive structure
D. Alcohol abuse
E. None of above
133. The commonest acute presentation of sickle cell anaemia is:
A. Priapism
B. * Bone pain
C. Fever
D. Splenomegaly
E. Pulmonary hypertension
134. The commonest hepatotropic virus progressing to chronicity is:
A. * HCV
B. HAV
C. HBV
D. HEV
E. HAV+ HEV
135. The first virological marker following acute infection with HBV is :
A. * HBs Ag
B. Anti HBs Ag
C. IgM anti HBc Ag
D. Anti HBe Ag
E. All of above
136. The following laboratory index is abnormally prolonged in ITP:
A. APTT
B. * Bleeding time
C. Prothrombin time
D. Clotting time
E. None of above

137. The frequent causes of hypoglycemia in hospitalized diabetic patients include all the following
EXCEPT:
A. Insulin
B. Sulfonylureas
C. * Adrenal insufficiency
D. Alcohol
E. Renal failure
138. The high level of C-peptide can be found in a case of:
A. Glucagonoma
B. * Insulinoma
C. Gastrinoma
D. Hepatoma
E. In all of them
139. The histological features of coeliac disease include all of the following, except:
A. * Increase in thickness of the mucosa
B. Crypt hyperplasia
C. Increase in intraepithelial lymphocytes
D. Increase in inflammatory cells in lamina propyria
E. None of above
140. The lowest recurrence of peptic ulcer is associated with
A. * Vagotomy + Antrectomy
B. Vagotomy + drainage
C. Gastric resection
D. Highly selective vagotomy
E. Resection of stomach by Bilroth II
141. The main cause of hypercalcemic crisis is :
A. * Parathyroid adenoma
B. CA Breast
C. Parathyroid hyperplasia
D. Paget's disease
E. Hyperthyroidism
142. The main cause of hyperosmolar coma is:
A. Insulin deficiency
B. * Dehydration
C. Extreme activity
D. Hypoxia
E. Intercurrent infection
143. The main cause of lactic acidosis is
A. Insulin deficiency
B. Dehydration
C. Extreme activity
D. * Hypoxia
E. Intercurrent infection
144. The major sign of hypoventilation is
A. * Cyanosis
B. Hypoxia

C. Hypercapnia
D. Dyspnea
E. All of the above
145. The most common cause of aplastic anemia is:
A. * Idiopathic
B. Chloramphenicol
C. Phenylbutazone
D. Petroleum products
E. Prednisolone
146. The most common cause of thyroid crisis is:
A. * Thyroid surgery
B. 131I administration
C. Administration of Beta blockers
D. Thyrostatic therapy
E. All of the above
147. The most common cause of thyrotoxicosis is:
A. * Grave's disease
B. Follicular adenoma
C. Multinodular goiter
D. Subacute thyroiditis
E. Iatrogenic
148. The most common community acquired infection is :
A. * Streptococcal pneumonia
B. Hepatitis A
C. Cholera
D. Meningitis
E. Nothing of the above
149. The most common presentation of primary hyperparathyroidism is:
A. Bone fracture
B. Increased serum creatinine
C. Osteitis fibrosa cystica
D. * Asymptomatic hypercalcemia
E. Kidney stones
150. The most common route of spread of hepatitis E is
A. * Feco-oral
B. Blood transfusion
C. injection IV
D. perinatal transmission
E. All of above
151. The most effective correction of acidosis in diabetic acidosis is:
A. IV bicarbonate
B. IV Saline
C. Oral bicarbonate
D. * IV Insulin
E. IV potassium
152. The most important regulator of serum 1,25(OH)2 vitamin D concentration is

A. Serum calcium
B. Serum magnesium
C. Serum 25(OH) vitamin D
D. TSH
E. * Parathyroid hormone
153. The most pre-cancerous condition for carcinoma colon is
A. * Familial polyposis
B. Hamartomatous polyps
C. Juvenile polyps
D. Hyperplastic polyps
E. None of above
154. The patient is taking enalapril (Vasotec), an angiotensin-converting enzyme (ACE) inhibitor, for
hypertension. Which respiratory side effect should you teach the patient to expect?
A. Wheezing on exhalation
B. Increased nasal stuffiness
C. Chest pressure or pain
D. * Persistent dry cough
E. Non of above
155. The place of 25-hydroxycholecalciferol formation is:
A. * Liver
B. Kidney
C. Intestines
D. Pancreas
E. Brain
156. The principal role of the kidneys in the synthesis of the active form of vitamin D is:
A. Conversion of 7-dehydrocholesterol to vitamin D3
B. Absorption of vitamin D2
C. Conversion of vitamin D to 25-OH vitamin D
D. Absorption of vitamin D3
E. * Conversion of 25-OH vitamin D to 1,25-(OH)2 vitamin D
157. The results of the glucose tolerance test: a fasting serum glucose is 6,5 mmol/l, 2-hour postprandial
serum glucose is 14,6 mmol/l. What is your diagnosis?
A. Normal.
B. Impaired glucose tolerance.
C. Impaired fasting glycemia.
D. * Diabetes mellitus.
E. Neither.
158. The shortest period of the action is present in such group of preparations as:
A. Thiazolidinediones
B. Biguanides
C. Sulfonilureas
D. Acarbose
E. * Glinides
159. The triad originally described by Zollinger-Ellison syndrome is characterized by:
A. * Peptic ulceration, gastric hypersecretion, non beta cell tumour
B. Peptic ulceration, gastric hypersecretion, beta cell tumor
C. Peptic ulceration, achlorhydria, non beta cell tumour

D. Peptic ulceration, achlorhydria, beta cell tumour


E. None of above
160. The true statement of tertiary hyperparathyroidism is:
A. Primary hyperparathyroidism with decrease Ca+ level
B. * Secondary hyperparathyroidism with chief cell adenoma
C. Secondary hyperparathyroidism following intestinal malabsorption
D. Metastasis with normal phosphate level
E. Secondary hyperparathyroidism following chronic renal failure
161. Thrombocytopenia, neutropenia are seen in
A. anemias of chronic disease
B. hemolytic anemias
C. anemia of acute hemorrhage
D. * aplastic anemias
E. iron deficiency anemia
162. Thyroid carcinoma associated with hypocalcemia is
A. Follicular carcinoma
B. * Medullary carcinoma
C. Anaplastic carcinoma
D. Papillary carcinoma
E. All of them
163. To remove onset of tetany you will order
A. * Calcium chloride
B. Diuretics
C. Prednisolone
D. Potassium citrate
E. Magnesium sulphate
164. Toxic megacolon is most commonly associated with
A. * Ulcerative colitis
B. Crohn's disease
C. Whipple's disease
D. Reiter's disease
E. None of above
165. Treatment of choice for aplastic anemia is:
A. Methotrexat
B. ampicillini
C. chloramphenicol
D. * Bone marrow transplantation
E. Plasmaphoresis
166. Treatment of gastric ulcer disease
A. * atropine is less effective than H2 receptor blockers
B. atropine is more effective than H2 receptor blockers
C. atropine and H2 receptor blockers are equally effective
D. atropine and H2 receptor blockers are not effective
E. Omeprazol is not used
167. Treatment of neonatal idiopathic thrombocytopenia is
A. Azathioprin

B. Dexamethasone
C. * Immunoglobulins
D. Platelets transfusion
E. Splenectomy
168. True about idiopathic thrombocytopenic purpura is:
A. * Increase megakaryocytes in bone marrow
B. Frequent joint haemorrhage
C. Most common in children
D. Males commonly affected
E. VIII factor deficiency
169. True about Ulcerative Colitis with malignancy
A. * Is related to duration of ulcerative colitis
B. Is related to disease activity
C. It has a better prognosis
D. Malignancy is more in anorectal ulcerative colitis
E. None of above
170. True statement about Hypercalcemia :
A. Treatment of the primary cause is effective
B. Malignancy does not produce hypercalcemia
C. * i.v. fluid with Furosemide is given
D. Pamidronate is not effective.
E. i.v. fluid with calcium chloride is effective
171. What is a possible level of platelets in the patient with Acute Lymphoblastic Leukemia?
A. * 100.000 – 150.000/L
B. Over 200.000/L
C. 250.000– 300.000/L
D. Below 300.000/L
E. Absence of Pl
172. What is drug of choice for ulcerative colitis :
A. * 5-amino salicylic acid
B. Prednisolone
C. Mercaptopurine
D. Salazopyrine
E. None of above
173. What is not true about blind loop syndrome?
A. * Surgery is almost always required to correct small bowel syndrome
B. Megaloblastic anemia is commonly seen
C. It manifets as diarrhoea, weight loss and deficiency of fat soluble vitamins.
D. Broad spectrum antibiotics are the treatment of choice
E. None of above
174. What is not true about the immune mechanism in the small bowel?
A. * Ig A acts by activating the complement pathway
B. Intestine contains more than 70% of IgA producing cells in the body
C. Ig A is produced by plasma cells in the lamina propria
D. Approximately 60% of the lymphoid cells are T cells
E. None of above

175. What is the Acute Lymphoblastic Leukemia?


A. * Disorder of the blood-forming tissue (white cells)
B. Disorder of the blood-forming tissue (red cells)
C. Leukocytosis
D. Increasing of blood pressure
E. Decreasing of blood pressure
176. What is the best marker for prognosis of the multiple myeloma presents with bony lesions:
A. Serum calcium level
B. Beta 1a microglobulin
C. Beta 1 microglobulin
D. * Bone marrow plasma cell
E. Number of lytic lesions
177. What is the most necessary in order to diagnose bronchial asthma:
A. * To notice the asthmatic attack of dyspnea
B. To find allergic antibodies
C. To find the presence of expiration dyspnea
D. To find the presence of signs of obstructive respiratory failure
E. To find the presence of eosinophylia in blood
178. What is true of haemophilia:
A. * Increased PTT
B. Platelets level decreased
C. Increased factor VIII
D. Increased factor IX
E. BT decreased
179. What is true regarding congenital hypertrophic pyloric stenosis:
A. * Hypochloremic alkalosis
B. More common in girls
C. Hellers myotomy is the procedure of choice.
D. Most often manifests at birth
E. None of above
180. Which among the following diseases may be causative agent for B12 deficiency anaemia?
A. Chronic gastritis of type B
B. Peptic gastric ulcer
C. * Chronic gastritis of type A
D. Chronic pancreatitis
E. Chronic cholecystitis
181. Which amongst the following is the earliest indicator of hypothyroidism?
A. Decreased serum T4
B. Decreased serum T3
C. Increased relaxation phase of deep tendon reflexes
D. * High serum TSH
E. High serum TRH
182. Which changes of a tongue are typical for vitamin B12 deficiency anaemia?
A. * Geographic tongue
B. Red
C. Coated

D. Clean
E. Swollen
183. Which does not predispose to Carcinoma stomach?
A. * Low Nitrate consumption
B. Salted meat and fish
C. Low fat and protein diet
D. HIgh Complex carbohydrate consumption
E. All of above
184. Which drug is mostly used for the treatment of chronic lymphatic leukemia?
A. Prednisone
B. * Chlorambucil
C. Methotrexate
D. Myleran
E. Ampicilin
185. Which drug is not effective against H. pylori:
A. * Erythromycin
B. Metronidazole
C. Amoxicillin
D. Colloidal Bismuth
E. None of above
186. Which from the following symptoms are characteristic for anaemia?
A. Petechia
B. Cyanosys
C. * Pallor of skin and mucous membranes
D. Edema
E. Hyperpigmentation
187. Which index can differentiate factitious hyperinsulinemia from insulinoma:
A. * C – peptides
B. Insulin antibodies
C. Serum glucose levels
D. None
E. All of them
188. Which is not a feature of multiple myeloma?
A. * Cutaneous nodules
B. Bony lesions
C. Renal failure
D. 'M' Spike
E. Hypercalciemia
189. Which is not a paraneoplastic syndrome for Hepatocellular Carcinoma
A. * Hyperglycemia
B. Hypoglycemia
C. Erythrocytosis
D. Hypercalcemia
E. None of above
190. Which of the following can help to put a diagnosis of subclinical hypothyroidism for a patient?
A. Low thyroid hormone levels but no symptoms

B. Classic symptoms of hypothyroidism but normal thyroid function test results


C. Low free T4 levels but normal serum thyroid-stimulating hormone (TSH) level
D. Low serum TSH but normal free T4 levels
E. * Increased serum TSH but normal free T4 levels
191. Which of the following conditions causes delayed, deep tissue-type bleeding?
A. Uremia
B. * Hemophilia A
C. Therapy with aspirin
D. von Willebrand's disease (vWD)
E. Idiopathic thrombocytopenic purpura (ITP)
192. Which of the following conditions is LEAST likely to be associated with a low serum 25(OH)
vitamin D level?
A. Dietary deficiency of vitamin D
B. Chronic severe cholestatic liver disease
C. Sedentary life-style
D. * Chronic renal failure
E. High-dose glucocorticoid therapy
193. Which of the following has the weakest association with Hepatocellular Carcinoma (HCC)
A. * oral contraceptives
B. Hepatitis C
C. Hepatitis B
D. Smoking
E. None of above
194. Which of the following hepatitis viruses have significant perinatal transmission
A. * Hepatitis B virus
B. Hepatitis C virus
C. Hepatitis E virus
D. Hepatitis A virus
E. None of above
195. Which of the following is a feature of nephrocalcionsis:
A. * Primary hyperparathyroidism
B. Medullary cystic kidney
C. Vitamin C intoxication
D. Pseudohypoparathyroidism
E. Primary hypothyroidism
196. Which of the following is False regarding H.Pylori infection
A. * With chronic infection urease breath test become negative
B. H.Pylori infection remain lifelong if untreated
C. Endoscopy is diagnostic
D. Toxigenic strains usually causes ulcer
E. None of above
197. Which of the following is not a congenital abnormality associated with Juvenile polyps
A. * Macrocephaly
B. Meckel's diverticulum
C. Malrotation
D. Mesenteric lymphangioma

E. None of above
198. Which of the following is NOT seen in Paroxysmal Nocturnal Hemoglobinuria:
A. Thrombosis
B. Hemosiderinuria
C. * Increased haptoglobin
D. Thrombocytopenia
E. Anemia
199. Which of the following is NOT the contraindication for sulfonilureas?
A. Diabetic with excessive weight
B. Gestation diabetes
C. Diabetic with progressive weight loss
D. * Type 2 DM, Non-proliferative rethinopathy
E. Pregnancy and lactation
200. Which of the following is not true about gastric lymphoma?
A. * MALT lymphoma is the commonest variety.
B. Peak incidence of lymphomas is seen in 6th-7th decade
C. Endoscopy usually reveals gastritis like picture or gastric ulcer.
D. Stomach is the most common organ in the gi system which is involved in Lymphoma
E. All of above
201. Which of the following is not true about Pneumatosis intestinalis of small bowel?
A. * Operative Procedures are required in most of the cases
B. Most common location is subserosa in the jejunum
C. It is seen equaly and males and females
D. It is associated with COPD and immunodeficiency states
E. None of above
202. Which of the following is not true for malignancy of Familial Adenomatous Polyposis
A. * Astrocytomas
B. Thyroid
C. Adrenals
D. Hepatoblastomas
E. None of above
203. Which of the following is not used in treatment of leukemia?
A. Steroid
B. Pentostatin
C. Splenectomy
D. Alpha-interferon
E. * Alendronic acid
204. Which of the following is the most common cause of death in Crohn's disease of small bowel
A. * Malignancy
B. Sepsis
C. Electrolyte Disorders
D. Thromboembolic Phenomenon
E. None of above
205. Which of the following is used in the treatment of the thyroid malignancy:
A. * I131
B. I125

C. 99Tc
D. P32
E. MIBG
206. Which of the following may be seen in multiple myeloma?
A. Increased Alkaline phosphatase
B. Decreased IgA
C. * Hypercalcemia
D. Hypouricemia
E. Fat bone marrow
207. Which of the following oral anti-diabetic drugs can be used in patients with renal failure:
A. Tolbutamide
B. Chlorpropamide
C. * Gliquidone
D. Glipizide
E. Metformin
208. Which of the following produces calcitonin?
A. A (alpha) cells
B. B (beta) cells
C. * C (parafollicular) cells
D. D (delta) cells
E. F cells
209. Which of the following statements about peptic ulcer disease is true :
A. * The incidence of complications has remained unchanged
B. Helicobacter pylori eradication increases the likelihood of occurrence of complications.
C. The incidence of Helicobacter pylori reinfection in India is very low.
D. Helicobacter pylori eradication does not alter the recurrence ratio.
E. None of above
210. Which of the following structures do not form the external part of anal canal mechanism?
A. * Internal Sphincter
B. Levator Ani
C. Pubo rectalis
D. External Sphincter
E. None of above
211. Which of the following structures does not form the portal triad
A. * Hepatic vein
B. Portal Vein
C. Hepatic Artery
D. Bile Duct
E. None of above
212. Which of these is true regarding CML?
A. * Size of splenomegaly indicates prognosis
B. Phagocytic activity of WBC is reduced
C. Sudan black stain is specific for myeloblast
D. Myeloblast, granuloblast and lymphoblast become PH chromosome +–ve following remission
E. Leukocytosis
213. Which one of the following is associated with increased bone resorbtion?

A. Estrogens
B. Calcitonin
C. Bisphosphonates
D. * Hyperparathyroidism
E. Hypothyroidism
214. Which patient is at greatest risk for developing a “community-acquired” pneumonia?
A. The 40-year-old first-grade teacher
B. * The 60-year-old smoker who is also an alcoholic
C. The 75-year-old with exercise-induced wheezing
D. The 35-year-old aerobics instructor who skips meals and eats only vegetables
E. None of above
215. Which signs are associated with tetany EXCEPT:
A. Chvostek’s sign
B. Trousseau's sign
C. Erb's sign
D. * All of them
E. None above them
216. Which statement is true in diabetes mellitus type 2?
A. Insulinitis of beta- cells
B. Hyalinization of beta- cells
C. Atrophy of beta cells
D. * Intact beta- cells
E. Degeneration of beta cells
217. Which test can you recommened as the most indicative of average recent blood glucose levels:
A. Fasting serum glucose level
B. Random serum glucose level
C. Oral glucose tolerance test
D. * Serum level of hemoglobin A1C
E. Urine glucose concentration
218. Which type diabetes is HLA associated:
A. * Type 1 diabetes
B. Type 2 diabetes
C. Tropic diabetes
D. Gestation diabetes
E. All of them
219. With which of the following theophylline has an antagonistic interaction?
A. Histamine receptors.
B. Bradykinin receptors.
C. * Adenosine receptors.
D. Imidazoline receptors.
E. Beta-adrenoreceptors
220. Zollinger-Ellison syndrome is characterized by all of the following except
A. * Massive HCL in response to histamine injection
B. Recurrent duodenal ulcer
C. Severe diarrhea
D. Post bulbar ulcer

E. None of above
221. A 64-year-old woman presents with a 1-year history of pain in her thumbs. A focused examination
reveals squaring at the base of both first digits, worse on the right, and pain on pressure over the
first carpal metacarpal joints. She also has non tender bony overgrowth at the distal interphalangeal
joints. The patient says that her mother had the same fingers and she worries that she will become
crippled. Choose the most likely diagnosis from the below list of options.
A. Ankylosing spondylitis
B. Rheumatoid arthritis
C. * Osteoarthritis
D. Gout
E. Psoriatic arthritis
222. A 66-year-old man has had a several-week history of fatigue and ankle swelling. His appetite is
normal, but his body weight has increased 4.5 kg (10 Ib). There is no history of exertional dyspnea,
paroxysmal nocturnal dyspnea, or orthopnea. He has had hypertension for 10 years, and diabetes
was diagnosed 4 months ago. He takes hydrochlorothiazide, 25 mg/d, and metoprolol, 50 mg/d. On
examination, the patient is alert and in no distress. Blood pressure is 120/75 mm Hg, pulse rate is
82/min, and temperature is 36.9 °C (98.4 °F). Skin examination is normal, and he has no
jugulovenous distention. Thyroid examination is normal. The lungs are clear. Cardiac examination
shows regular sinus rhythm, with no murmur or gallop. The abdomen is slightly protuberant with
shifting dullness, but no hepatosplenomegaly is noted. There is 4+ edema of the legs and thighs and
1+ edema of the sacrum. Laboratory studies: Complete blood count Normal Hemoglobin A1c
7.4%. Blood urea nitrogen - 20 mg/dL. Serum creatinine - 0.9 mg/dL. Serum sodium - 141 meq/L.
Serum chloride - 104 meq/L. Serum potassium - 4.3 meq/L. Serum bicarbonate - 30 meq/L. Serum
total protein - 4.4 g/dL. Serum albumin - 1.7 g/dL. Serum cholesterol - 376 mg/dL. 24-hour urine
protein 8.5 g. Urinalysis pH 5.5; specific gravity 1 .020; protein 4+, trace hemoglobin Urine
microscopy shows many granular and hyaline casts and oval fat bodies. Monoclonal protein is
identified on urine immunoelectrophoresis. Plasma immunoelectrophoresis shows an IgG level of
452 mg/dL, IgA of 284 mg/dL, 1gM of 122 mg/dL, K of 550 mg/dL, and 2 of 193 mg/dL with a
homogenous M band. What renal disease is most likely in this patient?
A. Idiopathic membranous glomerulopathy
B. Diabetic nephropathy
C. Focal and segmental glomerulosclerosis
D. Hypertensive nephropathy
E. * AL amyloidosis
223. A 68-year-old man is evaluated because of a 3-year history of bilateral knee pain and low back
pain. He has some stiffness for approximately 15 minutes when she awakens in the morning, and
during the afternoon her pain is worse. On physical examination, he has slight swelling and
tenderness to pressure of the distal interphalangeal joints 2-5 on both hands. There is slight crepitus
with motion of the right knee. Which of the following treatment do use?
A. Methotrexat
B. * Glucosamine
C. Colchicines
D. Tetracycline
E. Allopurinol

224. A 71-year-old man had non-Hodgkins lymphoma diagnosed 4 months ago. Treatment included
chemotherapy with rituximab and cyclosphosphamide, prednisone, and vincristine and
doxorubicin, followed by radiation therapy. He has had persistent anemia. He has had hypertension
for 15 years and prostatic hypertrophy, treated with doxazosin, for 10 years. He presents to the
emergency department with polyuria, weakness, and lassitude. On physical examination, blood
pressure is 124/78 mm Hg, with no orthostatic changes; pulse rate, 96/min; respiratory rate, 18/min;
and temperature 36.8 °C (98.2 °F). Mucous membranes are moist. There is no neck vein distention
or hepatojugular reflux. The cardiac examination is normal, and the chest is clear. The abdomen is
benign. No lower extremity edema is present. The electrocardiogram is normal. Hematocrit is 31%,
and leukocytosis is present, with a normal platelet count. The serum creatinine concentration,
previously normal, is now 2.4 mmol/L. Urinalysis shows a pH of 6.0 and 1 + proteinuria, but no
hematuria or ketonuria. No formed elements appear on microscopic examination. Urine sodium and
osmolality values have been requested. What is the most important next test to determine the reason
for decreased renal function in this patient?
A. Calculate the ratio of blood urea nitrogen to creatinine
B. * Perform renal ultrasonography
C. Perform renal scanning
D. Administer 1.5 L of normal saline as a fluid trial
E. Perform renal biopsy
225. A criterion for the diagnosis of asthma is:
A. * 15 % reversibility in spirography
B. 5% reversibility in spirography
C. 10% reversibility in spirography
D. X-ray changes
E. Sputum changes
226. A drug is to be delivered by a nebuliser. The size of a droplet for its humidification (in mcm) is:
A. * less than 5
B. 5-10
C. 10-15
D. 15-20
E. All of the above
227. A man aged 60 yrs has history of ischemic heart disease and atherosclerosis. He presents with
abdominal pain and maroon stools: likely diagnosis here is:
A. * Acute mesenteric ischemia
B. Acute intestinal obstruction
C. Peritonitis
D. Appendicitis
E. None of above
228. A patient was operated for colonic carcinoma and later a solitary nodule was found in the liver.
Treatment of choice is:
A. * Surgery
B. Radiation
C. Chemotherapy
D. Conservative treatment
E. None of above
229. A patient with BMI > 30 (kg/m2), serum glucose 24 mmol/L, urinary ketones ++++, he requires:
A. * Insulin
B. Glibenclamide
C. Metformin

D. Phenformin
E. Oral bicarbonate
230. A patient with BMI 34, serum glucose 26 mmol/l, urinary ketones 4+ requires:
A. * Insulin
B. Glibenclamide
C. Metformin
D. Glimepiride
E. Acarbose
231. A patient with DKA has deep sighing respiration. The main cause of Kussmaul breathing is:
A. Hyperglycemia
B. * Metabolic acidosis
C. Dehydration
D. Hypokalemia
E. Ketonemia
232. Above the cavity of abscess it is possible to hear such breathing
A. * Bronchial
B. Vesicular
C. Unclear
D. Hard
E. Weakened
233. Acid secretion phase, following food intake, defined by stimulation of mechanical and chemical
gastric wall receptors by luminal contents:
A. cephalic phase
B. * gastric phase
C. intestinal phase
D. All of the above
E. None of above
234. Activation of these/this receptor(s) on basolateral parietal cells inhibit(s) gastric acid secretion
A. * prostaglandins
B. gastrin
C. histamine
D. acetylcholine
E. all of the above
235. Acute abscess and gangrene of lungs belong to
A. * Pyogenic diseases of lungs
B. Chronic non-specific diseases of lungs
C. Congenital pathology of lungs
D. Obstructive diseases of lungs
E. Systemic diseases
236. Acute lymphocytic leukemia (ALL)
A. * A 4-year-old patient with pancytopenia and circulating blasts
B. A 60-year-old patient with pancytopenia and circulating blasts
C. A 20%-30% long-term survival
D. A patient with bleeding and infection
E. A patient with gum and skin infiltration
237. Acute myelogenous leukemia (AML)

A. A 4-year-old patient with pancytopenia and circulating blasts


B. * A 40-year-old patient with pancytopenia and circulating blasts
C. A 70%-80% long-term survival
D. A patient with bleeding and infection
E. A patient with gum and skin infiltration
238. Adenocarcinoma of esophagus is commonly found in :
A. * Barret's oesophagus
B. Achalasia acardia
C. Plummer vinson syndrome
D. Chronic smoking
E. None of above
239. After undergoing surgery, for Carcinoma of colon a patient developed single liver metastasis of
2cm. What you do next:
A. * Resection
B. Chemo radiation
C. Acetic acid injection
D. Radio frequency ablation
E. None of above
240. All are precancerous for carcinoma colon except
A. * carotene
B. Bile acids
C. Fats
D. crohn's disease
E. None of above
241. All are true about pseudopancreatic cyst of
A. * Most common site is in head of pancreas
B. Presents as an abdominal mass
C. Serum amylase is increased
D. Common after acute pancreatitis
E. None of above
242. All are true regarding Helicobacter pylori except:
A. * Less prevalent in developing countries
B. Toxicogenic strains usually causes ulcers
C. Urea breath test is positive
D. Gram negative organism
E. None of above
243. All are true regarding paroxysmal nocturnal haemoglobinuria, except:
A. Direct antiglobulin test (direct Coombs' test) is negative
B. Red urine
C. * Presence of antibodies
D. Reticulocytosis
E. Raised bilirubin
244. All are true regarding Zollinger Ellison syndrome, except:
A. * Decreased ratio of BAO to MAO
B. Recurrence after operation
C. Hypergastrinemia

D. Diarrhoea
E. None of above
245. All are used in treatment of Helicobactor pylori, EXCEPT:
A. * Cisapride
B. Colloid bismuth
C. Clarithromycin
D. Metronidazole
E. Lansoprazol
246. All clinical signs can be seen in ophthalmic Grave's disease EXCEPT:
A. Lid retraction
B. * Frequent blinking
C. Poor convergence
D. Upper lid "lad" on down gaze
E. Wide opened eyes
247. All medicines listed below are used in bronchial asthma, except:
A. * Morphine
B. Salbutamol
C. Aminophylline
D. Steroids
E. Ipratropium
248. All of the following are principles of treatment of DM, EXCEPT:
A. Achievement and maintenance of normal or reasonable body weight.
B. Normalization of metabolism and achievement of DM compensation.
C. * Education of the patients and self – control.
D. Maintenance (preservation) of working capacity.
E. Prophylaxis of acute and chronic complications.
249. All of the following are true regarding a patient with acid peptic disease except
A. * Misoprostol is drug of choice in pregnant patients
B. Duodenal ulcer is preventable by the use of single night time H2 blockers
C. Omeprazole may help ulcers refractory to H2 blockers
D. Misoprostol is the drug of choice in patients on NSAIDS
E. None of above
250. All of the following are used in the management of uncomplicated thyrotoxicosis EXCEPT:
A. Propylthiouracil
B. * Vasodilators
C. Radioactive iodine
D. Lugol's iodine
E. Propranolol
251. All of the following are useful for treating acute bronchial asthma except:
A. * Sodium chromoglycate inhalation
B. 100% Oxygen
C. Hydrocortisone infusion
D. IV aminophylline
E. All of the above
252. All of the following drugs are useful in the treatment of a patient with acute bronchial asthma
A. * except:
Montelukast

B. Ipratropium
C. Salbutamol
D. Hydrocortisone
E. All of the above
253. All of the following features are seen in the viral pneumonia except:
A. Presence of interstitial inflammation
B. * Predominance of alveolar exudates
C. Bronchiolitis
D. Multinucleate giant cells in the bronchiolar wall
E. All features are present
254. All of the following statements about Leukotriene modifiers in the management of bronchial
asthma are true except:
A. * May be used for acute asthma
B. May be used for exercise induced asthma
C. Zileuton is Leukotriene modifier
D. May uncover Churg Strauss syndrome
E. All of the above
255. All of the following statements about Pseudopancreatic cysts are true except:
A. * Percutaneous aspiration is treatment of choice
B. Cystojejunostomy is treatment of choice
C. Serum amylase levels are increased
D. Presents as an epigastric mass
E. None of above
256. All of the following statements are true about sickle cell disease except
A. Vaso-occlusive crisis
B. Aplastic Crisis
C. * Hypertensive crisis
D. Sequestration Crisis
E. Hyper hemolytic crisis
257. All of the following statements stand true for telangiectasia of colon except:
A. * 50% involve rectum
B. May be seen in person more than 60 years of age
C. Common site is caecum
D. May be seen in person less than 40 years of age
E. None of above
258. All of these signs may be present in patient with pituitary insufficiency, EXCEPT:
A. Hypotension.
B. * Hyperpigmentation.
C. Weight loss.
D. Hypogonadism.
E. Hypothyroidism.
259. All signs are associated with tetany EXCEPT:
A. Chvostek’s sign
B. Trousseau's sign
C. Erb's sign
D. * Cole's sign

E. None above them


260. All statements are true in hyperparathyroidism EXCEPT:
A. May cause hypercalcemia
B. * Commonly occurs after thyroidectomy
C. Solitary adenoma in parathyroid is a common cause
D. Nephrolithiasis is common
E. Frequent fractures
261. All the following are causes of Acute Pancreatitis except:
A. * Hemochromatosis
B. Alcohol
C. Gall stones
D. Hypercalcemia
E. None of above
262. All the following can be indications for parathyroidectomy in patients with hyperparathyroidism
EXCEPT
A. * Advanced age
B. Kidney stones
C. Osteoporosis
D. Calcium level &amp;amp;gt;2.9 mmol/L
E. Decreased creatinine clearance
263. All the following indicates early gastric cancer except
A. * Involvement of mucosa, submucosa and muscularis
B. Involvement of mucosa and submucosa
C. Involvement of mucosa
D. Involvement of mucosa, submucosa and adjacent lymph nodes
E. None of above
264. All the following may be the causes of hemorrhagic pleural effusion except:
A. * Congestive heart failure
B. Pulmonary embolism
C. Lung cancer
D. Tuberculosis
E. Severe chest trauma
265. Among the following, the most common site for Leiomyoma is
A. * Stomach
B. Small Intestine
C. Duodenum
D. Colon
E. None of above
266. Antacid: effective, but associated with systemic alkalosis
A. * sodium bicarbonate
B. calcium carbonate
C. magnesium hydroxide
D. aluminum hydroxide
E. All of above
267. Antacid: loose stools
A. * magnesium hydroxide

B. aluminum hydroxide
C. All of above
D. None of above
E. sodium bicarbonate
268. Antacid: most likely to cause systemic phosphate depletion:
A. * aluminum hydroxide
B. calcium carbonate
C. magnesium hydroxide
D. sodium bicarbonate
E. All of above
269. Antacid(s)
A. * All of above
B. magnesium hydroxide
C. calcium carbonate
D. aluminum hydroxide
E. None of above
270. Aplastic anemia can be caused by all except:
A. * ampicillini
B. chloramphenicol
C. quinine
D. phenytoin
E. carbamazepine
271. Aspirin sensitive asthma is associated with:
A. * Nasal polyps
B. Extrinsic asthma
C. Urticaria
D. Obesity
E. All of the above
272. Attack of bronchial asthma differs from heart asthma by such sign:
A. * By expiration dyspnea
B. By sudden development of attack
C. By orthostatic position of the patient
D. By duration of the attack
E. By inspiration dyspnea
273. Barrett's oesophagus is:
A. * Lower oesophagus lined by columnar epithelium
B. Upper oesophagus lined by columnar epithelium
C. Lower oesophagus lined by ciliated epithelium
D. Lower oesophagus lined by pseudostretifide epithelium
E. None of above
274. Basal acid secretion is inhibited by:
A. * all of the above
B. cimetidine (Tagamet)
C. nizatidine (Axid)
D. ranitidine (Zantac)
E. None of above

275. Basal acid secretion is stimulated by:


A. * histamine
B. cimetidine (Tagamet)
C. nizatidine (Axid)
D. ranitidine (Zantac)
E. famotidine (Pepcid)
276. Basolateral parietal cell membranes contained these receptor types:
A. * All of above
B. acetylcholine
C. prostaglandins
D. histamine
E. gastrin
277. Bence Jones protein may be found in the urine in
A. Chronic lymphocytic leukemia
B. Waldenstroms macroglobulinemia
C. Rheumatic fever
D. * Multiple myeloma
E. AML
278. Best antithyroid drug to be used to pregnancy is:
A. Carbimazole
B. Thiamazole
C. * Propylthiouracil
D. Radiocative I
E. Non of above
279. Best method for detecting of the minimal bronchiectasis is:
A. * CT scan
B. Bronchogram
C. Radionuclide lung scan
D. Chest X-ray
E. Nothing of the above
280. Best position to reveal the small pleural effusions on chest X-ray is
A. * Lateral decubitus view
B. AP view
C. PA view
D. Lateral view
E. All of the above
281. Best test for Small intestine malabsorption of carbohydrates is :
A. * D-Xylose test
B. Shilling test
C. Lund meal test
D. Follacin test
E. None of above
282. Bilateral malignant pleural effusion is most often seen in:
A. * Cancer of lung
B. Cancer of breast
C. Mesothelioma

D. Lymphoma
E. Nothing of the above
283. Bilateral pleural effusion is seen in:
A. Congestive cardiac failure
B. Nephritic syndrome
C. Constrictive pericarditis
D. * All of the above
E. Nothing of the above
284. Bilateral subtotal of thyroidectomy for Graves’ disease most likely can cause hypoparathyroidism
as a result as:
A. Loss of thyrocalcitonin
B. Reduction in parathyroid stimulating hormone
C. Removal of the four parathyroids
D. * Infarction of the parathyroids
E. Secondary hypopituitarism
285. Blocking of these structures can worsen the asthma
A. * ?2 -adrenoreceptors
B. ?-adrenoreceptors
C. Mast cells
D. Neutrophils
E. Eosinophils
286. Bone marrow transplant is the treatment of choice in all, except:
A. Aplastic anemia
B. AML in 1st remission
C. ALL in second remission
D. * ITP
E. Lympoma
287. Bronchial asthma is associated with raised levels of
A. * Leukotrienes
B. Thromboxane
C. Antinuclear antibodies
D. Corticosteroids
E. All of the above
288. Bronchiectasis is the most common in such lung lobe:
A. * Left lower
B. Right middle
C. Right upper
D. Left upper
E. Nothing of the above
289. Causes of haemorrhagic pleural effusion are all except:
A. * Bronchial adenoma
B. Pulmonary infarction
C. Mesothelioma
D. Tuberculosis
E. Nothing of the above
290. CBC revealed a WBC of 72,000 and a differential that was 83% lymphocytes. What is the MOST
likely diagnosis?

A. Acute lymphocytic leukemia


B. * Chronic lymphocytic leukemia
C. Acute prolymphocytic leukemia
D. Chronic prolymphocytic leukaemia
E. Lymphoma
291. Characteristic(s) of gastric mucosal acid secretion:
A. * all of the above
B. from parietal cells founded in mucosal glands of fundusof the stomach
C. stimulated by muscarinic cholinergic system (parietal cell innervation)
D. oxidative dephosphorylation dependent
E. None of above
292. Charcot’s joint in diabetes is NOT seen in:
A. None of them
B. Knee
C. Hip
D. Tarsal
E. * Elbow
293. Chose correct answer. Zollinger-Ellison syndrome -
A. * gastrin-secreting islet cell tumor
B. it is associated with Helicobacter pylori
C. it is associated withLamblia
D. All of the above
E. None of above
294. Common sites for cushing ulcers include all of the following except:
A. * Distal duodenum
B. Stomach
C. Is1 part of duodenum
D. Esophagus
E. None of above
295. Common X-ray finding of Staphylococcal pneumonia:
A. * Bilateral hilar lymphadenopathy
B. Involvement of lower lobes
C. Dense homogenous consolidation
D. Visible air bronchogram
E. Nothing of the above
296. Commonest site of peptic ulcer is:
A. * 1st part of Duodenum
B. Middle part of duodenum
C. Distal 1/3 of stomach
D. Pylorus of the stomach
E. None of above
297. Consider the following: Serum iron (1), MCV (2), serum total iron binding capacity (3), serum
ferritin (4). Which of these decreased in iron deficiency anaemia?
A. 1 and 2
B. 1, 3 and 4
C. * 1, 2 and 4

D. 1, 2, 3 and 4
E. 2,3 and 4
298. Constipation is caused by
A. * aluminum hydroxide
B. magnesium hydroxide
C. All of above
D. None of above
E. sodium bicarbonate
299. Contraindication for glucocorticosteroid therapy of bronchial asthma is:
A. * Bleeding from gastric ulcer
B. Chronic persistant hepatitis
C. Exacerbation of respiratory-heart failure
D. Emphysema of lungs
E. Respiratory failure
300. Curschmann's spirals in sputum are seen in:
A. * Bronchial asthma
B. Bronchiectasis
C. Chronic bronchitis
D. Wegenersgranulomatosis
E. All of the above
301. Dawn phenomenon refers to:
A. Early morning hyperglycemia followed by hypoglycemia
B. Hypoglycaemia followed by hyperglycemia
C. Early morning hypoglycemia
D. * Early morning hyperglycemia
E. High insulin level
302. Deaths from lactic acidosis in diabetes mellitus are associated with therapy:
A. Glibenclamide
B. Gliquidone
C. Chlorpropamide
D. * Phenformin
E. Glipizide
303. Decreased maximum mid-expiratory flow rate indicates obstruction in:
A. * Small airways
B. Trachea
C. Large airways
D. Trachea &amp;amp;amp; bronchi both
E. Nothing of the above
304. Diabetic ketoacidosis characterized by:
A. Low serum potassium
B. * Increased anion gap
C. Metabolic alkalosis
D. Respiratory acidosis
E. Hypoglycemia
305. Diagnostic tests for H. Pylori include all of the following except:
A. * Nechyporenco test

B. Rapid urease test


C. Gastric biopsy &amp;amp;amp; Warthin-starry stain
D. Urea-breath test
E. None of above
306. Drug of choice for inhalation in acute asthma not responding to salbutamol is:
A. * Ipratropium bromide
B. Atropine sulfate
C. Isopropamide
D. Hyoscine methyl bromide
E. All of the above
307. Drug of choice in asthma with heart disease is
A. * Ipratropium
B. Rimiterol
C. Terbutaline
D. Cromolyn sodium
E. All of the above
308. Drug of choice in Zollinger Ellison syndrome is
A. * Omeprazole
B. Ranitidine
C. Antacids
D. B-blocker
E. Atropin
309. Drug(s) used in eradication of Helicobacter pylori infection:
A. * All of above
B. metronidazole (Flagyl)
C. clarithromycin (Biaxin)
D. bismuth compounds
E. amoxicillin (Amoxil Polymox)
310. During an epidemic of hepatitis E, Fatality is maximum in :
A. * Pregnant women
B. Infants
C. Malnourished male
D. Adolescents
E. None of above
311. Early gastric cancer generally indicates
A. * Gastric adenocarcinoma confined to the mucosa &amp;amp;amp; submucosa
B. Gastric adenocarcinoma confined to the mucosa
C. Gastric adencarcinoma detected early
D. Gastric adenocarcinoma less than 1 cm In size
E. None of above
312. Endoscopic biopsy from a case of H. pylori related duodenal ulcer is most likely to reveal
A. * Antral predominant gastritis
B. Multifocal atrophic gastritis
C. Acute erosive gastritis
D. Gastric atrophy
E. None of above
313. Epidemiological studies of H. pylori are done by using:
A. * Serological markers
B. Urea-breath test
C. Culture
D. Gastric-biopsy urease test
E. None of above
314. Eradication of H. pylori effectiveness for bismuth compounds when using combination with
antibiotics:
A. * 95%
B. 20%
C. 50%
D. 80%
E. 5%
315. Example(s) of "aggressive factors" in peptic ulcer disease:
A. * pepsin
B. bicarbonate
C. prostaglandins
D. neither
E. All of above
316. Example(s) of "agressive factor(s)" in peptic ulcer disease:
A. * Helicobacter pylori
B. prostaglandins
C. gastric mucus
D. bicarbonate
E. all of the above
317. Example(s) of "protective factor(s)" in peptic ulcer disease:
A. * all of the above
B. prostaglandins
C. gastric mucus
D. bicarbonate
E. None of above
318. Examples of H2 (histamine receptor Type II) antagonists:
A. * All of above
B. ranitidine (Zantac)
C. nizatidine (Axid)
D. famotidine (Pepcid)
E. cimetidine (Tagamet)
319. Exercise induced asthma is not precipitated by :
A. * Swimming in hot water
B. High altitude climb and exercises
C. Cycling in cold weather
D. Swimming in cold water
E. All of the above
320. FEV1/FVC is reduced in case of:
A. * Asthma
B. Pleural effusion
C. Lung fibrosis
D. All of the above
E. Nothing of the above
321. Findings in hemophilia A include:
A. * Partial thromboplastin time increased
B. Increased clotting time
C. Increased prothrombin time
D. Bleeding time increased
E. Thrombocytopenia
322. Following are causes of unconjugated hyperbilirubinemia, except:
A. * Rotor syndrome
B. Large hematoma
C. Hemolytic anemia
D. Megaloblastic anemia
E. Irondeficiency anemia
323. For the patient who has sustained tracheobronchial trauma, which assessment finding alerts the
physician to the possibility of tracheal lacerations?
A. Hypertympanic sound on affected side
B. * Subcutaneous emphysema over the trachea
C. Hypotension and decreased capillary refill
D. Deviation of the trachea to the affected side
E. Non of above
324. Haemophilia A is characterised by the presence of following features, except
A. Bleeding into soft tissues
B. Reduced VIII levels
C. * Prolonged bleeding time
D. Prolonged partial thromboplastin time
E. Bleeding into muscles &amp;amp;amp; joints
325. Helicobacter pylori is not associated with :
A. * Gastric leiomyoma
B. Gastric cancer
C. Peptic ulcer of stomach
D. Peptic ulcer of duodenum
E. None of above
326. Histamine and gastric acid secretion:
A. * all of the above
B. release enhanced by increased cholinergic activity
C. most important gastric acid secretion stimulant
D. released from enterochromaffin-like cells
E. None of above
327. How long is the usual course of drug treatment for a patient with active tuberculosis?
A. 7 to 10 days
B. 6 weeks
C. * 6 months
D. 2 years
E. Non of above

328. Hypercalcemia may be treated by:


A. IV pamidronate
B. IV fluids + furosemide
C. Glucocorticoids
D. None above them
E. * All of them
329. Hypergastrinemia with hypochlorhydria is seen in:
A. * Pernicious anemia
B. All of above
C. Zollinger Ellison Syndrome
D. Glucagonoma
E. None of above
330. Hyperosmolar hyperglycemic non-ketonic coma is characterized by such level of glycemia:
A. * 55 mmol/1
B. 20 mmol/1
C. 30 mmol/1
D. 5 mmol/1
E. 25 mmol/1
331. Hypoglycemic coma can be caused by:
A. Insulin deficiency
B. Dehydration
C. * Extreme activity
D. Metabolic acidosis
E. Hypokalemia
332. Hypoglycemic effect of intermediate acting insulin lasts for:
A. * 12-16 hours
B. 20 -30 hours
C. 0.5 – 1.0 hours
D. 4 – 6 hours
E. 6 – 8 hours
333. Hypoglycemic unawareness that occurs in diabetic patients when transferred from oral
hypoglycemic agents to insulin, is due to :
A. * Autonomic neuropathy
B. Insulin resistance
C. Allergy
D. Somogi phenomenon
E. Down phenomenon
334. In a case of hypertrophic pyloric stenosis, the metabolic disturbance is
A. * Metabolic alkalosis with paradoxical aciduria
B. Metabolic acidosis
C. Respiratory alkalosis
D. Metabolic alkalosis with alkaline urine
E. None of above
335. In a patient only Anti HBsAg is positive in serum, all other viral markers are negative. It indicates :
A. * Immunized person with hepatitis B vaccine
B. Chronic active hepatitis

C. Persistent carrier
D. Acute hepatitis
E. None of above
336. In bronchial asthma glucocorticoids
A. * Reduce airway inflammation
B. Act as potent bronchodilators
C. Inhibit degranulation of mast cells
D. Block the action of humoral mediators
E. Nothing of the above
337. In endemic goiter such statement will be true:
A. Investigations invariably show some degree of hypothyroidism
B. There is a clear evidence of an increased incidence of carcinoma of the thyroid
C. Administration of thyroxine will often cause the goitre to shrink
D. Administration of iodine occasionally precipitates hyperthyroidism
E. * All of them
338. In gastric outlet obstruction in a peptic ulcer patient, the site of obstruction is most likely to be
A. * Duodenum
B. Antrum
C. Pylorus
D. Pyloric canal
E. None of above
339. In patients with nonketotic hyperosmolar coma the level of glycemia is :
A. Slightly elevated
B. Mildly elevated
C. Moderately elevated
D. May be decreased
E. * Grossly elevated
340. In pneumonia due to Mycoplasma all of the following are true except:
A. * Bilateral infiltration on chest X-ray
B. Myalgia
C. Dry cough
D. Pleuritic chest pain
E. All of the above
341. In restrictive lung disease :
A. * FEV1/FVC is high
B. FVC is high
C. FEV1 is high
D. All of the above
E. Nothing of the above
342. In severe bronchial asthma, it is present:
A. * Hyperresonant chest with prolonged expiration
B. Infrequent rhonchi and absent breath sounds
C. Increased fremitus and crackles
D. Decreased fremitus and crepitation
E. All of the above
343. In the time of attack of bronchial asthma in lungs are heard

A. * Dry loud wheezes


B. Moist loud wheezes
C. Moist soundless wheezes
D. Voice of pleural friction
E. Crepitation
344. In what specific way does the respiratory system function contribute to acid-base balance?
A. Prevents excessive loss of hydrogen ions by evaporation
B. Increases the potassium ion content in bronchial secretions
C. * Removes carbon dioxide generated as a result of metabolism
D. Maintains body water levels through mucous membrane function
E. Non of above
345. In what specific way does the respiratory system function contribute to acid-base balance?
A. Prevents excessive loss of hydrogen ions by evaporation
B. Increases the potassium ion content in bronchial secretions
C. * Removes carbon dioxide generated as a result of metabolism
D. Maintains body water levels through mucous membrane function
E. Non of above
346. In which case the dose of insulin in stable diabetics can be reduced:
A. Thyrotoxicosis
B. Steroid therapy
C. Pregnancy
D. * Chronic renal failure
E. None above them
347. Increased amylase may be seen in all of the following except:
A. * Appendicitis
B. Pancreatic pseudocyst
C. Perforated peptic ulcer
D. Ruptured ectopic pregnancy
E. None of above
348. Increased incidence of duodenal ulcer associated with:
A. * All of above
B. alcoholic cirrhosis
C. renal transplantation
D. systemic mastocytosis
E. None of above
349. Increased incidence of duodenal ulcer associated with:
A. * All of above
B. None of above
C. liver cirrhosis
D. chronic renal failure
E. hyperparathyroidism
350. Increased RBC level may be seen in:
A. Glomerulonephritis
B. Vitamin D excess
C. * Cor pulmonale
D. Acute heart failure

E. Pneumonia
351. Insulin resistance is associated with all of the following status, EXCEPT:
A. Acanthosis nigricans
B. Lipodystrophy
C. * Gout
D. Obesity
E. Metabolic syndrome
352. Intestinal hypomotility is seen in all the following except:
A. * All of the above
B. Parkinsonism
C. Amyloidosis
D. Diabetes
E. None of above
353. Intrinsic factor for absorption of vitamin Bl2 is secreted by the :
A. * Parietal cells
B. Peptic cells
C. Chief cells
D. Goblet cells
E. Endothelial cells
354. It is not Antacid
A. * Histamine
B. magnesium hydroxide
C. calcium carbonate
D. sodium bicarbonate
E. All of above
355. Location(s) of histamine in gastric mucosa:
A. * enterochromaffin-like cells (ECL) + mast cell cytoplasmic granules
B. mast cell cytoplasmic granules
C. enterochromaffin-like cells (ECL)
D. neither
E. are not localized in gastric mucosa
356. Macrophages containing large quantities of undigested and partial digested bacteria in intestine are
seen in
A. * Whipple's disease
B. Amyloidosis
C. Immunoapproliferative small instetinal disease
D. Vibro cholerae infection
E. None of above
357. Massive bleeding per rectum in a 70 yr old patient is due to :
A. * Diverticulosis
B. Carcinoma colon
C. Colitis
D. Polyps
E. None of above
358. Mechanism of action: cytoprotection, binds to the ulcer base, antibacterial effect:
A. * bismuth compounds

B. tetracycline (Achromycin)
C. metronidazole (Flagyl)
D. ranitidine (Zantac)
E. omeprazole (Prilosec)
359. Mechanism of Idiopathic thrombocytopenic purpura is:
A. Vasculitis
B. Antibodies to vascular epithelium
C. * Antibodies to platelets
D. Antibody to clotting factors
E. Fungal infectious
360. Mechanism(s) by which somatostatin reduces gastrin release:
A. * inhibits parietal cells secretion+ inhibits histamine release by enterochromaffin-like cells
B. inhibits histamine release by enterochromaffin-like cells
C. inhibits parietal cells secretion
D. stimulates parietal cells secretion
E. None of above
361. Melphalan is used in:
A. Wilm's tumor
B. Lymphoma
C. * Multiple myeloma
D. Nephroblastoma
E. ALL
362. Metabolic changes associated with excessive vomiting includes the following:
A. * Hypokalemia
B. Hyperchloremia
C. Metabolic acidosis
D. Decreases bicarbonates
E. None of above
363. Microcytic anemia is seen in:
A. Sprue
B. Pernicious anemia
C. Latun infestation
D. * Iron deficiency anemia
E. Aplastic anemia
364. Monotherapy effectiveness in eradication of H. pylori-- bismuth compounds
A. * 20%
B. 5%
C. 50%
D. 90%
E. 100%
365. More likely to affect the cytochrome P450 drug metabolizing system:
A. * cimetidine (Tagamet)
B. All of above
C. nizatidine (Axid)
D. ranitidine (Zantac)
E. famotidine (Pepcid)

366. Most common CNS manifestation of Whipple's disease is :


A. * Dementia
B. Supranuclear ophthalmoplegia
C. Seizure
D. Cerebellar ataxia
E. None of above
367. Most common complication of acute pancreatitis is:
A. * Pseudocyst
B. Pancreatic abscess
C. Phlegmon
D. Pleural effusion
E. None of above
368. Most common extraintestinal manifestation of Crohn's disease of small bowel is.
A. * Erythema Nodosum
B. Ankylosing Spondylitis
C. Iritis
D. Ureteral Obstruction
E. None of above
369. Most common sign of aspiration pneumonitis:
A. * Tachypnoe
B. Bronchospasm
C. Cyanosis
D. Crepitations
E. Nothing of the above
370. Most sensitive and specific test for diagnosis of iron deficiency is:
A. * Serum ferritin levels
B. Serum iron levels
C. Serum transferrin receptor population
D. Transferrin saturation
E. Hb, Ht
371. Most successful protocol for eradication of H. pylori/treatment of peptic ulcer disease:
A. * therapy using bismuth compounds, Clarythromycin (Flagyl), and amoxicillin (Amoxil Polymox)
B. bismuth monotherapy
C. bismuth compounds and amoxicillin (Amoxil Polymox)
D. all of the above equally effective
E. None of above
372. Most widely used antacid in treating ulcer disease:
A. * combination of aluminum hydroxide and magnesium hydroxide
B. calcium carbonate
C. magnesium hydroxide
D. aluminum hydroxide
E. sodium bicarbonate
373. Mycoplasma infection simulates:
A. * Viral pneumonia
B. Pnemuococcal pneumonia
C. Pulmonary oedema

D. Pulmonary infarction
E. Nothing of the above
374. Non-parenteral hepatitis is :
A. * Hepatitis E
B. Hep B
C. Hep C
D. Hep D
E. All of above
375. Omeprazole (Prilosec) and lansoprazole approved for clinical treatment of:
A. * All of above
B. erosive gastritis
C. Zollinger-Ellison syndrome and other gastric acid hypersecretory states
D. None of above
E. duodenal ulcer
376. Only class of antiulcer drugs that can eradicate Helicobacter pylori and cure associated gastritis:
A. * colloidal bismuth
B. sucralfate (Carafate)
C. H2 blockers
D. Anticholinergic agents
E. None of above
377. Oral anticoagulant therapy is monitored with indexes
A. * INR
B. PTT
C. TT
D. Clot lysis time
E. Bleeding time
378. Pancreatic pseudocyst most commonly occurs after
A. * Pancreatitis
B. Trauma
C. Pancreatic surgery
D. Pancreatic malignancy
E. None of above
379. Paroxysmal nocturnal haemoglobinuria is due to:
A. Antibodies
B. Congenital membrane defect
C. Inherited autoimmune disorder
D. * Complement mediated RBC lysis
E. Lysis of RBC in spleen
380. Pathogenic factor(s) in duodenal ulcer:
A. * all of the above
B. genetic factor
C. cigarette smoking
D. alcoholic cirrhosis
E. COPD (chronic obstructive pulmonary disease)
381. Patient D. presents elevated serum T4 and increased radioactive iodine uptake. Put diagnosis,
A. please.
Non-toxic goiter

B. Hashimoto's thyroiditis
C. Subacute thyroiditis
D. Hypothyroidism
E. * Graves' disease
382. Patient F., 29 yr. old, presents with subacute thyroiditis. Treatment will include:
A. Antibiotics
B. Vitamins
C. Antithyroid drugs
D. Iodine preparations
E. * Corticosteroids
383. Patient on heparin therapy should be monitored with:
A. * Clotting time
B. Bleeding time
C. Prothrombin time
D. Fibrinogen
E. PTT
384. Patient presents with recurrent duodenal ulcer of 2.5 cm size; procedure of choice
A. * Truncal vagotomy and antrectomy
B. Truncal vagotomy and gastrojejunostomy
C. Highly selective vagotomy
D. Laparoscopic vagotomy and gastrojejunostomy
E. None of above
385. Peptic ulcer classification:
A. * duodenal+ gastric
B. gastric
C. duodenal
D. neither
E. cardiac
386. Peptic ulcer should be differentiated with
A. * All of above
B. Cholecystitis
C. Gastritis
D. Miocardial infarction
E. Pancreatitis
387. Pernicious anemia is seen in:
A. Iron deficiency
B. Chronic liver disease
C. Bleeding
D. * Atrophic gastritis
E. Hemolysis
388. Platelet count is decreased in all of the following condition except:
A. Idiopathic thrombocytopenic purpura
B. Thrombotic thrombocytopenic purpura
C. Systemic lupus erythematosus
D. * Haemophilia
E. Hemolytic uremic syndrom

389. Pneumothorax is a possible complication of


A. * Staphylococcal pneumonia
B. Pneumococcal pneumonia
C. Klebsiella pneumonia
D. Viral pneumonia
E. All of the above
390. Presentation of hypothyroidism includes:
A. * Menorrhagia
B. Oligomenorrhoea
C. Amenorrhea
D. All of them
E. None above them
391. Primary atypical pneumonia is caused by:
A. * Mycoplasma
B. Mycobateriumkansasii
C. Str. pneumoniae
D. Pneumocystis carinii
E. Nothing of the above
392. Primary hyperparathyroidism most likely caused by:
A. Multiple parathyroid adenomas
B. * Solitary parathyroid adenoma
C. Adrenal hyperplasia
D. Ectopic PTH production
E. None above them
393. Pseudopolyps are features of
A. * Ulcerative colitis
B. Crohn's disease
C. Celiac sprue
D. Whipple's disease
E. None of above
394. pylori is known to cause all of the following except:
A. * Fundal atrophic gastritis
B. Duodenal ulcer
C. Gastritis type B
D. Gastric ulcer
E. None of above
395. Raised serum amylase levels are used to diagnose
A. * Acute pancreatitis
B. Degenerative diseases
C. Acute cholecystitis
D. Autoimmune disease
E. None of above
396. Ranitidine (Zantac) and gastric acid secretion:
A. * inhibits basal acid secretion+ inhibits secretion in response to vagal stimulation or feeding
B. inhibits secretion in response to vagal stimulation or feeding
C. inhibits basal acid secretion

D. neither
E. stimulates basal acid secretion
397. Relapse rate for duodenal ulcer following H. pylori eradication:
A. * 15%
B. 75%
C. 50%
D. 90%
E. 5%
398. Reserve transcriptase of hepatitis B virus is coded on the following gene :
A. * P gene
B. S gene
C. C gene
D. X gene
E. All of above
399. Risk factor for development of gastric Carcinoma
A. * Intestinal metaplasia type III
B. Duodenal ulcer
C. Intestinal hyperplasia
D. Blood group O
E. None of above
400. Screening method for medullary carcinoma thyroid is:
A. * Serum calcitonin
B. Serum calcium
C. Serum alkaline phosphate
D. Serum acid phosphatase
E. None above them
401. Secondary hyperparathyroidism is associated with:
A. Parathyroid adenoma
B. Marked hypercalcaemia
C. * Chronic renal failure
D. Parathyroidectomy
E. All of them
402. Serum unconjugated bilirubin and urine urobilinogen concentration are elevated in
A. anemias of chronic disease
B. * hemolytic anemias
C. anemia of acute hemorrhage
D. aplastic anemias
E. iron deficiency anemia
403. Sharko-Leiden crystals are seen in:
A. * Bronchial asthma
B. Bronchiectasis
C. Chronic bronchitis
D. Wegenersgranulomatosis
E. All of the above
404. Side effect of antiulcer medication: urinary retention, blurred vision, xerostromia:
A. * atropine

B. ranitidine (Zantac)
C. nifedipine (Procardia, Adalat)
D. sucralfate (Carafate)
E. None of above
405. Significant high ESR is seen in :
A. Polycythemia vera
B. CHF
C. * Multiple myeloma
D. Sickle cell anemia
E. CLL
406. Skip granulomatous lesions are seen in
A. * Crohn's disease
B. Ulcerative colitis
C. Whipple's disease
D. Reiter's disease
E. None of above
407. Small capillaries bleeds in:
A. * Thrombocytopenia
B. Coagulation failure deficiency
C. Anemia
D. Agranulocytosis
E. Hemophilia
408. Somogi phenomenon characterized by :
A. * Early morning hyperglycemia
B. Early morning hypoglycemia
C. Hypoglycemia followed by hyperglycemia
D. High insulin levels
E. Night hyperglycemia
409. Spherocytosis is best diagnosed by:
A. Splenic puncture
B. BM aspiration
C. Plasma
D. * Peripheral blood smear
E. Phenotyping
410. Such radiological findings as "hour-glass" vertebrae and "triturated pelvis" are seen in:
A. Thyrotoxicosis
B. Myxedema
C. Cretinism
D. * Hyperparathyroidism
E. Hypothyroidism
411. Such results of investigations as positive Chvostek sign and elevated serum parathyroid hormone
(PTH) level are associated with:
A. * Pseudohypoparathyroidism
B. Primary hypoparathyroidism
C. Primary hyperparathyroidism
D. Osteoporosis

E. Hypercalcemia of malignancy
412. The actions of insulin includes which one of the following?
A. * Increased utilization of glucose by muscle
B. Increased hepatic output of glucose
C. Increased lipolysis
D. Increased amino acid flux from muscle due to protein breakdown
E. Activated glycogenolysis
413. The adenocarcinoma of esophagus-developed in
A. * Barret's esophagus
B. Long standing achalasia
C. Corrosive structure
D. Alcohol abuse
E. None of above
414. The commonest acute presentation of sickle cell anaemia is:
A. Priapism
B. * Bone pain
C. Fever
D. Splenomegaly
E. Pulmonary hypertension
415. The commonest hepatotropic virus progressing to chronicity is:
A. * HCV
B. HAV
C. HBV
D. HEV
E. HAV+ HEV
416. The first virological marker following acute infection with HBV is :
A. * HBs Ag
B. Anti HBs Ag
C. IgM anti HBc Ag
D. Anti HBe Ag
E. All of above
417. The following laboratory index is abnormally prolonged in ITP:
A. APTT
B. * Bleeding time
C. Prothrombin time
D. Clotting time
E. None of above
418. The frequent causes of hypoglycemia in hospitalized diabetic patients include all the following
EXCEPT:
A. Insulin
B. Sulfonylureas
C. * Adrenal insufficiency
D. Alcohol
E. Renal failure
419. The high level of C-peptide can be found in a case of:
A. Glucagonoma

B. * Insulinoma
C. Gastrinoma
D. Hepatoma
E. In all of them
420. The histological features of coeliac disease include all of the following, except:
A. * Increase in thickness of the mucosa
B. Crypt hyperplasia
C. Increase in intraepithelial lymphocytes
D. Increase in inflammatory cells in lamina propyria
E. None of above
421. The lowest recurrence of peptic ulcer is associated with
A. * Vagotomy + Antrectomy
B. Vagotomy + drainage
C. Gastric resection
D. Highly selective vagotomy
E. Resection of stomach by Bilroth II
422. The main cause of hypercalcemic crisis is :
A. * Parathyroid adenoma
B. CA Breast
C. Parathyroid hyperplasia
D. Paget's disease
E. Hyperthyroidism
423. The main cause of hyperosmolar coma is:
A. Insulin deficiency
B. * Dehydration
C. Extreme activity
D. Hypoxia
E. Intercurrent infection
424. The main cause of lactic acidosis is
A. Insulin deficiency
B. Dehydration
C. Extreme activity
D. * Hypoxia
E. Intercurrent infection
425. The major sign of hypoventilation is
A. * Cyanosis
B. Hypoxia
C. Hypercapnia
D. Dyspnea
E. All of the above
426. The most common cause of aplastic anemia is:
A. * Idiopathic
B. Chloramphenicol
C. Phenylbutazone
D. Petroleum products
E. Prednisolone

427. The most common cause of thyroid crisis is:


A. * Thyroid surgery
B. 131I administration
C. Administration of Beta blockers
D. Thyrostatic therapy
E. All of the above
428. The most common cause of thyrotoxicosis is:
A. * Grave's disease
B. Follicular adenoma
C. Multinodular goiter
D. Subacute thyroiditis
E. Iatrogenic
429. The most common community acquired infection is :
A. * Streptococcal pneumonia
B. Hepatitis A
C. Cholera
D. Meningitis
E. Nothing of the above
430. The most common presentation of primary hyperparathyroidism is:
A. Bone fracture
B. Increased serum creatinine
C. Osteitis fibrosa cystica
D. * Asymptomatic hypercalcemia
E. Kidney stones
431. The most common route of spread of hepatitis E is
A. * Feco-oral
B. Blood transfusion
C. injection IV
D. perinatal transmission
E. All of above
432. The most effective correction of acidosis in diabetic acidosis is:
A. IV bicarbonate
B. IV Saline
C. Oral bicarbonate
D. * IV Insulin
E. IV potassium
433. The most important regulator of serum 1,25(OH)2 vitamin D concentration is
A. Serum calcium
B. Serum magnesium
C. Serum 25(OH) vitamin D
D. TSH
E. * Parathyroid hormone
434. The most pre-cancerous condition for carcinoma colon is
A. * Familial polyposis
B. Hamartomatous polyps
C. Juvenile polyps

D. Hyperplastic polyps
E. None of above
435. The patient is taking enalapril (Vasotec), an angiotensin-converting enzyme (ACE) inhibitor, for
hypertension. Which respiratory side effect should you teach the patient to expect?
A. Wheezing on exhalation
B. Increased nasal stuffiness
C. Chest pressure or pain
D. * Persistent dry cough
E. Non of above
436. The place of 25-hydroxycholecalciferol formation is:
A. * Liver
B. Kidney
C. Intestines
D. Pancreas
E. Brain
437. The principal role of the kidneys in the synthesis of the active form of vitamin D is:
A. Conversion of 7-dehydrocholesterol to vitamin D3
B. Absorption of vitamin D2
C. Conversion of vitamin D to 25-OH vitamin D
D. Absorption of vitamin D3
E. * Conversion of 25-OH vitamin D to 1,25-(OH)2 vitamin D
438. The results of the glucose tolerance test: a fasting serum glucose is 6,5 mmol/l, 2-hour postprandial
serum glucose is 14,6 mmol/l. What is your diagnosis?
A. Normal.
B. Impaired glucose tolerance.
C. Impaired fasting glycemia.
D. * Diabetes mellitus.
E. Neither.
439. The shortest period of the action is present in such group of preparations as:
A. Thiazolidinediones
B. Biguanides
C. Sulfonilureas
D. Acarbose
E. * Glinides
440. The triad originally described by Zollinger-Ellison syndrome is characterized by:
A. * Peptic ulceration, gastric hypersecretion, non beta cell tumour
B. Peptic ulceration, gastric hypersecretion, beta cell tumor
C. Peptic ulceration, achlorhydria, non beta cell tumour
D. Peptic ulceration, achlorhydria, beta cell tumour
E. None of above
441. The true statement of tertiary hyperparathyroidism is:
A. Primary hyperparathyroidism with decrease Ca+ level
B. * Secondary hyperparathyroidism with chief cell adenoma
C. Secondary hyperparathyroidism following intestinal malabsorption
D. Metastasis with normal phosphate level
E. Secondary hyperparathyroidism following chronic renal failure
442. Thrombocytopenia, neutropenia are seen in

A. anemias of chronic disease


B. hemolytic anemias
C. anemia of acute hemorrhage
D. * aplastic anemias
E. iron deficiency anemia
443. Thyroid carcinoma associated with hypocalcemia is
A. Follicular carcinoma
B. * Medullary carcinoma
C. Anaplastic carcinoma
D. Papillary carcinoma
E. All of them
444. To remove onset of tetany you will order
A. * Calcium chloride
B. Diuretics
C. Prednisolone
D. Potassium citrate
E. Magnesium sulphate
445. Toxic megacolon is most commonly associated with
A. * Ulcerative colitis
B. Crohn's disease
C. Whipple's disease
D. Reiter's disease
E. None of above
446. Treatment of choice for aplastic anemia is:
A. Methotrexat
B. ampicillini
C. chloramphenicol
D. * Bone marrow transplantation
E. Plasmaphoresis
447. Treatment of gastric ulcer disease
A. * atropine is less effective than H2 receptor blockers
B. atropine is more effective than H2 receptor blockers
C. atropine and H2 receptor blockers are equally effective
D. atropine and H2 receptor blockers are not effective
E. Omeprazol is not used
448. Treatment of neonatal idiopathic thrombocytopenia is
A. Azathioprin
B. Dexamethasone
C. * Immunoglobulins
D. Platelets transfusion
E. Splenectomy
449. True about idiopathic thrombocytopenic purpura is:
A. * Increase megakaryocytes in bone marrow
B. Frequent joint haemorrhage
C. Most common in children
D. Males commonly affected

E. VIII factor deficiency


450. True about Ulcerative Colitis with malignancy
A. * Is related to duration of ulcerative colitis
B. Is related to disease activity
C. It has a better prognosis
D. Malignancy is more in anorectal ulcerative colitis
E. None of above
451. True statement about Hypercalcemia :
A. Treatment of the primary cause is effective
B. Malignancy does not produce hypercalcemia
C. * i.v. fluid with Furosemide is given
D. Pamidronate is not effective.
E. i.v. fluid with calcium chloride is effective
452. What is a possible level of platelets in the patient with Acute Lymphoblastic Leukemia?
A. * 100.000 – 150.000/L
B. Over 200.000/L
C. 250.000– 300.000/L
D. Below 300.000/L
E. Absence of Pl
453. What is drug of choice for ulcerative colitis :
A. * 5-amino salicylic acid
B. Prednisolone
C. Mercaptopurine
D. Salazopyrine
E. None of above
454. What is not true about blind loop syndrome?
A. * Surgery is almost always required to correct small bowel syndrome
B. Megaloblastic anemia is commonly seen
C. It manifets as diarrhoea, weight loss and deficiency of fat soluble vitamins.
D. Broad spectrum antibiotics are the treatment of choice
E. None of above
455. What is not true about the immune mechanism in the small bowel?
A. * Ig A acts by activating the complement pathway
B. Intestine contains more than 70% of IgA producing cells in the body
C. Ig A is produced by plasma cells in the lamina propria
D. Approximately 60% of the lymphoid cells are T cells
E. None of above
456. What is the Acute Lymphoblastic Leukemia?
A. * Disorder of the blood-forming tissue (white cells)
B. Disorder of the blood-forming tissue (red cells)
C. Leukocytosis
D. Increasing of blood pressure
E. Decreasing of blood pressure
457. What is the best marker for prognosis of the multiple myeloma presents with bony lesions:
A. Serum calcium level
B. Beta 1a microglobulin

C. Beta 1 microglobulin
D. * Bone marrow plasma cell
E. Number of lytic lesions
458. What is the most necessary in order to diagnose bronchial asthma:
A. * To notice the asthmatic attack of dyspnea
B. To find allergic antibodies
C. To find the presence of expiration dyspnea
D. To find the presence of signs of obstructive respiratory failure
E. To find the presence of eosinophylia in blood
459. What is true of haemophilia:
A. * Increased PTT
B. Platelets level decreased
C. Increased factor VIII
D. Increased factor IX
E. BT decreased
460. What is true regarding congenital hypertrophic pyloric stenosis:
A. * Hypochloremic alkalosis
B. More common in girls
C. Hellers myotomy is the procedure of choice.
D. Most often manifests at birth
E. None of above
461. Which among the following diseases may be causative agent for B12 deficiency anaemia?
A. Chronic gastritis of type B
B. Peptic gastric ulcer
C. * Chronic gastritis of type A
D. Chronic pancreatitis
E. Chronic cholecystitis
462. Which amongst the following is the earliest indicator of hypothyroidism?
A. Decreased serum T4
B. Decreased serum T3
C. Increased relaxation phase of deep tendon reflexes
D. * High serum TSH
E. High serum TRH
463. Which changes of a tongue are typical for vitamin B12 deficiency anaemia?
A. * Geographic tongue
B. Red
C. Coated
D. Clean
E. Swollen
464. Which does not predispose to Carcinoma stomach?
A. * Low Nitrate consumption
B. Salted meat and fish
C. Low fat and protein diet
D. HIgh Complex carbohydrate consumption
E. All of above
465. Which drug is mostly used for the treatment of chronic lymphatic leukemia?

A. Prednisone
B. * Chlorambucil
C. Methotrexate
D. Myleran
E. Ampicilin
466. Which drug is not effective against H. pylori:
A. * Erythromycin
B. Metronidazole
C. Amoxicillin
D. Colloidal Bismuth
E. None of above
467. Which from the following symptoms are characteristic for anaemia?
A. Petechia
B. Cyanosys
C. * Pallor of skin and mucous membranes
D. Edema
E. Hyperpigmentation
468. Which index can differentiate factitious hyperinsulinemia from insulinoma:
A. * C – peptides
B. Insulin antibodies
C. Serum glucose levels
D. None
E. All of them
469. Which is not a feature of multiple myeloma?
A. * Cutaneous nodules
B. Bony lesions
C. Renal failure
D. 'M' Spike
E. Hypercalciemia
470. Which is not a paraneoplastic syndrome for Hepatocellular Carcinoma
A. * Hyperglycemia
B. Hypoglycemia
C. Erythrocytosis
D. Hypercalcemia
E. None of above
471. Which of the following can help to put a diagnosis of subclinical hypothyroidism for a patient?
A. Low thyroid hormone levels but no symptoms
B. Classic symptoms of hypothyroidism but normal thyroid function test results
C. Low free T4 levels but normal serum thyroid-stimulating hormone (TSH) level
D. Low serum TSH but normal free T4 levels
E. * Increased serum TSH but normal free T4 levels
472. Which of the following conditions causes delayed, deep tissue-type bleeding?
A. Uremia
B. * Hemophilia A
C. Therapy with aspirin
D. von Willebrand's disease (vWD)

E. Idiopathic thrombocytopenic purpura (ITP)


473. Which of the following conditions is LEAST likely to be associated with a low serum 25(OH)
vitamin D level?
A. Dietary deficiency of vitamin D
B. Chronic severe cholestatic liver disease
C. Sedentary life-style
D. * Chronic renal failure
E. High-dose glucocorticoid therapy
474. Which of the following has the weakest association with Hepatocellular Carcinoma (HCC)
A. * oral contraceptives
B. Hepatitis C
C. Hepatitis B
D. Smoking
E. None of above
475. Which of the following hepatitis viruses have significant perinatal transmission
A. * Hepatitis B virus
B. Hepatitis C virus
C. Hepatitis E virus
D. Hepatitis A virus
E. None of above
476. Which of the following is a feature of nephrocalcionsis:
A. * Primary hyperparathyroidism
B. Medullary cystic kidney
C. Vitamin C intoxication
D. Pseudohypoparathyroidism
E. Primary hypothyroidism
477. Which of the following is False regarding H.Pylori infection
A. * With chronic infection urease breath test become negative
B. H.Pylori infection remain lifelong if untreated
C. Endoscopy is diagnostic
D. Toxigenic strains usually causes ulcer
E. None of above
478. Which of the following is not a congenital abnormality associated with Juvenile polyps
A. * Macrocephaly
B. Meckel's diverticulum
C. Malrotation
D. Mesenteric lymphangioma
E. None of above
479. Which of the following is NOT seen in Paroxysmal Nocturnal Hemoglobinuria:
A. Thrombosis
B. Hemosiderinuria
C. * Increased haptoglobin
D. Thrombocytopenia
E. Anemia
480. Which of the following is NOT the contraindication for sulfonilureas?
A. Diabetic with excessive weight

B. Gestation diabetes
C. Diabetic with progressive weight loss
D. * Type 2 DM, Non-proliferative rethinopathy
E. Pregnancy and lactation
481. Which of the following is not true about gastric lymphoma?
A. * MALT lymphoma is the commonest variety.
B. Peak incidence of lymphomas is seen in 6th-7th decade
C. Endoscopy usually reveals gastritis like picture or gastric ulcer.
D. Stomach is the most common organ in the gi system which is involved in Lymphoma
E. All of above
482. Which of the following is not true about Pneumatosis intestinalis of small bowel?
A. * Operative Procedures are required in most of the cases
B. Most common location is subserosa in the jejunum
C. It is seen equaly and males and females
D. It is associated with COPD and immunodeficiency states
E. None of above
483. Which of the following is not true for malignancy of Familial Adenomatous Polyposis
A. * Astrocytomas
B. Thyroid
C. Adrenals
D. Hepatoblastomas
E. None of above
484. Which of the following is not used in treatment of leukemia?
A. Steroid
B. Pentostatin
C. Splenectomy
D. Alpha-interferon
E. * Alendronic acid
485. Which of the following is the most common cause of death in Crohn's disease of small bowel
A. * Malignancy
B. Sepsis
C. Electrolyte Disorders
D. Thromboembolic Phenomenon
E. None of above
486. Which of the following is used in the treatment of the thyroid malignancy:
A. * I131
B. I125
C. 99Tc
D. P32
E. MIBG
487. Which of the following may be seen in multiple myeloma?
A. Increased Alkaline phosphatase
B. Decreased IgA
C. * Hypercalcemia
D. Hypouricemia
E. Fat bone marrow

488. Which of the following oral anti-diabetic drugs can be used in patients with renal failure:
A. Tolbutamide
B. Chlorpropamide
C. * Gliquidone
D. Glipizide
E. Metformin
489. Which of the following produces calcitonin?
A. A (alpha) cells
B. B (beta) cells
C. * C (parafollicular) cells
D. D (delta) cells
E. F cells
490. Which of the following statements about peptic ulcer disease is true :
A. * The incidence of complications has remained unchanged
B. Helicobacter pylori eradication increases the likelihood of occurrence of complications.
C. The incidence of Helicobacter pylori reinfection in India is very low.
D. Helicobacter pylori eradication does not alter the recurrence ratio.
E. None of above
491. Which of the following structures do not form the external part of anal canal mechanism?
A. * Internal Sphincter
B. Levator Ani
C. Pubo rectalis
D. External Sphincter
E. None of above
492. Which of the following structures does not form the portal triad
A. * Hepatic vein
B. Portal Vein
C. Hepatic Artery
D. Bile Duct
E. None of above
493. Which of these is true regarding CML?
A. * Size of splenomegaly indicates prognosis
B. Phagocytic activity of WBC is reduced
C. Sudan black stain is specific for myeloblast
D. Myeloblast, granuloblast and lymphoblast become PH chromosome +–ve following remission
E. Leukocytosis
494. Which one of the following is associated with increased bone resorbtion?
A. Estrogens
B. Calcitonin
C. Bisphosphonates
D. * Hyperparathyroidism
E. Hypothyroidism
495. Which patient is at greatest risk for developing a “community-acquired” pneumonia?
A. The 40-year-old first-grade teacher
B. * The 60-year-old smoker who is also an alcoholic
C. The 75-year-old with exercise-induced wheezing

D. The 35-year-old aerobics instructor who skips meals and eats only vegetables
E. None of above
496. Which signs are associated with tetany EXCEPT:
A. Chvostek’s sign
B. Trousseau's sign
C. Erb's sign
D. * All of them
E. None above them
497. Which statement is true in diabetes mellitus type 2?
A. Insulinitis of beta- cells
B. Hyalinization of beta- cells
C. Atrophy of beta cells
D. * Intact beta- cells
E. Degeneration of beta cells
498. Which test can you recommened as the most indicative of average recent blood glucose levels:
A. Fasting serum glucose level
B. Random serum glucose level
C. Oral glucose tolerance test
D. * Serum level of hemoglobin A1C
E. Urine glucose concentration
499. Which type diabetes is HLA associated:
A. * Type 1 diabetes
B. Type 2 diabetes
C. Tropic diabetes
D. Gestation diabetes
E. All of them
500. With which of the following theophylline has an antagonistic interaction?
A. Histamine receptors.
B. Bradykinin receptors.
C. * Adenosine receptors.
D. Imidazoline receptors.
E. Beta-adrenoreceptors
501. Zollinger-Ellison syndrome is characterized by all of the following except
A. * Massive HCL in response to histamine injection
B. Recurrent duodenal ulcer
C. Severe diarrhea
D. Post bulbar ulcer
E. None of above
Назва наукового напрямку (модуля): Семестр: 12
Внутрішня медицина 6 курс
Опис:
СРС
Перелік питань:
1. A 67-year-old man complains of dyspnea on exertion, attacks of retrosternal pain, dizziness. He has
no history of rheumatism. Objectively: pale skin, acrocyanosis. There are crackles in the lower
lungs. There is systolic thrill in the II intercostal space on the right, coarse systolic murmur
conducted to the vessels of neck. BP- 130/90 mm Hg, heart rate - 90/min., regular rhythm. The
liver extends 5 cm from under the edge of costal arch, shin edemas are present. Specify the
A. * suspected valvular defect:
Aortic stenosis
B. Pulmonary artery stenosis
C. Mitral insufficiency
D. Ventricular septal defect
E. Tricuspid regurgitation
2. A 70-year-old patient consulted a doctor about arrhythmic cardiac activity, dyspnea. Objectively:
BP - 150/90 mm Hg, extrasystole arrhythmia (10-12 beats per minute), left ventricular systolic
dysfunction (ejection fraction at the rate of 42%). Which of antiarrhythmic drugs should be
administered as initial therapy in this case?
A. * Amiodarone
B. Flecainide
C. Encainide
D. Moracizine
E. Digoxin
3. A patient complains of fatigue, lack of appetite, pain and burning sensation in the tongue,
numbness of the distal limbs, diarrhea. Objectively: pale skin with lemon-yellow tint, face
puffiness, brown pigmentation in the form of a "butterfly", bright red areas on the tongue. The liver
is 3 cm below the costal margin, soft. Blood count: RBCs - 1, 5 • 1012/l, colour index -1,2, WBCs -
3, 8•109 /l, thrombocytes - 180•109 /l, eosinophiles - 0%, stab neutrophiles - 1%, segmented
neutrophiles - 58%, lymphocytes - 38% monocytes - 3%, RBC macrocytosis. ESR - 28 mm/h.
What diagnosis are these presentations typical for?
A. * B12-deficiency anemia
B. Iron deficiency anemia
C. Aplastic anemia
D. Acute erythromyelosis
E. Chronic adrenal failure
4. A patient complains of weight gain, chill, edemas, xeroderma, somnolence, difficulties with
focusing. Objectively: height is 165 cm; weightis90kg;body proportions are of female type, to- 35,
8o C, heart rate - 58/min, BP -105/60 mm Hg. Heart sounds are weakened, bradycardia is observed.
Other internal organs have no changes. Thyroid gland cannot be palpated. Milk secretion from
mammary glands is observed. Hormone investigation revealed increased levels of
thyroid-stimulating and level of thyroxine(T4).Which one is the cause for obesity?
A. * Primary hypothyroidism
B. Secondary hypothyroidism
C. Prolactinoma
D. Hypopituitarism
E. Adiposogenitaldystrophy
5. A patient complains of weight gain, chill, edemas, xeroderma, somnolence, difficulties with
focusing. Objectively: height is 165 cm; weight is 90 kg; body proportions are of female type, to-
35, 8oC , heart rate - 58/min, BP - 105/60 mm Hg. Heart sounds are weakened, bradycardia is
observed. Other internal organs have no changes. Thyroid gland cannot be palpated. Milk secretion
from mammary glands is observed. Hormone investigation revealed increased levels of
thyroid-stimulating hormone (TSH) and prolactin, and decreased level of thyroxine (T4). Which
one is the cause for obesity?
A. * Primary hypothyroidism
B. Secondary hypothyroidism
C. Prolactinoma
D. Hypopituitarism
E. Adiposogenital dystrophy
6. A patient has been provisionally di-agnosed with pheochromocytoma at the stage of intermission.
BP is within norm, there is a tendency towards tachycardia. No urine pathologies. The decision has
been made to perform a provocative test with histamine. What drug should be kept close at hand for
emergency aid in case of positive test result?
A. * Phentolamine
B. Pipolphen
C. Nifedipine
D. Mesaton (Phenylephrine)
E. Prednisolone
7. A patient with autoimmune thyroiditis accompanied by multinodular goiter underwent the right
lobeectomy and subtotal resection of the left lobe.What drug should be administered to prevent
postoperative hypothyroidism?
A. * L-thyroxine
B. Merkazolil
C. Iodomarin
D. Lithium drugs
E. Insulin
8. A patient’s X-ray image (anteroposterior projection) shows deformation of lung pattern,
pneumofibrosis, reticular (honeycomb) lung pattern of lower pulmonary segments,cylindric and
fusiform lumps. The most likely diagnosis is:
A. * Multiple bronchiectasis
B. Right lungabscess
C. Non-hospital-acquired pneumonia
D. Lung small development
E. Diffuse pulmonary fibrosis
9. A patient’s X-ray image (anteroposterior projection) shows deformation of lung pattern,
pneumofibrosis, reticular (honey comb) lung pattern of lower pulmonary segments, cylindric and
fusiform lumps. The most likely diagnosis is:
A. * Multiple bronchiectasis
B. Right lung abscess
C. Non-hospital-acquired pneumonia
D. Lungs maldevelopment
E. Diffuse pulmonary fibrosis
10. A tractor driver with the record of service of 24 years has undergone palestesiometry test (Vibration
Sensitivity Measurement). Test revealed increased vibration sensitivity threshold at the frequencies
of 63-125-259 Hz to 25 dB. Dynamometry is 20 kg on the right and 16 kg on the left. Cold
stimulustestis positive, time of hand temeperature restoration is 52 minutes. Blanching at pressure
symptom is positive and equals 21. Hypesthesia of upper and lower limbs is observed and can be
classified as "gloves"and "socks"polyneuritic pattern. Make the provisional diagnosis.
A. * Vibration disease, I stage, caused by combinedvibration
B. Vibration disease, I stage, caused by local vibration
C. Vibration disease, II stage, caused by combined vibration
D. Vibration disease, II stage, caused by local vibration
E. Vibration disease, I stage, caused by general vibration
11. A woman complains of muscle weakness and general fatigue, dyspnea, vertigo, brittleness of her
hair and nails, an urge to eat chalk. Anamnesis states uterine fibroid. Common blood analysis:
erythrocytes - 2,8 Т/l, Hb- 80 g/l, color index - 0,78, anisocytosis, poikilocythemia, serum iron - 10
mcmol/l. What diagnosis is most likely?
A. * Iron-deficiency anemia
B. B12-deficient anemia
C. Autoimmune hemolytic anemia
D. Aplastic anemia
E. Hypoplastic anemia
12. A woman consulted a therapeutist about fatigability, significant weight loss, weakness, loss of
appetite. She has been having amenorrhea for 8 months. A year ago she born a full-term child.
Haemorrhage during labour made up 2 l. She got blood and blood substi-tute transfusions. What is
the most probable diagnosis?
A. * Sheehan’s syndrome
B. Stein-Leventhal syndrome
C. Shereshevsky-Turner’s syndrome
D. Homological blood syndrome
E. Vegetovascular dystonia
13. A woman has developed sudden thoracic pain on the right with expectoration of pink sputum and
body temperature rise up to 37, 7oC on the 4th day after the surgery for cystoma of the right ovary.
On lung examination: dullness of the lung sound on the lower right is observed. Isolated moist
crackles can be auscultated in the same area. What complication is the most likely?
A. * Pulmonary infarction
B. Pneumonia
C. Pulmonary abscess
D. Exudative pleurisy
E. Pneumothorax
14. A woman has developed sudden thoracicpain on the right with expectoration of pi-nk sputum and
body temperature rise up to 37, 7oC on the 4th day after the surgery for cystoma of the right ovary.
On lung examination: dullness of the lung sound on the lower right is observed. Isolated moist
crackles can be auscultated in the same area. What complication is the most likely?
A. * Pulmonary infarction
B. Pneumonia
C. Pulmonary abscess
D. Exudative pleurisy
E. Pneumothorax
15. A worker, who was involved in fire fighting inside the building that stored 2 kg of mercury, has
been delivered to a hospital with complaints of emotional expansiveness, palpitations, excessive
sweating, body tremor, heart pain. Within one day his condition aggravated. Objectively: the skin is
pale and moist. The patient is depressed. Permanent red dermographism, erethism, unstable BP are
observed. What drug is necessary in this case?
A. * Unithiol
B. Atropine sulfate
C. Calcium tetacine
D. Amyl nitrite
E. Dipyroxime
16. After having the flu, a 39-year-old male patient with a history of Addison’s disease developed a
condition manifested by weakness,depression,nausea,vomiting,diarrhea,hypoglycemia.AP-75/50
mm Hg.Blood tes results:low corticosterone and cortisol,13-oxycorticosteroids,
17-oxycorticosteroids levels.What condition developed in the patient?
A. * Acute adrenal insufficiency
B. Acute gastritis
C. Acute enterocolitis
D. Collapse
E. Diabetes mellitus
17. Against the background of angina a patient has developed pain in tubular bones. Examination
revealed generalized enlargement of lymph nodes, hepatolienal syndrome, sternalgia. In blood:
RBCs - 3, 6 • 1012/l, Hb-87 g/l, thrombocytes - 45 • 109/l, WBCs -13•109/l,blasts-87%,
stabneutrophiles-1%, segmented neutrophiles - 7%, lymphocytes - 5%, ESR - 55 mm/h. What is the
most likely diagnosis?
A. * Acuteleukemia
B. Erythremia
C. Chronic lymphocytic leukemia
D. Chronic myeloid leukemia
E. Multiplemyeloma
18. An 8-year-old girl periodically has sudden short-term heart pain, sensation of chest compression,
epigastric pain, dizziness, vomiting. Objectively: the patient is pale, respiratory rate - 40/min,
jugular pulse is present. Heart rate - 185 bpm, of poor volume. BP - 75/40 mm Hg. ECG taken
during an attack shows ectopic P waves, QRS wave is not deformed. At the end of an attack a
compensatory pause is observed. The most likely cause of the attack is:
A. * Paroxysmal atrial tachycardia
B. Sinus tachycardia
C. Paroxysmal ventricular tachycardia
D. Complete AV-block
E. Atrial fibrillation
19. An electrogas welding operator working at a machine workshop performs welding and cutting of
metal, which is accompanied by intense UV-radiation. His welding station is equipped with
efficient mechanical venti-lation. What occupational disease is most likely to develop in an
electro-gas welding operator?
A. * Photoelectric ophthalmia
B. Heatstroke
C. Vegetative-vascular dystonia
D. Chronic overheating
E. Pneumoconiosis
20. During examination a patient is unconsci-ous, his skin is dry and hot, face hyperemia is present.
The patient has Kussmaul’s respi-ration, there is also smell of acetone in the air. Symptoms of
peritoneum irritation are positive. Blood sugar is 33 millimole/l. What emergency actions should be
taken?
A. * Intravenous infusion of short-acting insulin
B. Intravenous infusion of glucose along with insulin
C. Introduction of long-acting insulin
D. Intravenous infusion of neohaemodesum along with glutamic acid
E. Intravenous infusion of sodium chloride saline
21. Examination of an electric welder with 15 years of servicerecordrevealed dry rales in the lower
lung fields. Radiograph shows diffuse nodules sized 3-4 mm in the middle and lower lung fields.
What disease can be suspected?
A. * Heavy-metalconiosis
B. Silicosis
C. Silicatosis
D. Carbon pneumoconiosis
E. Bronchitis
22. On the 4th day after recovering from acold a patient was hospitalized with complai-nts of solitary
spittings of mucoid sputum. On the 2nd day there was a single discharge of about 250 ml of
purulent blood-streaked sputum. Objectively: the patient’s condition is moderately severe.
Respiratory rate - 28-30/min., Ps- 96/min., BP- 110/70 mm Hg. Respiration above the left lung is
vesicular, weak above the right lung. There are various moist crackles above the lower lobe and
amphoric breath near the angle of scapula. What is the most likely diagnosis?
A. * Acute pulmonary abscess
B. Exudative pleuritis
C. Acute focal pneumonia
D. Pleural empyema
E. Pyopneumothorax
23. Residents of an industrial community situated near a plant suffer from increased morbidity rate
caused by nervous and endocrine system conditions and kidney diseases. Blood test: decrease of
sulfhydric groups content in blood. The pathologies developed can be caused by environment being
polluted by the following:
A. * Mercury
B. Cadmium
C. Boron
D. Chromium
E. Lead
24. Residents of an industrial community situated near a plant suffer from increased morbidity rate
caused by nervous and endocrine system conditions and kidney diseases. Blood test: decrease of
sulfhydric groups content in blood. The pathologies developed can be caused by environment being
polluted by the following:
A. * Mercury
B. Cadmium
C. Boron
D. Chromium
E. Lead
25. Survey radiograph of a 52-year-old worker of an agglomeration plant (28 years of experience, the
concentration of metal dust is 22-37mg/m3) shows mild lypronounced interstitial fibrosis with
diffused contrast well-defined small nodular shadows. The patient has no complaints. Pulmonary
function is not compromised. What is the provisional diagnosis?
A. * Siderosis
B. Silicosis
C. Anthraco-silicatosis
D. Silicatosis
E. Anthracosis
26. Survey radiograph of a 52-year-old worker of an agglomeration plant (28 years of experi-ence, the
concentration of metal dust is 22-37 mg/m3) shows mildly pronounced interstitial fi-brosis with
diffused contrast well-defined small nodular shadows. The patient has no complaints. Pulmonary
function is not compromised. What is the provisional diagnosis?
A. * Siderosis
B. Silicosis
C. Anthraco-silicatosis
D. Silicatosis
E. Anthracosis
27. Example(s) of "protective factor(s)" in peptic ulcer disease:
A. * all of the above
B. prostaglandins
C. gastric mucus
D. bicarbonate
E. None of above
28. Examples of H2 (histamine receptor Type II) antagonists:
A. * All of above
B. ranitidine (Zantac)
C. nizatidine (Axid)
D. famotidine (Pepcid)
E. cimetidine (Tagamet)
29. Exercise induced asthma is not precipitated by :
A. * Swimming in hot water
B. High altitude climb and exercises
C. Cycling in cold weather
D. Swimming in cold water
E. All of the above
30. FEV1/FVC is reduced in case of:
A. * Asthma
B. Pleural effusion
C. Lung fibrosis
D. All of the above
E. Nothing of the above
31. Findings in hemophilia A include:
A. * Partial thromboplastin time increased
B. Increased clotting time
C. Increased prothrombin time
D. Bleeding time increased
E. Thrombocytopenia
32. Following are causes of unconjugated hyperbilirubinemia, except:
A. * Rotor syndrome
B. Large hematoma
C. Hemolytic anemia
D. Megaloblastic anemia
E. Irondeficiency anemia
33. For the patient who has sustained tracheobronchial trauma, which assessment finding alerts the
physician to the possibility of tracheal lacerations?
A. Hypertympanic sound on affected side
B. * Subcutaneous emphysema over the trachea
C. Hypotension and decreased capillary refill
D. Deviation of the trachea to the affected side
E. Non of above
34. Haemophilia A is characterised by the presence of following features, except
A. Bleeding into soft tissues
B. Reduced VIII levels
C. * Prolonged bleeding time
D. Prolonged partial thromboplastin time
E. Bleeding into muscles &amp;amp;amp; joints
35. Helicobacter pylori is not associated with :
A. * Gastric leiomyoma
B. Gastric cancer
C. Peptic ulcer of stomach
D. Peptic ulcer of duodenum
E. None of above
36. Histamine and gastric acid secretion:
A. * all of the above
B. release enhanced by increased cholinergic activity
C. most important gastric acid secretion stimulant
D. released from enterochromaffin-like cells
E. None of above
37. How long is the usual course of drug treatment for a patient with active tuberculosis?
A. 7 to 10 days
B. 6 weeks
C. * 6 months
D. 2 years
E. Non of above
Назва наукового напрямку (модуля): Семестр: 12
Внутрішня медицина 6 курс
Опис:
КРОК
Перелік питань:
1. A 23-year-old woman, who works as a milk and dairy inspector, after the miscarriage suffers from
high fever up to 38, 6oC , recurring chills, excessive sweating. Objectively: polyadenitis, pain in
the lumbosacral spine, swollen left knee joint, enlarged liver and spleen. What diagnosis is the most
likely?
A. * Brucellosis
B. Sepsis
C. Toxoplasmosis
D. Polyarticular rheumatoid arthritis
E. Yersiniosis
2. A 24-year-old male patient had been diagnosed with class III diffuse toxic goiter.There is moderate
hyperthyroidism. A surgery was suggested,and the patient agreed to it.What preoperative measures
should be taken for prevention of thyrotoxic crisis in the postoperative period?
A. * Administration of antithyroid drugs
B. Minimally invasive surgical techniques
C. Bed rest
D. Detoxification therapy
E. Administration of corticosteroids
3. A 24-year-old patient visited a doctor complaining of enlargement of his submaxillary lymph
nodes. Objectively: submaxillary, axillary and inguinal lymph nodes are enlarged. Chest X-ray
shows: enlarged lymph nodes of mediastinum. Blood test: erythrocytes - 3, 4 • 1012/l, Hb- 100 g/l,
blood colour index - 0,88, platelets - 190 • 109/l, leucocytes - 7, 5•109 /l, eosinophiles - 8%, band
neutrophiles - 2%, segmented neutrophiles - 67%, lymphocytes - 23%, ESR- 22 mm/hour. What
test must be prescribed to verify the cause of lymphadenopathy?
A. * Open biopsy of lymph nodes
B. Ultrasonography of abdominal cavity
C. Mediastinum tomography
D. Puncture biopsy of lymph nodes
E. Sternal puncture
4. A 24-year-old woman, teacher by profession, complains of dizziness and heart pain irradiating to
the left nipple. Pain is not associated with physical activity and cannot be relieved by nitroglycerin,
it abates after taking Valocordin and lasts for an hour or more. The patient has a nearly 2-year
history of this di-sease. Objectively: Heart rate - 76 bpm. BP - 110/70 mm Hg. Heart borders are
normal, heart sounds are clear. The ECG shows respiratory arrhythmia. X-ray of the
cervicothoracic spine shows no pathology. Lungs, abdomen are unremarkable. What changes in
blood formula can be expected?
A. * No changes
B. Leukocytosis
C. Thrombocytopenia
D. Leukemic hiatus
E. Increased ESR
5. A 25-year-old female patient complains of marked weakness, sleepiness, blackouts, dizziness, taste
disorder.The patient has a history of menorrhagia.Objectively:the patient as marked weakness,
paleskin, cracksin the corners of mouth, peelingnails, systolic apical murmur. Blood test results:
RBC- 3, 4 • 1012/l, Hb-70g/l, colorindex-0,75, platelets - 140 • 109/l, WBC- 6, 2 • 109/l. What is
the most likely diagnosis?
A. * Chronic posthemorrhagic anemia
B. Acute leukemia
C. Acute post hemorrhagic anemia
D. B12-deficiency anemia
E. Werlhof’s disease
6. A 25-year-old patient has been admitted to the hospital with the following problems: weakness,
sweating, itching, weight loss, enlarged submandibular, cervical, axillary, inguinal lymph nodes.
Objectively: hepatomegaly. Lymph node biopsy revealed giant Berezovsky-Reed-Sternberg cells,
polymorphocellular granuloma composed of lymphocytes, reticular cells, neutrophils, eosinophils,
fibrous tissue, plasma cells. What is the most likely diagnosis?
A. * Lymphogranulomatosis
B. Lymph node tuberculosis
C. Lymphoreticulosarcoma
D. Cancer metastases to lymph nodes
E. Macofollicular reticulosis
7. A 26-year-old female patient has an11- year history of rheumatism.Four years ago she suffered 2
rheumatic attacks. Over the last 6 months there have been paroxysms of atrial fibrillation every 2-3
months. What option of antiarrhythmic therapy or tactics should be proposed?
A. Immediate hospitalization
B. Defibrillation
C. Lidocaine administration
D. Heparin administration
E. * Prophylactic administration of cordarone
8. A 26-year-old male patient complains of piercing pain during breathing, cough, dyspnea.
Objectively: to- 37, 3oC , respiration rate is 19/min, heart rate is 92/min; BP is 120/80 mm Hg.
Vesicular respiration is observed. In the inferolateral parts of chest auscultation in both inspiration
and expiration phase revealed noise that was getting stronger at phonendoscope pressing and could
be still heard after cough. ECG showed no pathological changes. What is the most likely diagnosis?
A. * Acute pleuritis
B. Intercostal neuralgia
C. Subcutaneous emphysema
D. Spontaneous pneumothorax
E. Pericarditis sicca
9. A 27-year-old woman complains of bleeding gums, nasal hemorrhages, multiple hematomas on the
skin of her limbs and on the front of her torso, extreme general fatigue. Blood test: Hb- 64 g/l,
erythrocytes - 2, 5 • 1012/l, reticulocytes - 16%, platelets 30 • 109/l, ESR- 22 mm/hour. What
approach would be most efficient for treatment of this pathology?
A. * Splenectomy
B. Dicynone (Etamsylate)
C. Platelet concentrate transfusion
D. Cytostatics
E. Group B vitamins
10. A 28-year-old woman complains of increased intermenstrual periods up to 2 months, hirsutism.
Gynaecological examinati-on revealed that the ovaries were enlarged, painless, compact, uterus had
no pecularities. Pelvic ultrasound revealed that the ovari-es were 4-5 cm in diameter and had
multiple enlarged follicles on periphery. X-ray of skull base showed that sellar region was dilated.
What is the most probable diagnosis?
A. * Stein-Leventhal syndrome (Polycystic ovary syndrome)
B. Algodismenorrhea
C. Sheehan’s syndrome

D. Premenstrual syndrome
E. Morgagni-Stewart syndrome
11. A 29-year-old female patient complains of dyspnea and palpitations on exertion. According to her
mother, as a child she had heart murmur, did not undergo any examinations. Objectively: the
patient has pale skin, Ps- 94/min, rhythmic. AP-120/60mmHg. In the II intercostal space on the left
auscultation reveals a continuous rasping systolodiastolic murmur, diastolic-above the pulmonary
artery. Blood and urine are unremarkable. What is the most likely diagnosis?
A. * Patent ductus arteriosus
B. Atrial septal defect
C. Ventricular septal defect
D. Aortarctia
E. Tetralogy of Fallot
12. A 29-year-old patient works as a motor mechanic. Anamnes is shows frequent exposure to cold,
exacerbation of chronic bronchitis attended by cough with relativly small amount of mucopurulent
sputum, subfebrility, sometimes joined by hemoptysis and pain in the right side of chest. Breathing
is vesicular. X-ray shows darkening and sharp decrease in size of the lower lobe distinctly visible
on the X-ray image as a streak 2-3 cm wide situated at the angle from lung root to the frontal
costodiaphragmatic recess. The most likely diagnosis is:
A. * Peripheral lung cancer
B. Bronchiectasis
C. Pneumonia
D. Middle lobe syndrome
E. Inter lobular pleurisy
13. A 30-year-old male patient complains of inertness, low-gradefever, bleeding gums, frequent
quinsies, aching bones. Objectively: the patient has pale skin and mucous membranes, sternalgia,+2
cm liver, +5cm painless spleen. Blood test results: RBC- 2, 7 • 1012/l, Нb-80g/l,WBC- 3 • 109/l,
eosinophils -4%, basophils-5%, blasts- 4 segmented neutrophils-17%, lymphocytes- 29%,
myelocytes-25%, promyelocytes-12%, monocytes-2%, platelets- 80 • 109/l, ESR- 57 mm/h.What
test should be performed to verify the diagnosis?
A. * Sternal puncture
B. Trephinebiopsy
C. Lymphnodebiopsy
D. Lumbar puncture
E. Chest X-ray
14. A 30-year-old woman with a long history of chronic pyelonephritis complains of considerable
weakness, sleepiness, decrease in diuresis down to 100 ml per day. BP is 200/120 mmHg.In
blood:creatinine-0,62millimole/l, hypoproteinemia, albumines - 32 g/l, potassi- um - 6,8
millimole/l, hypochromic anemia, increased ESR. What is the first step in the patient treatment
A. * tactics?
Haemodialysis
B. Antibacterialtherapy
C. Enterosorption
D. Haemosorption
E. Blood transfusion
15. A 30-year-old woman with a long history of chronic pyelonephritis complains of considerable
weakness, sleepiness, decrease in di-uresis down to 100 ml per day. BP is 200/120 mm Hg. In
blood: creatinine - 0,62 millimole/l, hypoproteinemia, albumines - 32 g/l, potassium - 6,8
millimole/l, hypochromic anemia, increased ESR. What is the first step in the patient treatment
A. * tactics?
Haemodialysis
B. Antibacterial therapy
C. Enterosorption

D. Haemosorption
E. Blood transfusion
16. A 32 year old patient complains of cardiac irregularities, dizziness, dyspnea at physical exertion.
He has never suffered from such condition before. Objectively: Ps- 74/min., rhythmic. BP- 130/80
mm Hg. Auscultation revealed systolic murmur above aorta, the first heart sound was normal. ECG
showed hypertrophy of the left ventricle, signs of repolarization disturbance in the I , V5 and V6
leads. Echocardiogram revealed that interventricular septum was 2 cm. What is the most probable
diagnosis?
A. * Hypertrophic cardiomyopathy
B. Aortic stenosis
C. Essential hypertension
D. Myocardium infarction
E. Coarctation of aorta
17. A 32-year-old welder complains of weakness and fever. His illness initially presented as tonsillitis
one month earlier. On examination: BT- 38, 9oC , RR- 24/min., HR-100/min., BP- 100/70 mm Hg,
hemorrhages on the legs, enlargement of the lymph nodes. CBC shows Hb- 70 g/l, RBC- 2, 2 •
1012/l,WBC- 3, 0 • 109/l with 32% of blasts, 1% of eosinophiles, 3% of bands, 36% of segments,
20% of lymphocytes, and 8% of monocytes, ESR- 47 mm/hour. What is the cause of anemia?
A. * Acute leukemia
B. Chronic lympholeukemia
C. Aplastic anema
D. B12-deficient anemia
E. Chronic hemolytic anemia
18. A 32-year-old woman complains of di-zziness, headache, palpitation, tremor. For the last several
months she has been under outpatient observation for increased arteri-al pressure. Since recently
such attacks have become more frequent and severe. Objectively: the skin is covered with clammy
sweat, tremor of the extremities is present. HR-110/min., BP- 220/140 mm Hg. Heart sounds are
muffled. Blood test results: WBCs- 9, 8 •/l, ESR- 22 mm/hour. Blood glucose - 9,8 millimole/l.
What disease is the most likely cause of this crisis?
A. * Pheochromocytoma
B. Essential hypertension
C. Preeclampsia
D. Primary hyperaldosteronism
E. Diabetic glomerulosclerosis
19. A 32-year-old woman complains of dizziness, headache,palpitation, tremor. For the last several
months she has been under outpatient monitoring for increased arterial pressure. Recently such
attacks have become more frequent and severe. Objectively:skin is covered with clammy sweat,
tremor of the extremitiesispresent.Heartrate-110/min,BP220/140 mm Hg. Heart sounds are
weakened. In blood: WBCs - 9, 8 • 109/l, ESR - 22 mm/h. Blood glucose - 9,8 millimole/l. What
disease is the most likely cause of this crisis?
A. * Pheochromocytoma
B. Essentialhypertension
C. Preeclampsia
D. Primary hyper aldosteronism
E. Diabetic glomerulosclerosis

20. A 34-year-old patient after vacation in Crimea has developed severe pain in her elbow joints,
dyspnea and weakness. Body temperature is 37, 6o C, skin pallor anderythema on the cheeks and
bridge of nose are observed, lip mucosa is ulcerated. The joints are not visibly deformed,
movement of the right elbow joint is restricted. Pleura friction sound is detected in the lungs on the
right side below the angle of scapula. Heart sounds are dull, tachycardia, gallop rhythm, heart rate -
114/min, BP - 100/60 mm Hg. The most likely diagnosisis:
A. * Systemic lupus erythematosus
B. Rheumatic carditis
C. Rheumatoid arthritis
D. Infectious allergic myocarditis
E. Dry pleurisy
21. A 34-year-old patient complains of profuse sweating at night, skin itching, weight loss (9 kg within
the last 3 months). Examination revealed malnutrition, skin pallor. Palpation of neck and inguinal
areas revealed dense elastic lymph nodes of about 1 cm in diameter, nonmobile, non-adhering to
skin. What is the most probable diagnosis?
A. * Lymphogranulomatosis
B. Chronic lymphadenitis
C. Lymphosarcoma
D. Burkitt’s lymphoma
E. Cancer metastases
22. A 35-year-old female patient has gained 20 kg weight within a year with the normal diet. She
complains of chill,sleepiness,dyspnea. The patient’s mother and sister are corpulent. Objectively:
height - 160 cm, weight - 92 kg, BMI - 35,9. Obesity is uniform, there are no striae. The face is
amimic. The skin is dry. The tongue is thickened. Heart sounds are muffled. Heart rate - 56/min, BP
- 140/100 mm Hg. The patient has constipations, amenorrhea for 5 months.TSH-28mkME/l(normal
rate-0,32- 5). Craniogram shows no pathology. What is the etiology ofobesity?
A. * Hypothyroid
B. Hypoovarian
C. Hypothalamic-pituitary
D. Alimentary andconstitutive
E. Hypercorticoid
23. A 35-year-old female patient has gained 20 kg weight within a year with the normal diet. She
complains of chill, sleepi-ness, dyspnea. The patient’s mother and sister are corpulent. Objectively:
height - 160 cm, weight - 92 kg, BMI- 35,9. Obesity is uniform, there are no striae. The face is
amimic. The skin is dry. The tongue is thickened. Heart sounds are muffled. Heart rate - 56/min.,
BP- 140/100 mm Hg. The patient has consti-pations, amenorrhea for 5 months. TSH- 28 mkME/l
(normal rate - 0,32-5). Craniogram shows no pathology. What is the etiology of obesity in this
A. * case?
Hypothyroid
B. Hypo-ovarian
C. Hypothalamic-pituitary
D. Alimentary and constitutive
E. Hypercorticoid
24. A 37-year-old man suffers from attacks of unconsciousness, dyspnea during physical exertion,
periodical sensations of heart rate disorder. Father of the patient died suddenly at the age of 45.
Objectively: heart rate is 90/min., BP is 140/90 mm Hg. On heart US: ejection fraction - 49%,
significant myocardium thickening of the left ventricle and interventricular septum. What drug
should be prescribed for the treatment
A. * Bisoprolol
B. Enalapril
C. Phenyhydinum (Nifedipine)

D. Hydrochlorothiazide
E. Furosemide
25. A 39-year-old patient complains of morning headache, appetite loss, nausea, morning vomi-ting,
periodic nasal haemorrhages. The patient had acute glomerulonephritis at the age of 15.
Examination revealed rise of arterial pressure up to 220/130 mm Hg, skin haemorrhages on his
arms and legs, pallor of skin and mucous membranes. What biochemical parameter is the most
important for making diagnosis in this case?
A. * Blood creatinine
B. Blood bilirubin
C. Blood sodium
D. Uric acid
E. Fibrinogen
26. A 39-year-old patient complains of morning headache, appetite loss, nausea, morning vomiting,
periodic nasal haemorrhages. The patient had a case of acute glomerulonephritis at the age of 15.
Examination revealed rise of arterial pressure up to 220/130 mm Hg, skin haemorrhages on his
arms and legs, pallor of skin and mucous membranes. What biochemical parameter is most
important for making diagnosis in this case?
A. * Blood creatinine
B. Blood bilirubin
C. Blood sodium
D. Uric acid
E. Fibrinogen
27. A 40-year-old female patient complains of having a bulge on the anterior surface of neck for 5
years. Objectively: Heart rate - 72 bpm, arterial pressure - 110/70 mm Hg, in the right lobe of
thyroid gland palpation reveals a mobile 4x2cm node,the left lobe is not palpable, the basal
metabolicrate is 6%.What is the most likely diagnosis?
A. * Nodular euthyroidgoiter
B. Nodular hyperthyroidgoiter
C. Riedel’sthyroiditis
D. Mixed euthyroidgoiter
E. The median cervicalcyst
28. A 40-year-old female patient complains of having a bulge on the anterior surface of neck for 5
years. Objectively: Heart rate - 72 bpm, arterial pressure - 110/70 mm Hg, in the right lobe of
thyroid gland palpation reveals a mobile 4x2 cm node, the left lobe is not palpable, the basal
metabolic rate is 6%. What is the most likely diagnosis?
A. * Nodular euthyroid goiter
B. Nodular hyperthyroid goiter
C. Riedel’s thyroiditis
D. Mixed euthyroid goiter
E. The median cervical cyst
29. A 40-year-old female patient complains of having a bulge on the anterior surface of neck for 5
years. Objectively: Heart rate - 72 bpm, arterial pressure - 110/70 mm Hg, in the right lobe of
thyroid gland palpation reveals a mobi-le 4x2 cm node, the left lobe is not palpable, the basal
metabolic rate is 6%. What is the most likely diagnosis?
A. * Nodular euthyroid goiter
B. Nodular hyperthyroid goiter
C. Riedel’s thyroiditis
D. Mixed euthyroid goiter
E. The median cervical cyst

30. A 40-year-old female patient complains of having a bulge on the anterior surface of neck for 5
years. Objectively: Ps- 72/min., arterial pressure - 110/70 mm Hg, in the ri-ght lobe of thyroid
gland palpation reveals a mobile node 4x2 cm in size, the left lobe is not palpable, the basal
metabolic rate is 6%. What is the most likely diagnosis?
A. * Nodular euthyroid goiter
B. Nodular hyperthyroid goiter
C. Riedel’s thyroiditis
D. Mixed euthyroid goiter
E. Median cervical cyst
31. A 40-year-old patient has acute onset of disease caused by overexposure to cold. Temperature has
increased up to 39oC . Foul-smelling sputum is expectorated during coughig. Various moist
crackles can be auscultated above the 3rd segment on the ri-ght. Blood test: leukocytes - 15, 0 •
109/l, stab neutrophils - 12%, ESR- 52 mm/hour. On X-ray: in the 3rd segment on the right there is
a focus of shadow 3 cm in diameter, low density, with fuzzy smooth margins and a clearing in its
center. What disease is most likely in the given case?
A. * Pneumonia complicated by an abscess
B. Infiltrative tuberculosis
C. Peripheral pulmonary cancer
D. Cystic echinococcosis
E. Pulmonary cyst
32. A 40-year-old patient has acute onsetof disease caused by overexposure to cold. Temperature has
increased up to 39oC . Foul-smelling sputum is expectorated during coughig. Various moist
crackles can be auscultated above the 3rd segment on the right. Blood test: leukocytes - 15, 0 •
109/l, stab neutrophils - 12%, ESR- 52 mm/hour. On X-ray: in the 3rd segment on the right there is
a focus of shadow 3 cm in diameter, low density, with fuzzy smooth margins and a clearing in its
center. What disease is most likely in the given case?
A. * Pneumonia complicated by an abscess
B. Infiltrative tuberculosis
C. Peripheral pulmonary cancer
D. Cystic echinococcosis
E. Pulmonary cyst
33. A 40-year-old patient suffers from influenza. On the 5th day of illness there are pain behind
sternum, cough with sputum, inertness. Temperature is 39, 5o C. Faceispale. Mucosa of
conjunctivas and pharynx is hyperemic. Heart rate is 120/min, breathing rate is 38/min. In the
lower lung segments shortening of percussion sound and moist rales (crackles) can be detected.
What additional investigation should be performed first of all to specify the diagnosis?
A. * Lung X-ray
B. ECG
C. Heart US
D. Mantoux test
E. Spirography
34. A 40-year-old patient suffers from influenza. On the 5th day of illness there are pain behind
sternum, cough with sputum, inertness. Temperature is 39, 5 oC. Face is pale. Mucosa of
conjunctivas and pharynx is hyperemic. Heart rate is 120/min, breathing rate is 38/min. In the
lower lung segments shortening of percussion sound and moist rales (crackles) can be detected.
What additional investigation should be performed first of all to specify the diagnosis?
A. * Lung X-ray
B. ECG
C. Heart US
D. Mantoux test

E. Spirography
35. A 40-year-old woman with a history of combined mitral valve disease with predominant stenosis
complains of dyspnea, asthma attacks at night, heart problems. At present, she is unable to do even
easy housework. What is the optimal tactics of the patient treatment?
A. * Mitral commissurotomy
B. Implantation of an artificial valve
C. Antiarrhythmic therapy
D. Treatment of heart failure
E. Antirheumatic therapy
36. A 41-year-old male patient was delivered to a hospital unconscious. During the previous 7 days he
had been taking large doses of biseptolum for a cold.The night before, he began complaining of
dyspnea, especially when lying down, swollen legs, 2-day urinary retention. In the morning he had
seizures and lost consciousness. Objctively: noisy breathing at the rate of 30/min,edematous legs
and lumbar region, Ps-50/min. Plasma creatinine is 0,586mmol/l, plasma potassium-7,2mmol/l.
What treatment is necessary for this patient?
A. * Hemodialysis
B. Large doses of verospiron
C. Plasma volumeex panders
D. Glucocorticosteroids
E. Heparin
37. A 41-year-old patient with Addison’s disease had influenza. After that he developed adynamia,
depression, nausea, vomiting, diarrhea and hypoglycemia. BP is 75/50 mm Hg. Blood test:
decreased content of corticosterone, hydrocortisone, 13-oxycorticosteroids, 17-oxycorticosteroids.
What condition has developed in thepatient?
A. * Acute adrenal gland insufficiency
B. Acute gastritis
C. Acute enterocolitis
D. Collapse
E. Diabetes mellitus
38. A 43-year-old female patient complains of dyspnea, swelling of legs, abdomen enlargement,
pricking heart pain. She has a history of tuberculousbronchoadenitis, quinsies.The patient’s
condition deteriorated 6 months ago. Objectively: cyanosis, bulging neck veins, vesicular
breathing. Heart borders are not displaced. Heart sounds are muffled, Ps- 106/min, liver is +4cm,
ascites is present. Low voltage on the ECG has been revealed. Radiograph shows a thin layer of
calcium deposits along the left contour of heart. What treatment should be recommended to the
A. * patient?
Treatment by a cardiac surgeon
B. Digitalis preparations
C. Anti-TB drugs
D. Diuretics
E. Vasodilators, nitrates
39. A 44-year-old man complains of dyspnea with sensation of lack of air on inhale, palpitations
occurring during slight physical exertion, and shin edemas that appear in evening and resolve in
morning. His condition has been lasting for 5 months already, deterioration is gradual. What
method of instrumental diagnostics allows to verify decrease of systolic function in this patient?
A. * Echocardiography
B. Electrocardiography
C. Computed tomogram
D. Phonocardiography
E. Holter blood pressure monitoring

40. A 45-year-old man was delivered to a hospital with complaints of vomiting wi-th streaks of blood,
loss of weight. On esophagofiberscopy a cauliflower-shaped mucosal growth was detected in the
abdominal esophagus. The mucosa there bleeds on contact. What preliminary diagnosis can be
made?
A. * Esophageal tumor
B. Barrett esophagus
C. Abdominal esophagitis
D. Esophageal diverticulum
E. Esophageal achalasia
41. A 45-year-old patient complains of pain in the epigastric region, left subcostal area, abdominal
distension, diarrhea, loss of wei-ght. He has been suffering from this condition for 5 years.
Objectively: tongue is moist with white coating near the root; deep palpation of abdomen reveals
slight pain in the epigastric region and Мауо-Robson’s point. Liver is pai-nless and protrudes 1 cm
from the costal arch. Spleen cannot be palpated. What disease can be primarily suspected?
A. * Chronic pancreatitis
B. Atrophic gastritis
C. Peptic stomach ulcer
D. Chronic cholecystitis
E. Acute cholecystitis
42. A 45-year-old patient complains of pain in the epigastric region, left subcostal area, abdominal
distension, diarrhea, loss of wei-ght. He has been suffering from this condition for 5 years.
Objectively: tongue is moist with white coating near the root; deep palpation of abdomen reveals
slight pain in the epigastric region and Мауо-Robson’s point. Liver is pai-nless and protrudes 1 cm
from the costal arch. Spleen cannot be palpated. What disease can be primarily suspected?
A. * Chronic pancreatitis
B. Atrophic gastritis
C. Peptic stomach ulcer
D. Chronic cholecystitis
E. Chronic enteritis
43. A 46-year-old male patient complains of periodic epigastric painthat occurs at night. Objectively:
HR-70/min, AP-125/75 mm Hg, tenderness in the epigastric region is present. EGD confirms
duodenal ulcer of 0,6cm in diameter.TestforH.Pyloriispositive.Which of the given antisecretory
drugs will be a compulsory element of the treatment regimen?
A. * Omeprazole
B. Famotidine
C. Pirenzepine
D. Atropine
E. Maalox
44. A 46-year-old patient with temporarily undetermined diagnosis was prescribed pleurocentesis based
on the results of the X-ray. The puncture yielded 1000 ml of a liquid with the following properties:
clear, specific gravity - 1,010, protein content - 1%, Rivalta’s test is negative, erythrocytes - 2-3 in
the field of vision. What disorder are these pathologic changes characteristic of?
A. * Cardiac failure
B. Pleuropneumonia
C. Pleural mesothelioma
D. Pulmonary tuberculosis
E. Pulmonary cancer

45. A 47-year-old female patient complains of cough with purulent sputum, pain in the lower left chest,
periodical body temperature rise. She has been suffering from this condition for about 10 years.
Objectively: "drumstick" distal phalanges. What examination would be the most informative for
making a diagnosis?
A. * Bronchography
B. Bronchoscopy
C. Survey radiograph of lungs
D. Pleural puncture
E. Bacteriological analysis of sputum
46. A 47-year-old female patient complains of cough with purulent sputum, pain in the lower left chest,
periodical body temperature rise. She has been suffering from this condition for about 10 years.
Objectively: "drumstick"distal phalanges. What examination would be the most informative for
making a diagnosis?
A. * Bronchography
B. Bronchoscopy
C. Survey radiograph of lungs
D. Pleural puncture
E. Bacteriological analysis of sputum
47. A 47-year-old male patient complains of compressive chest pain that occurs both at rest and during
light physical activity; irregular heartbeat. These problems arose 3 months ago. The patient’s
brother died suddenly at the age of 30. Objectively: Ps-84/min ,arrhythmic, AP- 130/80 mm Hg.
ECG confirms signs of left ventricular hypertrophy, abnormal Q- waves in V 4 - V 6 leads.
EchoCG revealst hat interventricular septum is 1,7cm, left ventricular wall thickness is 1,2cm.What
is the most likely diagnosis?
A. * Hypertrophic cardiomyopathy
B. Neurocirculatoryasthenia
C. Exertional angina
D. Myocarditis
E. Pericarditis
48. A 47-year-old male patient has been lately complaining of compressing chest pain that occurs when
he walks a distance of 700-800 m. Once a week, he drinks 2 liters of beer. Rise in arterial pressure
has been observed for the last 7 years. Objectively: Ps- 74/min, AP- 120/80 mm Hg. The bicycle
ergometry performed at workload of 75 watts shows 2mm ST-segment depressionin V 4?V6
leads.What is the most likely diagnosis?
A. * Exertional stenocardia, II functional class
B. Exertional stenocardia, III functional class
C. Exertional stenocardia, IV functional class
D. Vegetative-vascular dystonia of hypertensive type
E. Alcoholic cardiomyopathy
49. A 48-year-old man complains of constant pain in the upper abdomen, predominantly on the left,
which aggravates after eating, diarrhea, loss of weight. The patient has alcohol use disorder. Two
years ago he had a case of acute pancreatitis. Blood amylase is 4 g/hour•l. Feces analysis:
steatorrhea, creatorrhea. Blood sugar is 6,0 mmol/l. What treatment should be prescribed?
A. * Panzinorm forte (Pancreatin)
B. Insulin
C. Gastrozepin (Pirenzepine)
D. Contrykal (Aprotinin)
E. No-Spa (Drotaverine)

50. A 48-year-old patient has the following symptoms: diffuse enlargement of thyroid
gland,exophthalmus,weight loss up to 4kg wi- thin 2months,excessive sweating.Objectively:
heartrate 105/min,BP-180/70mmHg. Stool is normal. What therapy is advisable in the given case?
A. * Mercazolil (Thiamazole)
B. Potassium iodide
C. Propranolol
D. Iodomarin
E. Thyroxin
51. A 48-year-old patient was found to have diffuse enlargement of the thyroid gland, exophthalmia,
weight loss of 4 kg in 2 months, sweating. Objectively: HR- 105/min., BP-140/70 mm Hg.
Defecation act is normal. What kind of therapy is recommended in this case?
A. * Mercazolil
B. Radioiodine
C. Propranolol
D. Lugol’s solution
E. Thyroxine
52. A 48-year-old woman has been suffering from chronic pancreatitis for the last 7 years. Lately she
has been noticing an increase in daily feces with foul smell, abdominal di-stention, gurgling. The
patient complains of diarrhea, weakness, fatigability, loss of appeti-te, loss of weight. What
syndrome can be suspected in this case?
A. * Malabsorption
B. Irritable colon
C. Maldigestion
D. Exudative enteropathy
E. Endocrine gland failure
53. A 49-year-old female patient has type 1 diabetes of moderate severity.The disease is complicated by
retinopathy and polyneuropathy. Besides that, repeated analyses of the daily urinary excretion of
albumin revealed microalbuminuria(200- 300mg/day).Glomerular filtration rate is 105ml/min.
Blood pressure is within normal range. Normalization of the following indicator should be the
first-priority task in the secondary prevention of diabetic nephropathy:
A. * Glycosylated hemoglobin
B. C-peptide
C. Blood insulin
D. Fastingglucose
E. Glycemia 2hours after a meal
54. A 49-year-old male patient complains of retrosternal pain, heartburn,weight loss of 8 kg over the
last year,constipation,weakness. The patient has been a smoker for 20years, and has a10-year
history of gastroesophageal reflux disease.The patient is asthenic,has dry skin. EGD revealed an
ulcer in the lower third of the sophagus and esophageal stricture accompanied by edema,hyperemia
and multiple erosions of the mucosa.What study is required for more accurate diagnosis?
A. * Biopsy of the esophageal mucosa
B. X-rayexamination of the esophagus
C. Respiratory test for Helico bacter pylori
D. pH-metry of the esophagus and the stomach
E. Fecal occult blood test

55. A 49-year-old patient 2 years ago was diagnosedwithstage1silicosis. He complains of increased


dyspnea and pain in the infrascapular regions. X-ray shows diffuse intensification and distortion of
lung pattern with numerous nodularshadows2-4mmindiameter.Hardening of right interlobar pleura
is detected. Lung roots are hardened and expanded. What X-ray type of pneumosclerosis does the
patient have?
A. * Nodular
B. Interstitial
C. Interstitial-nodular
D. Nodal
E. Tumor-like
56. A 49-year-old patient complains of swallowing disorder that intensifies duri-ng eating solid food,
hiccups, hoarse voice, nausea, regurgitation, significant weight loss (15 kg within 2,5 months).
Objectively: body weight is reduced; the skin is pale and dry; vesicular respiration; heart sounds are
suffi-ciently sonorous; heart rate is rhythmic. The abdomen is soft, no pain on palpation. The liver
is not enlarged. What investigation is most necessary for making the diagnosis in this case?
A. * Esophagoduodenoscopy with biopsy
B. Clinical blood analysis
C. X-ray of the gastrointestinal tract
D. X-ray in the Trendelenburg position
E. Investigation of gastric secretion
57. A 50-year-old man, who works as a polisher at a combine-building factory, addressed the factory’s
sectorial doctor with complaints of general fatigue, sensations of numbness and pain in his fingers.
Objectively: the skin of his fingers is pale. Reaction to pain, tactile and thermal stimuli was
revealed to be slightly disrupted. No disruptions can be observed within the other organs and
systems. What disorder is the most likely?
A. * Pneumatic hammer disease
B. Multiple neuritis
C. Raynaud’s disease
D. Syringomyelia
E. Deforming arthrosis
58. A 50-year-old patient complains about having pain attacks in the right subcostal area for about a
year. The pain arises mainly after taking fatty food. Over the last week the attacks occurred daily
and became more pai-nful. On the 3rd day of hospitalization the pati-ent presented with
icteritiousness of skin and scleras, light-coloured feces and dark urine. In blood: neutrophilic
leukocytosis - 13, 1 • 109/l, ESR - 28 mm/h. What is the most likely diagnosis?
A. * Chronic calculous cholecystitis
B. Chronic recurrent pancreatitis
C. Fatty degeneration of liver
D. Chronic cholangitis, exacerbation stage
E. Hypertensive dyskinesia of gallbladder
59. A 51-year-old female patient complains of frequent defecation and liquid blood-streaked stools
with mucus admixtures, diffuse pain in the inferolateral abdomen, 6 kg weight loss within the
previous month. Objectively: body temperature - 37, 4oC , malnutrition, skin is pale and dry.
Abdomen is soft, sigmoid is pai-nful and spasmodic, makes a rumbling sound. Liver is dense,
painful, extends 3 cm below the costal margin. What is the most likely di-agnosis?
A. * Non-specific ulcerative colitis
B. Bacillary dysentery
C. Sprue
D. Intestinal enzymopathy
E. Helminthic invasion

60. A 52-year-old patient complains of pain in the right part of her chest, dyspnea, cough with a lot of
albuminoid sputum emitting foul smell of "meat slops". Objectively: the patient’s condition is
grave, cyanosis is observed, breathing rate is 31/min, percussion sound above the right lung is
shortened, auscultation revealed various moist rales (crackles). What is the most probable
A. * diagnosis?
Lung gangrene
B. Lung abscess
C. Pleura empyema
D. Multiple bronchiectasis
E. Chronic pneumonia
61. A 52-year-old patient, who has been suffering from angina pectoris for 2 weeks, has more and more
frequent pain attacks in the area behind his sternum and his need for nitroglycerine increased.
Objectively: the condition is of moderate severity. Skin is pale. Heart sounds are weakened,
rhythmic. Heart rate is 76 per minute. ECG shows no signs of focal myocardial damage. What is
the most likely diagnosis?
A. * Progressive angina pectoris
B. First-time angina pectoris
C. Stable FC II angina pectoris
D. Variant angina pectoris
E. Acute cardiac infarction
62. A 53-year-old woman complains of weight loss up to 10 kg within the last 2 years, liquid
foul-smelling stool two times a day that poorly washes off the toilet, periodic bouts of nausea,
girdle pain in the upper abdomen. Objecti-vely: pain in Gubergrits zone (on the right from navel)
and Mayo-Robson’s point. Biochemical blood analysis: glucose - 3,2 mmol/l, bilirubin - 16,5
micromole/l, crude protein - 56,4 g/l. Urine diastase - 426 g/h/l. D-xylose test (oral admi-nistration
of 25 g of d-xylose) after 5 hours reveals 3 g of xylose in urine. The most likely diagnosis is:
A. * Pancreatitis. Malabsorption syndrome
B. Pseudomembranous colitis
C. Nonspecific ulcerative colitis
D. Irritable bowel syndrome
E. Chronic gastritis
63. A 54-year-old man had been drowning at sea, when he was found and evacuated to the shore.
Objectively: unconscious, pale face, no breathing can be auscultated, thready pulse. Resuscitation
measures allowed to save the man. What complication can develop in him in the nearest future?
A. * Pulmonary edema
B. Respiratory arrest
C. Encephalopathy
D. Cardiac arrest
E. Bronchial spasm
64. A 54-year-old patient complains of weakness, jaundice, itching skin. Disease onset was 1,5 months
ago: fever up to 39oC appeared at first, with progressive jaundice developed 2 weeks later. On
hospitalisation jaundice was severely progressed. Liver cannot be palpated. Gall bladder is enlarged
and painless. Blood bilirubin is 190 micromole/l (accounting for direct bilirubin). Stool is acholic.
What is the most likely jaundice genesis in this patient?
A. * Mechanical jaundice
B. Hepatocellular jaundice
C. Hemolytic jaundice
D. Caroli syndrome
E. Gilbert’s syndrome

65. A 54-year-old patient complains of weakness, weight loss despite the unchanged appetite, frequent
urination, skin itch for six months. Some time ago the patient underwent treatment for furunculosis.
She has not been examined recently. Objectively: malnutrition, dry skin with scratch marks. Small
lymph nodes can be palpated in the axillary regions. Changes in the internal organs are absenr.
What test must be performed in the first place?
A. * Fasting blood sugar
B. Complete blood count
C. Endoscopy of stomach
D. Lymph node biopsy
E. Blood sterility testing
66. A 54-year-old patient complains of weakness, weight loss despite the unchanged appetite, frequent
urination, skin itch for six months. Some time ago the patient underwent treatment for furunculosis.
She has not been examined recently. Objectively: malnutrition, dry skin with scratch marks. Small
lymph nodes can be palpated in the axillary regions. Changes in the internal organs are absenr.
What test must be performed in the first place?
A. * Fasting blood sugar
B. Complete blood count
C. Endoscopy of stomach
D. Lymph node biopsy
E. Blood sterility testing
67. A 54-year-old patient has been suffering from diabetes mellitus for 5 years, with diet being his only
treatment. Within the last half a year he lost 7 kg of body weight, complains of thirst, vertigo when
raising from bed, decrease of erectile function, frequent stool, especially at night. Objectively:
malnutrition, dry skin. BP in lying position is 160/90 mm Hg; BP in standing position is 170/85
mm Hg. No edemas. Fasting plasma glucose level is 12 mmol/l. Glycated hemoglobin accounts for
11%. Albumin excreted with urine is 20mg per day.The most likely diagnosis is:
A. * Diabetes mellitus type 2 with visceral neuropathy
B. Diabetes mellitus type 1with ketoacidosis
C. Diabetes mellitus type 1 with encephalomyelopathy
D. Diabetes mellitus type 2 with nephropathy
E. Diabetes mellitus type 2 with polyneuropathy
68. A 56-year-old patient complains of pain in the epigastrium after eating, eructation, loss of appetite,
slight loss of weight, fatigabili-ty. The patient smokes; no excessive alcohol consumption.
Objectively: pale mucosa, BP-110/70 mm Hg. The tongue is ”lacquered”. The abdomen is soft,
sensitive in the epi-gastric area. Blood test: erythrocytes - 3,0 T/l, Hb- 110 g/l, color index - 1,1;
macrocytosis; leukocytes - 5,5 g/l, ESR- 13 mm/hour. On fibrogastroduodenoscopy: atrophy of
fundic mucosa. What pathogenesis does this disorder have?
A. * Producing antibodies to parietal cells
B. Н.pyloripersistence
C. Alimentary factor
D. Chemical factor
E. Gastropathic effect
69. A 56-year-old patient with diffuse toxic goiter has ciliary arrhythmia, heart rate is 110-120/min.,
arterial hypertension, BP is 165/90 mm Hg. What drug besides Mercazolil (Thi-amazole) should be
prescribed in this case?
A. * Propranolol
B. Radioactive iodine
C. Novocainamide (procainamide)
D. Verapamil
E. Corinfar

70. A 56-year-old patient with diffuse toxic goiter has ciliary arrhythmia, heart rate is 110-120/min.,
arterial hypertension, BP is 165/90 mm Hg. What drug besides Mercazolil (Thi-amazole) should be
prescribed in this case?
A. * Propranolol
B. Radioactive iodine
C. Novocainamide (procainamide)
D. Verapamil
E. Corinfar
71. A 56-year-old woman complains of itchi-ng skin of her torso, constant nausea, constipation,
sensation of heaviness and pain in the right subcostal area, extreme general fatigue. The patient
suffers from biliary cirrhosis. The skin is pale icteric. The abdomen is soft, the liver protrudes 2,0
cm from under the margin of the right costal arch, sensitive on palpation. Biochemical
investigation: total bilirubin - 142,0 mcmol/l, conjugated bilirubin - 139,0 mcmol/l, alanine
aminotransferase - 0,98 mmol/hour•l, aspartate aminotransferase - 0,82 mmol/hour•l, alkaline
phosphatase - 8,7 mmol/hour•l. What drug should be prescribed in the first place?
A. * Ursodeoxycholic acid
B. Sirepar
C. Allochol
D. Essentiale forte (Phospholipides)
E. Livolin forte
72. A 57-year-old male patient had an attack of retrosternal pain that lasted more than 1,5 hours.
Objectively: the patient is inert, adynamic, has pale skin, cold extremities, poor volume pulse, heart
rate-120/min, AP-70/40 mm Hg. ECG shows ST elevationin II, III, aVF leads. What condition are
these changes typical for?
A. * Cardiogenic shock
B. Arrhythmogenic shock
C. Perforated gastriculcer
D. Acute pericarditis
E. Acute pancreatitis
73. A 57-year-old woman complains of havi-ng a sensation of esophageal compresion, palpitation,
difficult breathing during eating solid food, occasional vomiting with a full mouth, ”wet pillow”
sign at night for the last 6 months. Objectively: body tempearture - 39oC , height - 168 cm, weight
- 72 kg, Ps-76/min., BP- 120/80 mm Hg. X-ray revealed a considerable dilation of the esophagus
and its constriction in the cardial part. What pathology is most likely to have caused dysphagia in
this patient?
A. * Achalasia cardiae
B. Primary esophagism
C. Hiatal hernia
D. Esophageal carcinoma
E. Reflux esophagitis
74. A 57-year-old woman complains of having a sensation of esophageal compresion, palpi-tation,
breathing difficulties when eating solid food, occasional vomiting with a full mouth, "wet
pillow"sign at night for the last 6 months. Objectively: body tempearture - 39oC , height - 172cm,
weight - 72 kg, heart rate - 76/min, BP-120/80 mm Hg. X-ray revealed considerable dilation of
esophagus and its constriction in the cardial part. What pathology is most likely to have caused
dysphagia in this patient?
A. * Achalasia cardiae
B. Primary esophagism
C. Hiatal hernia
D. Esophageal carcinoma

E. Reflux esophagitis
75. A 58-year-old female patient complains of spontaneous bruises, weakness, bleeding
gums,dizziness.Objectively:the mucous membranes and skin are pale with numerous hemorrhages
of various time of origin. Lymph nodes are not enlarged.Ps-100/min,AP- 110/70 mmHg.There are
no changes of internal organs. Blood test results:RBC- 3,0•1012/l, Нb-92g/l,colour index-0,9,
anisocytosis, poikilocytosis,WBC-10•109/l, eosinophils -2%,stab neutrophils-12%, segmented
neutrophils-68%,lymphocytes - 11%, monocytes-7%, ESR-12mm/h.What laboratory test is to be
determined next for making adiagnosis?
A. * Platelets
B. Reticulocytes
C. Clottingtime
D. Osmotic resistance of erythrocytes
E. Fibrinogen
76. A 58-year-old patient complains of general weakness, weight loss up to 10 kg within the last 1,5
months, progressive pain in the small of the back, raise of blood pressure to 220/160 mm Hg,
subfebrile body temperature. Objectively: tuberous slightly movable lump can be palpated in the
right subcostal area; veins of spermatic cord and scrotum are dilated. Blood test: Hb - 86 g/l, ESR -
44 mm/h. Urine test: specific gravity - 1020, proteine - 0,99 g/l,erythrocytes-all field of vision,
leukocytes4-6 in the field of vision. The provisional diagnosisis:
A. * Kidney tumor
B. Urolithiasis
C. Acute pyelonephritis
D. Acute glomerulonephritis
E. Nephroptosis
77. A 58-year-old patient complains of headache in the occipital region, nausea, choking, opplotentes.
The presentations appeared after a physical exertion. Objectively: the patient is excited. Face is
hyperemic. Skin is pale. Heart sounds are regular, the 2nd aortic sound is accentuated. BP - 240/120
mm Hg, heart rate - 92/min. Auscultation reveals some fine moist rales (crackles) in the lower parts
of the lungs. Liver is not enlarged. ECG shows signs of hypertrophy and left ventricular overload.
What is the most likely diagnosis?
A. * Complicated hypertensive crisis
B. Acute myocardial infarction, pulmonaryedema
C. Bronchial asthma exacerbation
D. Uncomplicated hypertensic crisis
E. Community-acquired pneumonia
78. A 58-year-old patient was delivered to an admission room with complaints of pain in the thorax on
the left. On clinical examination: aside from tachycardia (102/min.) no other changes. On ECG:
pathologic wave Q in I, аVL, QS in V1, V2, V3 leads and ’domed’ ST elevation with negative T.
What diagnosis is the most likely?
A. * Acute left ventricular anterior myocardialinfarction
B. Variant angina pectoris
C. Aortic dissection
D. Acute left ventricular posterior myocardialinfarction
E. Exudative pericarditis
79. A 60-year-old woman developed weakness, vertigo, rapid fatigability during the last year. Recently
there have been dyspnea and paresthesia observed. Objectively: skin and mucous membranes are
pale and icteric. Lingual papillae are smoothed out. Liver and spleen are situated at the edge of
costal arch. Blood test: Hb- 70 g/l, erythrocytes - 1, 7•1012 /l, blood color index - 1,2, macrocytes.
What drug can be prescribed on pathogenetic grounds?
A. * VitaminB12

B. VitaminB6
C. Ascorbic acid
D. Iron preparations
E. VitaminB1
80. A 60-year-old woman started feeling weakness, vertigo, rapid fatigability during the last year.
Recently there have been dyspnea and paresthesia observed. Objectively: skin and mucous
membranes are pale and icteric. Lingual papillae are smoothed out. Liver and spleen are situated at
the edge of costal arch. Bloodtest: Hb-70g/l,erythrocytes-1,7•1012/l, blood colorindex-1,2,
macrocytes.What drug can be prescribed on pathogenetic grounds?
A. * Vitamin B12
B. Vitamin B6
C. Ascorbic acid
D. Iron preparations
E. Vitamin B1
81. A 62-year-old man addresed a urologist with complaints of frequent urination at night (5-6 times
per night), sensation of incomplete voiding of the urinary bladder, pain in the lower abdomen, slow
urination. Anamnesis: the II degree essential hypertension (peak BP is 160/100 mm Hg). Current
case: the II degree enlargement of the prostate gland, PSA is 2,2 ng/ml. Select the drug suitable for
long-term therapy of the patient’s combined pathology:
A. * Doxazosin
B. Propranolol
C. Indapamide
D. Amlodipine
E. Captopril
82. A 63-year-old female complains of general weakness, a feeling of heaviness, compression in the
epigastrium, postprandial fullness, nausea, belching after meals.These symptoms have been
observed for about 15 years. Objectively: body temperatureis 36, 4oC, respiratory rate-20/min,
Ps-88/min, blood pressure-115/75 mm Hg. Skin and mucous membranes are pale. Blood test
results:RBC- 2,0•1012/l, Hb-100g/l.Tests revealed parietal- cell antibodies. What is the most likely
reason for the development of anemia in this patient?
A. * Production of antibodies to intrinsic factor
B. Disruption of hemoglobin synthesis
C. Disruption of erythropoietin synthesis
D. Impaired iron absorption
E. Increased lossofiron
83. A 63-year-old male patient with persistent atrial fibrillation complains of moderate dyspnea.
Objectively: peripheral edemata are absent, vesicular breathing is present, heart rate-72/min,
AP-140/90 mm Hg.What combination of drugs will be most effective for the secondary prevention
of heart failure?
A. * Beta-blockers, ACE inhibitors
B. Beta-blockers, cardiac glycosides
C. Cardiac glycosides, diuretics
D. Cardiac glycosides, ACE inhibitors
E. Diuretics, beta-blockers
84. A 64-year-old patient has been hospi-talised with complaints of progressive jaundi-ce that
developed over 3 weeks ago without pain syndrome, along with general weakness, loss of appetite.
Objectively: temperature is 36, 8oC , heart rate is 78/min., abdomen is soft and painless,
peritoneum irritation symptoms are not detected, palpation reveals sharply enlarged tense
gallbladder. What disease can be characterised with these symptoms?
A. * Cancer of pancreas head

B. Duodenal ulcer
C. Acute cholecystitis
D. Chronic cholecystitis
E. Cholecystitis caused by lambliasis
85. A 64-year-old patient has been hospitalised with complaints of progressive jaundice that developed
over 3 weeks ago without pain syndrome, general weakness, loss of appetite. Objectively:
temperature is 36, 8oC , heart rate is 78/min, abdomen is soft and painless, peri-toneum irritation
symptoms are not detected, palpation reveals sharply enlarged tense gallbladder. What disease can
be characterised with these symptoms?
A. * Cancer of pancreas head
B. Duodenal ulcer
C. Acute cholecystitis
D. Chronic cholecystitis
E. Cholecystitis caused by lambliasis

You might also like